Sei sulla pagina 1di 152

1

Naxos syndrome is characterized by a right sided cardiomyopathy, wooly hair, and keratoderma. The
epidermal structure defective in Naxos syndrome is:
1

Desmoglein 1
2

Desmoglein 3
3

Plakoglobin
4

Desmoplakin
5

Keratin 1/10
Q/Q(M)-477949 Report a Problem


Naxos syndrome is characterized by a right sided cardiomyopathy, wooly hair, and keratoderma. The
epidermal structure defective in Naxos syndrome is:
3

Plakoglobin
Plakoglobin is an intracellular desmosomal component which binds desmogleins/desmocollins on one
side and to desmoplakin on the other. Desmoplakin in turn binds to the keratin intermediate filaments,
K1/10 in most cases. Mutation of desmoplakin leads to CarvajaL syndrome, which is associated with a
striate palmoplantar keratoderma, woolly hair and Left sided cardiomyopathy. A simple way to
remember this is the L in Carvajal cooresponds to the Left sided cardiomyopathy vs. the right sided
disease in Naxos disease.
Q/Q(M)-477949 Report a Problem


A 45 year old woman with history of uterine fibroids presents with painful firm papules of the left
upper arm. Biopsy of one of these lesions reveals smooth muscle fascicles. She notes a history of renal
cell cancer in her family. The gene implicated in this syndrome has which of the following functions?
1

Telomere maintenance
2

Involved in the Kreb's cycle
3

Inhibits mTOR signalling
4

Impairs folliculin function
5

Encodes a deubiquitinating enzyme
Q/Q(M)-482522 Report a Problem


A 45 year old woman with history of uterine fibroids presents with painful firm papules of the left
upper arm. Biopsy of one of these lesions reveals smooth muscle fascicles. She notes a history of renal
cell cancer in her family. The gene implicated in this syndrome has which of the following functions?
2

Involved in the Kreb's cycle
The patient scenario describes Reed's syndrome (multiple cutaneous and uterine leiomyomatosis)
characterized by uterine and cutaneous leiomyomas, in addition to papillary renal cell cancer. This
autosomal dominant disorder is due to mutations within the fumarate hydratase gene, which catalyses
the conversion of fumarate to malate in the Kreb\'s cycle. It is also thought to act as a tumor suppressor
gene. With regards to the other answer choices, telomere maintenance is disrupted in dyskeratosis
congenita. The tuberous sclerosis genes hamartin (TSC1) and tuberin (TSC2) inhibit mTOR signalling.
Mutations in the FLCN gene, with encodes folliculin (thought to act as a tumor suppressor), cause Birt
Hogg Dube syndrome. The CYLD gene, a deubiquitinating enzyme, has been implicated in Brooke-
Spiegler syndrome.
2

Q/Q(M)-482522 Report a Problem


Which of the following pairs of diseases are caused by mutations in DNA helicases?
1

Bloom syndrome and Rothmund-Thompson syndrome
2

Xeroderma pigmentosum and Cockayne syndrome
3

Muir-Torre syndrome and Birt-Hogg-Dube syndrome
4

Dyskeratosis congenita and Peutz-Jeghers syndrome
5

Cowden syndrome and Bannayan-Riley-Ruvalcaba syndrome
Q/Q(M)-474293 Report a Problem


Which of the following pairs of diseases are caused by mutations in DNA helicases?
1

Bloom syndrome and Rothmund-Thompson syndrome
Bloom Syndrome and Rothmund-Thompson syndromes are caused by autosomal recessive mutations
in DNA helicases. The common features include photodistributed poikiloderma. Patients with Bloom
syndrome also demonstrate facial dysmorphism, hypogammaglobulinemia with recurrent respiratory
and gastrointestinal tract infections, hypogonadism, leukemias and lymphomas, gastrointestinal
adenocarcinomas, and oral/esophageal SCCs. Patients with Rothmund-Thompson syndrome develop
premalignant acral keratoses, alopecia, nail dystrophy, cataracts, hypogonadism, and occasional solid
tumors.
Q/Q(M)-474293 Report a Problem

Which of the following diseases is caused by a mutation in a gap junction protein?
1

Striated PPK
2

Schopf-Schulz-Passarge syndrome
3

Mal de Meleda
4

Vohwinkel syndrome (classic)
5

Vohwinkel syndrome (ichthyotic)
Q/Q(M)-474248 Report a Problem

Which of the following diseases is caused by a mutation in a gap junction protein?
4

Vohwinkel syndrome (classic)
Classic Vohwinkel syndrome is caused by mutations in connexin 26, a gap junction protein. Ichthyotic
Vohwinkel syndrome is caused by mutations in loricrin and has ichthyosis but not deafness.
Q/Q(M)-474248 Report a Problem

Which of the following hereditary skin disorders is associated with the RAS-ERK-MAPK pathway?
1

Costello syndrome
2

Rothmund-Thompson Syndrome
3

Carney complex
3

4

Tuberous Sclerosis
5

Griscelli syndrome
Q/Q(M)-482510 Report a Problem


Which of the following hereditary skin disorders is associated with the RAS-ERK-MAPK pathway?
1

Costello syndrome
Skin disorders associated with the RAS-ERK-MAPK pathway include: Cardio-facio-cutaneous
syndrome, Costello, LEOPARD, NF, and Noonan. Rothmund-Thompson is a RecQ DNA helicase
defect. Carney complex and Tuberous sclerosis are associated with the cAMP & AMP activated protein
kinase pathway. Griscelli syndrome involves defective vesicle trafficking/transport.
Q/Q(M)-482510 Report a Problem

Connexin 30 (GJB6 gene) is defective in which of the following syndromes?
1

KID syndrome
2

Vohwinkel syndrome
3

Vohwinkel syndrome variant
4

Clouston syndrome
5

Erythrokeratoderma variabilis
Q/Q(M)-477954 Report a Problem



Connexin 30 (GJB6 gene) is defective in which of the following syndromes?
4

Clouston syndrome
Clouston syndrome is associated with a defect in Connexin 30 (GJB6 gene). Findings include
palmoplantar keratoderma with transgradiens, dystrophic nails, sparse hair with absent body, eyelash,
eyebrow hair after puberty. KID syndrome and Vohwinkel syndrome are associated with a defect in
Connexin 26 (GJB2). Vohwinkel syndrome variant is associated with a loricrin defect.
Erythrokeratoderma variabilis has mutations in Connexin 31 (GJB3) and 30.3(GJB4).
Q/Q(M)-477954 Report a Problem


Pseudoxanthoma elasticum (PXE) can be transmitted in an autosomal dominant, recessive or sporadic
manner. Which of the following genes is mutated in PXE?
1

ABCC6
2

MAN1
3

Collagen III
4

Collagen V
5

Collagen I
Q/Q(M)-477880 Report a Problem
4


Pseudoxanthoma elasticum (PXE) can be transmitted in an autosomal dominant, recessive or sporadic
manner. Which of the following genes is mutated in PXE?
1

ABCC6
ABCC6 is mutated in PXE. Other findings include fragmented and calcified elastin in skin, eyes,
arteries. There is the appearance of plucked chicken skin on the flexures and yellow papules on the
mucous membranes. Angioid streaks are present in the eye. Other findings include gastric hemorrhage
and arterial disease. The others are not involved in PXE.
Q/Q(M)-477880 Report a Problem

A middle-aged woman complains of multiple rough lesions on her trunk. Biopsy reveals connective
tissue nevi. Because other family members have had similar lesions, genetic testing is performed,
revealing a defect in the LEMD3 gene. What is the classic bone finding in patients with a defective
LEMD3 gene?
1

Osteopoikilosis
2

Osteopathia striata
3

Tibial bowing
4

Lumbar ankylosis
5

Frequent fractures of the long bones
Q/Q(M)-482615 Report a Problem


A middle-aged woman complains of multiple rough lesions on her trunk. Biopsy reveals connective
tissue nevi. Because other family members have had similar lesions, genetic testing is performed,
revealing a defect in the LEMD3 gene. What is the classic bone finding in patients with a defective
LEMD3 gene?
1

Osteopoikilosis
BuschkeOllendorff syndrome is a rare genetic syndrome characterized by multiple connective
tissue nevi and osteopoikilosis. Osteopoikilosis is dysplasia of bone leading to the presence of multiple
bone islands in the skeleton. The defective gene is LEMD3. Osteopathia striata is associated with focal
dermal hypoplasia. Tibial bowing is seen in rickets. Lumbar ankylosis is found in ankylosing
spondylitis. Frequent fractures of long bones is seen in osteogenesis imperfecta.
Q/Q(M)-482615 Report a Problem

Which of the following is the first symptom of ataxia telangiectasias?
1

Conjunctival telangictases
2

Facial telangiectases
3

Hematologic malgignancy
4

Cerebellar ataxia
5

Breast cancer
Q/Q(M)-474265 Report a Problem


Which of the following is the first symptom of ataxia telangiectasias?
5

4

Cerebellar ataxia
Ataxia Telangiectasia (Louis-Bar syndrome) is an autosomal recessive disorder usually caused by
mutations in the ATM gene, which is a chromosomal strand break repair enzyme. Cerebellar ataxia is
the first sign, followed by telangiectases of the conjunctiva and skin. Thymic hypoplasia predisposes to
increased infections. There is increased sensitivity to ionizing radiation resulting in hematologic and
solid tumors. Female carriers have increased risk of breast cancer.
Q/Q(M)-474265 Report a Problem
A patient presents with focal symmetric palmoplantar keratoderma, thickened, hyperkeratotic
fingernails and toenails with a "pincer" appearance and frequent staph and candida paronychial
infections, follicular hyperkeratosis of the knees and elbows and oral leukokeratosis. The patients
mother and grandfather both have similar skin findings. What is the defect in PC type I?
1

Keratin 6a/16
2

Keratin 6b/17
3

Keratin 1/10
4

Keratin 2e/10
5

None of these options are correct
Q/Q(M)-481621 Report a Problem


A patient presents with focal symmetric palmoplantar keratoderma, thickened, hyperkeratotic
fingernails and toenails with a "pincer" appearance and frequent staph and candida paronychial
infections, follicular hyperkeratosis of the knees and elbows and oral leukokeratosis. The patients
mother and grandfather both have similar skin findings. What is the defect in PC type I?
1

Keratin 6a/16
Keratin 6a/16 are defective in PC type I. K6b17 is defective in PC type II, K1/10 in Unna-Thost PPK
and epidermolytic hyperkeratosis, K2e/10 in Ichthyosis bullosa of Siemens.
Q/Q(M)-481621 Report a Problem


A child presents with sparse, short hair and sensorineural deafness. On microscopic examination of the
hair, pili torti is noted. Which of the following syndromes is the most likely diagnosis?
1

Bjornstad syndrome
2

Menkes kinky hair syndrome
3

Argininosuccinic aciduria
4

Trichothiodystrophy
5

None of the options are correct
Q/Q(M)-477947 Report a Problem

A child presents with sparse, short hair and sensorineural deafness. On microscopic examination of the
hair, pili torti is noted. Which of the following syndromes is the most likely diagnosis?
1

Bjornstad syndrome
Bjornstad syndrome is the most likely diagnosis. This rare syndrome (~25 cases) is autosomal
recessive. Findings are of deafness and pili torti. The most common hair finding in Menkes syndrome is
pili torti, but it is not associated with hearing loss. Argininosuccinic aciduria is associated with
6

trichorrhexis nodosa and has no associated hearing loss.
Q/Q(M)-477947 Report a Problem


A patient presents with focal symmetric palmoplantar keratoderma, thickened, hyperkeratotic
fingernails and toenails with a "pincer" appearance and frequent staph and candida paronychial
infections, follicular hyperkeratosis of the knees and elbows and oral leukokeratosis. The patients
mother and grandfather both have similar skin findings. Which syndrome is described?
1

Jadassohn-Lewandowsky syndrome
2

Jackson-Lawler syndrome
3

Schafer-Branauer syndrome
4

Pachyonychia congenita tarda
5

None of the options are correct
Q/Q(M)-481620 Report a Problem


A patient presents with focal symmetric palmoplantar keratoderma, thickened, hyperkeratotic
fingernails and toenails with a "pincer" appearance and frequent staph and candida paronychial
infections, follicular hyperkeratosis of the knees and elbows and oral leukokeratosis. The patients
mother and grandfather both have similar skin findings. Which syndrome is described?
1

Jadassohn-Lewandowsky syndrome
The description above is the classic description for pachyonychia congenita (PC) type I or Jadassohn-
Lewandowsky syndrome. Jackson-Lawler syndrome is PC type II, Schafer-Branauer syndrome is PC
type III and pachyonychia congenita tarda is PC type IV.
Q/Q(M)-481620 Report a Problem

What is this syndrome which is histologically characterized by widely dispersed granular material
amidst normal fibers?
1

Ehlers Danlos Syndrome
2

Pseudoxanthoma Elasticum
3

Buschke-Ollendorf Syndrome
4

Focal Dermal Hypoplasia
5

Lipoid Proteinosis
Q/Q(M)-476806 Report a Problem
7



What is this syndrome which is histologically characterized by widely dispersed granular material
amidst normal fibers?
2

Pseudoxanthoma Elasticum
Pseudoxanthoma elasticum is genodermatosis characterized by redundant skin, angioid streaks, yellow
papules on the mucous membranes and bleeding from gastric artery. On histology, readily apparent
denerative changes of the elastic fibers are prominent, even without special stains.
Q/Q(M)-476806 Report a Problem

Hypoplasia of the breast can be seen in which disease?
1

Anhidrotic ectodermal dysplasia
2

Maffucci syndrome
3

Congenital syphilis
4

Marfan syndrome
5

Osteogenesis imperfecta
Q/Q(M)-477180 Report a Problem

Hypoplasia of the breast can be seen in which disease?
1

Anhidrotic ectodermal dysplasia
Anhidrotic ectodermal dysplasia is a X-linked recessive disease caused by mutations in ectodysplasin, a
member of the tumor necrosis family. Patients may have dry skin with pigmentation periorbitally,
hypohidrosis, sparse hair, hypo-anodontia, nail dystrophy, and frontal bossing, and saddle nose
deformity. In addition to abnormalities of other ectodermally derived structures, the breast and nipple-
areolar complex may be absent or hypoplastic.
Q/Q(M)-477180 Report a Problem

Underlying defect for the disease shown in picture is
1

ATP2A2
2

ATP2C1
3

BPAG1
4

BPAG2
8

5

Collagen type 17
Q/Q(M)-482120 Report a Problem



Underlying defect for the disease shown in picture is
2

ATP2C1
The disease shown in image is Hailey-Hailey disease (Familial Benign Pemphigus) which is an
autosomal dominant genodermatosis, caused by mutation in ATP2C1, encoding a calcium pump
protein related to SERCA2. It is characterized by recurrent vesicles and erosions, which most
commonly appear on the sides and back of the neck, in the axillae, in the groin, and in the perianal
regions. The disorder is not seen before puberty and usually has its onset in the late teens or early 20s.
In the intertriginous area lesions tend to form erythematous plaques with dry crusting and soft, flat, and
moist granular vegetations. Burning or pruritus is common, and, particularly in the intertriginous areas,
lesions tend to become irritating, painful, and exceedingly uncomfortable. ATP2A2 is underlying
defect in Darier's disease, other choices are defects seen in pemphigoid and epidermolysis bullosa.
Q/Q(M)-482120 Report a Problem

What nail change is seen in patients with Mal de Meleda Syndrome?
1

Onycholysis
2

Longitudinal ridging
3

Koilonychia
4

leukonychia
5

pterygium
Q/Q(M)-478197 Report a Problem

What nail change is seen in patients with Mal de Meleda Syndrome?
3

Koilonychia
Mal de Meleda is an autosomal recessive disease characterized by transgedient malodorous PPK,
hyperhidrosis, keratotic plaques at knees and elbows, subungual hyperkeratosis, and koilonychia. The
gene defect is SLURP 1.
Q/Q(M)-478197 Report a Problem

Ichthyosis hystrix is characterized by the following gene defects?
1

Keratins 1 and 9
2

Keratins 1 and 10
3

Keratins 5 and 14
9

4

Keratins 6 and 16
5

None of these answers are correct
Q/Q(M)-477852 Report a Problem


Ichthyosis hystrix is characterized by the following gene defects?
2

Keratins 1 and 10
Ichthyosis hystrix or extensive epidermal nevi occurs secondary to a somatic mosaicism for keratins 1
and 10. If the mosaicism occurs on gonadal cells, offspring may have full blown epidermolytic
hyperkeratosis (EHK).
Q/Q(M)-477852 Report a Problem

An infant girl of short stature and shortened 4th and 5th metacarpals is being evaluated for coarctation
of the aorta and horseshoe kidneys. Physical examination most likely reveals:
1

Webbed neck
2

Alopecia
3

Hemangioma
4

Giant congenital melanocytic nevus
5

Arachnodactyly
Q/Q(M)-474025 Report a Problem

An infant girl of short stature and shortened 4th and 5th metacarpals is being evaluated for coarctation
of the aorta and horseshoe kidneys. Physical examination most likely reveals:
1

Webbed neck
Turners syndrome results from nondysjunction during gametogeneiss leading to the XO genotype.
Clinical features include short stature, redundant neck folds/webbed neck, multiple pigmented nevi,
low set hairline, triangular facies, low-set ears, ptosis, wide-set nipples, shortened 4th and 5th
metacarpals, hypoplasia of lymphatics, coarctation of the aorta, and horseshoe kidneys.
Q/Q(M)-474025 Report a Problem

You are consulted on a patient with possible Nethertons Syndrome. Which location of the body would
most likely have hairs demonstrating trichorrhexis invaginata?
1

Scalp
2

Eyebrow
3

Eyelash
4

All of these answers are correct
5

None of these answers are correct
Q/Q(M)-477877 Report a Problem

You are consulted on a patient with possible Nethertons Syndrome. Which location of the body would
most likely have hairs demonstrating trichorrhexis invaginata?
2

Eyebrow
10

Eyebrow hair is most common site with hairs demonstrating trichorrhexis invaginata.
Q/Q(M)-477877 Report a Problem

Homocystinuria is caused by a defect in:
1

Phenylalanine hydroxylase
2

Biotinidase
3

Holocarboxylase synthetase
4

Cystathione beta-synthetase
5

Gp91-phox
Q/Q(M)-478066 Report a Problem

Homocystinuria is caused by a defect in:
4

Cystathione beta-synthetase
Cystathione beta-synthetase is defective in homocystinuria, an autosomal recessive conditions
characterized by increased homocystine and methionine levels in blood and urine. Other findings
include a malar flush, DVTs/emboli, cardiovascular disease, livedo reticularis, leg ulcers, blonde
hair/fiar complexion, downward lens dislocation, glaucoma, mental retardation, seizures, psychiatric
disorders and a marfanoid body habitus. The other enzymes are not involved in this condition.
Q/Q(M)-478066 Report a Problem

Which of the following laboratory test might prove useful in the diagnosis of Fabry disease:
1

Complete blood count with differential
2

Fasting lipids
3

Urinary sediment exam with polarizing light microscopy
4

Stool guaiac
5

Bleeding time
Q/Q(M)-474059 Report a Problem

Which of the following laboratory test might prove useful in the diagnosis of Fabry disease:
3

Urinary sediment exam with polarizing light microscopy
Patients with Fabry disease have a defect in the alpha-galactosidase A enzyme, leading to an
accumulation of glycosphingolipids in all tissues. Although patients are at increased risk for myocardial
infrctions and strokes, the serum lipid levels are normal. Ischemic events occur as a result of
glycosphingolipid accumulation in endothelial cells leading to swelling. In the brain, strokes occur
from direct vessel occlusion or stretching and distention of branches of dolichoectatic parent vessels.
Deposits in the kidneys leads to progressive renal failure with urine exam exhibiting proteinuria and
birefringent lipid globules (maltese crosses) seen with polarizing light microscopy.
Q/Q(M)-474059 Report a Problem

Findings of dysplastic nevi and melanoma inherited in an autosomal dominant fashion is linked with
which of the following?
11

1

Breast malignancy
2

Thyroid malignancy
3

Pancreatic malignancy
4

Renal cell carcinoma
5

Colon carcinoma
Q/Q(M)-478022 Report a Problem

Findings of dysplastic nevi and melanoma inherited in an autosomal dominant fashion is linked with
which of the following?
3

Pancreatic malignancy
Familial dysplastic nevi/melanoma syndrome is linked with increased risk of pancreatic cancers and
astrocytomas. There are no reported increased risks for the other types of cancers listed.
Q/Q(M)-478022 Report a Problem

Giant lysosomal granules are seen in which disease?
1

Chediak-Higashi syndrome
2

Griscelli syndrome
3

Piebaldism
4

Incontinentia pigmenti
5

Carney complex
Q/Q(M)-474253 Report a Problem



Giant lysosomal granules are seen in which disease?
1

Chediak-Higashi syndrome
Chediak-Higashi syndrome is caused by an autosomal recessive mutation in a lysosomal transport gene
(LYST, CHS1). This disorder is characterized by oculocutaneous albinism, ataxia, muscle weakness,
and giant lysosomal granules. There is an accelerated phase characterized by lymphohistiocytic
infiltration of reticuloendothelial system, pancytopenia and death.
Q/Q(M)-474253 Report a Problem


Which of the following is true regarding tuberous sclerosis?
1

Confetti-like macules are typically present at birth
2

Facial angiofibromas are the most common cutaneous manifestation
3

Hypomelanotic macules (ash leaf spots) have a decreased number of melanocytes
4

Periungual fibromas are considered a major feature in the diagnosis of tuberous sclerosis
5

6 or more hypomelanotic macules (ash leaf spots) are considered a major feature in the diagnosis
of tuberous sclerosis
Q/Q(M)-482518 Report a Problem
12



Which of the following is true regarding tuberous sclerosis?
4

Periungual fibromas are considered a major feature in the diagnosis of tuberous sclerosis
The earliest and most common cutaneous manifestation of tuberous sclerosis are hypomelanotic
macules (ash leaf spots), typically presenting at birth or early infancy. 3 or more of these lesions are
considered a major criteria in diagnosis. Melanocyte numbers are normal. Confetti-like macules, on the
other hand, are usually not apparent until the second decade of life. Facial angiofibromas occur in
approximately 75% of patients, and tend to become more prominent with age. Periungual fibromas are
considered a major feature for diagnosis, usually presenting around puberty to early adulthood.
Q/Q(M)-482518 Report a Problem

In one variant of epidermolysis bullosa simplex, those affected have muscular dystrophy in addition to
the skin findings. Which protein mutation has been linked to this finding?
1

Plakoglobin
2

Plakophilin
3

Plectin
4

Desmoglein
5

Desmocollin
Q/Q(M)-477710 Report a Problem

In one variant of epidermolysis bullosa simplex, those affected have muscular dystrophy in addition to
the skin findings. Which protein mutation has been linked to this finding?
3

Plectin
A plectin mutation is linked with this subtype of epidermolysis bullosa simplex.
Q/Q(M)-477710 Report a Problem

Familial macular and lichen amyloidosis is a feature of which of the following conditions?
1

Sipple syndrome
2

Peutz-Jeghers syndrome
3

Marfan syndrome
4

Dyskeratosis congenita
5

Birt-Hogg-Dube syndrome
Q/Q(M)-474292 Report a Problem


Familial macular and lichen amyloidosis is a feature of which of the following conditions?
1

Sipple syndrome
Sipple syndrome (MEN 2a) is caused by autosomal dominant mutations in the ret protooncogene.
Patients develop parathyroid cancers, pheochromocytomas, and medullary cancer of the thyroid gland.
Familial macular and lichen amyloidosis is also a feature of this syndrome.
13

Q/Q(M)-474292 Report a Problem

Mutations in which of the following receptors underlie chronic mucocutaneous candidiasis disease
(CMCD)?
1

IL-12 receptor
2

IL-15 receptor
3

AIRE receptor
4

IL-17 receptor
5

IL-23 receptor
Q/Q(M)-482535 Report a Problem


Mutations in which of the following receptors underlie chronic mucocutaneous candidiasis disease
(CMCD)?
4

IL-17 receptor
Chronic mucocutaneous candidiasis disease (CMCD) is characterized by recurrent or persistent
infections of the skin, nails, and oral and genital mucosae caused by Candida albicans and, to a lesser
extent, Staphylococcus aureus, in patients with no other infectious or autoimmune manifestations.
Mutations in IL-17 receptor A (IL-17RA, autosomal recessive) and IL-17 receptor F (IL-17F,
autosomal dominant) have been reported. IL-17RA deficiency is complete, abolishing cellular
responses to IL-17A and IL-17F homo- and heterodimers. By contrast, IL-17F deficiency is partial,
with mutant IL-17F-containing homo- and heterodimers displaying impaired, but not abolished,
activity.
Q/Q(M)-482535 Report a Problem





A patient with 20 nail dystrophy, steatocystoma multiplex and natal teeth likely has a mutation in the
genes coding for:
1

Keratins 5
2

Laminin 5
3

Plakophilin 1
4

Keratins 6b & 17
5

Keratins 6 &16
Q/Q(M)-474052 Report a Problem


A patient with 20 nail dystrophy, steatocystoma multiplex and natal teeth likely has a mutation in the
genes coding for:
4

Keratins 6b & 17
Pachyonychia congenital is an autosomal dominant condition with 20 nail dystrophy. The patient
described has Type II (Jackson-Sertole) disease, which includes steatocystoma multiplex, natal teeth,
multiple cysts, and micropthalmia, and is caused by mutations in keratins 6b& 17. Type I (Jadassohn-
14

Lewandowsky) also includes focal symmetric PPK, follicular hyperkeratosis, oral leukokeratoses and is
caused by mutations in keratins 6 &16. Type III includes the clinical features of type I + corneal
leukokeratosis. Mutations in keratins 5&14 represents EB simplex, Laminin 5 mutation is seen in
Junctional EB, and plakophilin 1 mutation is seen in ectodermal dysplasia with skin fragility.
Q/Q(M)-474052 Report a Problem



Which opthamologic disease is associated with this disorder?
1

Glaucoma
2

Ectopia lentis
3

Cataracts
4

Posterior subcasular lentiular opacity
5

Retinitis pigmentosa
Q/Q(M)-476643 Report a Problem


Which opthamologic disease is associated with this disorder?
1

Glaucoma
Sturge-Weber syndrome is a sporadic disease characterized by a capillary malformation in the
trigeminal distribution. Patients may have associated cerebral atrophy, vascular malformations of the
leptomeninges, and seizures. All patients with Sturge-Weber should be referred to the opthamologist
for glaucoma screening.
Q/Q(M)-476643 Report a Problem

Cutaneous meningiomas have been associated with what syndrome?
1

Neurofibromatisis
2

Cowden
3

Neurocutaneous melanosis
4

Gorlin
5

Glomangiomatosis
Q/Q(M)-482216 Report a Problem
15


Cutaneous meningiomas have been associated with what syndrome?
1

Neurofibromatisis
Cutaneous meningiomas are sperated into 3 types. Type 1 are thought to be misplaced meningeal cells,
or rudimentary mengioceles. Type II develop along the course of cranial nerves. Type III lesions
represent cutaneous metastasis or an underlying primary meningioma. Cutaneous meningiomas have
been associated with cranial developmental anomalies and in neurofibromatosis.
Q/Q(M)-482216 Report a Problem


Exposure to what medicine in-utero has been most closely associated with aplasia cutis congenita?
1

Methimazole
2

Levothyroxine
3

Magnesium
4

Isotretinoin
5

Trimethoprim/sulfamethoxazole
Q/Q(M)-482704 Report a Problem

Exposure to what medicine in-utero has been most closely associated with aplasia cutis congenita?
1

Methimazole
Aplasia cutis congenita presents as well-demarcated erosions at birth with heal with atrophic, alopecic
scars. It can be inherited in autosomal dominant, autosomal recessive, or sporadic forms. Aplasia cutis
congenia can be caused by teratogens, particular methimazole.
Q/Q(M)-482704 Report a Problem

Which genetic defect could explain cutaneous findings in addition to abnormal immunoglobulin levels,
recurrent respiratory infections, hypogonadism, and an increased risk of leukemia and lymphoma?
1

RecQL3
2

ERCC6
3

WAS gene
4

NADPH oxidase
5

Adenosine deaminase
Q/Q(M)-476540 Report a Problem



Which genetic defect could explain cutaneous findings in addition to abnormal immunoglobulin levels,
recurrent respiratory infections, hypogonadism, and an increased risk of leukemia and lymphoma?
1

RecQL3
Bloom's syndrome is an autosomal recessive disorder caused by mutations in the RecQL3 gene
encoding a DNA helicase. Clinically, individuals with Bloom's syndrome have a photodistributed
erythema with telangectasia on the malar eminences. The may also have decreased IgM and IgA levels,
16

hypogonadism, and an increased risk for leukemia and lymphoma.
Q/Q(M)-476540 Report a Problem


Which of the following is caused by a defect in a gap junction protein?
1

Epidermolysis bullosa simplex
2

Hailey-hailey
3

Erythrokeratoderma variabilis
4

Dyskeratosis congenita
5

Bullous ichthyosis of siemens
Q/Q(M)-474024 Report a Problem



Which of the following is caused by a defect in a gap junction protein?
3

Erythrokeratoderma variabilis
Erythrokeratoderma variabilis is also known as Mendes da Costa Syndrome. It is caused by a defect in
connexin 31, a gap junction protein. EB simplex is caused by a mutation in keratins 5 & 14, Hailey-
Hailey is caused by a mutation in calcium transporters, dyskeratosis congenita is caused by a defect in
rRNA synthesis, and bullous icthyosis of Siemens is caused by mutations in keratin 2e.
Q/Q(M)-474024 Report a Problem

A patient with myotonic dystrophy and multiple skin lesions most likely has activating mutations in
which of the following?
1

Beta-catenin
2

Desmoplakin
3

Plakoglobin
4

Desmoglein
5

Alpha 6-beta 4 integrin
Q/Q(M)-477996 Report a Problem

A patient with myotonic dystrophy and multiple skin lesions most likely has activating mutations in
which of the following?
1

Beta-catenin
Myotonic dystrophy with multiple pilomatricomas is described above. Activating mutations in Beta-
catenin are found in this syndrome. The other listed options are desmosomal proteins and are not
involved in this syndrome.
Q/Q(M)-477996 Report a Problem

Epidermolysis bullosa simplex is caused by blistering in which structure?
1

Granular layer keratinocyte
2

Spinous layer keratinocyte
17

3

Basal layer keratinocyte
4

Lamina densa
5

Sublamina densa
Q/Q(M)-474233 Report a Problem


Epidermolysis bullosa simplex is caused by blistering in which structure?
3

Basal layer keratinocyte
Epidermolysis bullosa simplex is caused by mutations in keratin 5 and 14 resulting in bullae within
basal cell keratinocytes.
Q/Q(M)-474233 Report a Problem



Which of the following is caused by a defect in lysosomal transport:
1

oculocutaneous albinism I
2

oculocutaneous albinism II
3

Piebaldism
4

Chediak-higashi
5

Blooms syndrome
Q/Q(M)-474020 Report a Problem


Which of the following is caused by a defect in lysosomal transport:
4

Chediak-higashi
Chediak higashi is an autosomal recessive disorder caused by a mutation in the LYST gene codes for a
lysosomal tracking protein. This protein regulates microtubule mediated lysosomal fusion. A defect in
this gene leads to giant lysosomal granules seen in neutrophils (leading to defecting phagocytosis and
decreased chemotaxis), melanocytes (pigment dilution), and neurons. OCA1 is tyrosinase negative
albinism; OCA2 is tyrosinase positive albinism with a mutation in P gene on chromosome 15.
Q/Q(M)-474020 Report a Problem


Which ocular finding may be seen in a patient with this skin condition?
1

Comma-shaped corneal opacities
2

Retinitis pigmentosa
3

Congenital hypertrophy of the retinal pigmented epithelium
4

Angioid streaks
5

Pingueculae
Q/Q(M)-476807 Report a Problem
18




Which ocular finding may be seen in a patient with this skin condition?
4

Angioid streaks
Pseudoxanthoma elasticum is caused by a defect in connective tissue. Angioid streaks develop when a
rupture occurs in Bruch's membrane.
Q/Q(M)-476807 Report a Problem

Which of the following syndromes is associated with markedly increased IgE levels, cold abscesses and
a characteristic coarse facies?
1

Wiskott-Aldrich syndrome
2

Chronic granulomatous disease
3

Job syndrome
4

Severe combined immunodeficiency
5

Leiner's disease
Q/Q(M)-478040 Report a Problem

Which of the following syndromes is associated with markedly increased IgE levels, cold abscesses and
a characteristic coarse facies?
3

Job syndrome
Job syndrome or Hyper IgE syndrome is characterized by these findings. In addition, there is a
peripheral eosinophilia, eczematous dermatitis, frequent bronchitis and pneumonia, otitis media and
sinusitis. The other listed conditions are associated with immunodeficiency. Wiskott-Aldrich can have
eczematous dermatitis and all of these syndromes will have abnormal infections. They do not have
markedly increased levels of IgE like Job syndrome.
Q/Q(M)-478040 Report a Problem



A 2 year old female patient has linear atrophy in a Blasckoid distribution, alopecia, nail dystrophy,
abnormal teeth, and colobomas. What is the inheritance pattern of this condition?
1

X-linked dominant
2

X-linked recessive
3

Autosomal dominant
4

Autosomal recessive
5

Mitochondrial
Q/Q(M)-482702 Report a Problem
19


A 2 year old female patient has linear atrophy in a Blasckoid distribution, alopecia, nail dystrophy,
abnormal teeth, and colobomas. What is the inheritance pattern of this condition?
1

X-linked dominant
Focal dermal hypoplasia, or Goltz syndrome, is an x-linked dominant condition with a defect in the
PORCN gene. It is characterized by linear atrophy in a Blasckoid distribution, alopecia, nail dystrophy,
abnormal teeth, and colobomas.
Q/Q(M)-482702 Report a Problem

What protein is deficient in the condition shown?
1

Calcium ATPase IIC1
2

Calcium ATPase IIA2
3

PEX-7
4

SPINK5
5

Desmoglein 3
Q/Q(M)-477715 Report a Problem




What protein is deficient in the condition shown?
1

Calcium ATPase IIC1
The picture shown is Hailey-Hailey disease. This is an autosomally dominant condition with a defect in
Calcium ATPase IIC1. On H&E stain, an acantholytic dilapidated brick wall appearance is seen.
Calcium ATPase IIA2 is defective in Dariers Disease, PEX-7 in autosomal recessive type Conradi-
Hunermann disease, SPINK5 in Nethertons disease and Desmoglein 3 in pemphigus vulgaris.
Q/Q(M)-477715 Report a Problem

Which of the following syndromes is characterized by follicular atrophoderma, hypohidrosis,
hypotrichosis and multiple basal cell carcinomas?
1

Bazex syndrome
2

Rombo syndrome
3

Rasmusen syndrome
20

4

Gorlin syndrome
5

Incontinentia Pigmenti
Q/Q(M)-477989 Report a Problem

Which of the following syndromes is characterized by follicular atrophoderma, hypohidrosis,
hypotrichosis and multiple basal cell carcinomas?
1

Bazex syndrome
Bazex syndrome has the findings of follicular atrophoderma, hypohidrosis, hypotrichosis and multiple
basal cell carcinomas (BCC). Rombo syndrome is associated with BCC and hypotrichosis, but not the
other listed findings. The atrophoderma in Rombo syndrome is vermicular, not follicular. Rasmusen
syndrome is not associated with BCC.
Q/Q(M)-477989 Report a Problem

The nucleotide excision DNA repair pathway is defective in which disease:
1

Bourneville's disease
2

Severe combined deficiency syndrome
3

Griscelli syndrome
4

Xeroderma pigmentosa
5

Sjogren-Larssen syndrome
Q/Q(M)-478581 Report a Problem

The nucleotide excision DNA repair pathway is defective in which disease:
4

Xeroderma pigmentosa
The pathogensis of Xeroderma Pigmentosum shows mutations i genes encoding DNA repair enzymes,
leading to defective DNA excision repair upon exposure to UV radiation. Severe combined deficiency
syndrome-major defect in cell-mediated and humoral immunity; most lack antibody-dependent cellular
cytotoxicity and natural killer cell function. The pathogensis for Griscelli Syndrome is a mutation in
gene encoding for myosin Va or RAB27 a. Sjogren-Larsson Syndrome has mutations in the FALDH
gene. Bourneville's syndrome (AKA Tuberous Sclerosis) shows a mutation in either TSC1 ancoding
hamartin or TSC2 encoding tuberin.
Q/Q(M)-478581 Report a Problem
What gene defect would you expect to find in a child with white forelock, dystopia canthorum, and
upper limb abnormalities?
1

Pax3
2

MITF
3

SOX10
4

Endothelin-3
5

C-kit proto-oncogene
Q/Q(M)-476739 Report a Problem
What gene defect would you expect to find in a child with white forelock, dystopia canthorum, and
upper limb abnormalities?
1

Pax3
21

Waardenburg's syndrome is characterized by depigmented patches, white forelock, and deafness. Both
type 1 and 3 are caused by mutations in Pax3. Type 3 is also associated with limb abnormalities. MITF
and SOX10 defects are responsible for types 2 and 4 respectively. C-kit proto-oncogene mutatios are
seen in piebaldism.
Q/Q(M)-476739 Report a Problem
What condition is associated with this finding of inflammatory keratotic facial papules which may
result in scarring and atrophy?
1

Chloracne
2

Systemic lupus erythematosus
3

Keratosis pilaris
4

Reiter's syndrome
5

Ulerythema ophryogenes
Q/Q(M)-476831 Report a Problem


What condition is associated with this finding of inflammatory keratotic facial papules which may
result in scarring and atrophy?
5

Ulerythema ophryogenes
Ulerythema ophryogenes is a rare disorder that affects children and young adults. It is characterized by
keratosis pilaris atrophicans and loss of lateral third of eyebrow.
Q/Q(M)-476831 Report a Problem

Which of the following disorders is associated with delayed separation of the umbilical cord?
1

Leukocyte adhesion deficiency type 1 (LAD-1)
2

Immunedysregulation, polyendocrinopathy, enteropathy, x-linked (IPEX)
3

Severe combined immunodeficiency disorder(SCID)
4

X-linked agammaglobulinemia
5

Myeloperoxidase deficiency
Q/Q(M)-477478 Report a Problem

Which of the following disorders is associated with delayed separation of the umbilical cord?
1

Leukocyte adhesion deficiency type 1 (LAD-1)
LAD-1 manifests as a B2-integrin deficiency and often times presents as at birth with a delayed
umbilical cord separation.
Q/Q(M)-477478 Report a Problem

Which keratins are expressed in the suprabasal palmoplantar epidermis?
22

1

Keratins 1 and 9
2

Keratins 1 and 10
3

Keratins 4 and 13
4

Keratins 5 and 14
5

Keratins 8 and 18
Q/Q(M)-474251 Report a Problem
Which keratins are expressed in the suprabasal palmoplantar epidermis?
1

Keratins 1 and 9
Keratins are expressed in pairs of acidic and basic keratins and are tissue- and differentiation-specific.
Keratins 1 and 9 are expressed in the suprabasal palmoplantar epidermis, keratins 1 and 10 in
suprabasal nonpalmoplantar epidermis, keratins 2e and 10 in the granular layer, keratins 4 and 13 in
mucosal epithelium, keratins 5 and 14 in the basal layer, and keratins 8 and 18 in simple epithelium.
Q/Q(M)-474251 Report a Problem

Most common malignancy to develop in a patient with tricholemmomas, acral verrucous papules and
cobble-stoning of buccal and gingival mucosa?
1

Thyroid cancer
2

Breast cancer
3

Colon cancer
4

Lymphoma
5

Melanoma
Q/Q(M)-476533 Report a Problem

Most common malignancy to develop in a patient with tricholemmomas, acral verrucous papules and
cobble-stoning of buccal and gingival mucosa?
2

Breast cancer
Cowden's syndrome is an autosomally dominant inherited defect of PTEN. Patient may present with
multiple trichilemmoma, hamartomatous tumors of the breast, thyroid and endometrium, acral
keratoses and papillomatous papules. Breast cancer is the most common cancer to arise while thyroid
carcinoma is the second most common.
Q/Q(M)-476533 Report a Problem

The treatment for acrodermatitis enteropathica is:
1

Zinc supplementation
2

Iron supplementation
3

Vitamin B1 supplementation
4

Vitamin B12 supplementation
5

Phlebotomy
Q/Q(M)-478064 Report a Problem

23

The treatment for acrodermatitis enteropathica is:
1

Zinc supplementation
Acrodermatitis enteropathica is due to a defect in zinc absorption and will respond to zinc
supplementation. Iron, Vitamin B1/12 supplementation will not result in improvement in this condition.
Findings include periorificial, scalp, and acral dermatitis, scaling, vesicles/bullae, erosions, alopecia,
diarrhea and stomatitis.
Q/Q(M)-478064 Report a Problem

A 2 year old patient is small for his age. He has thin, pale, translucent skin, with prominent veins. His
father died from aortic rupture when he was in his twenties. Vascular Ehlers-Danlos is caused by
mutations in what gene?
1

Type III Collagen
2

Tenascin-X
3

Type V Collagen
4

Lysyl hydroxylase
5

Type I Collagen
Q/Q(M)-482613 Report a Problem

A 2 year old patient is small for his age. He has thin, pale, translucent skin, with prominent veins. His
father died from aortic rupture when he was in his twenties. Vascular Ehlers-Danlos is caused by
mutations in what gene?
1

Type III Collagen
Vascular Ehlers-Danlos (Type 4) is characterized by the skin findings discussed above, as well as the
onset of severe complications of fragile blood vessels and organs in early adulthood. The disease is
inherited in an autosomal dominant manner. Defects are seen in type III collagen. Tenascin-X
mutations are seen in hypermobile Ehlers-Danlos (Type 3), type V collagen in classical Ehlers-Danlos
(Type 1 and 2), lysyl hydroxylase in kyphoscoliosis Ehlers-Danlos (Type 6), and type I collagen in
Ehlers-Danlos types 1, 2, and 7.
Q/Q(M)-482613 Report a Problem

Papillon-Lefevre and Haim-Munk syndromes have which of the following symptoms?
1

Right-ventricular cardiomyopathy
2

Pseudoainhum
3

Esophageal cancer
4

Eccrine syringofibradenoma
5

Periodontitis with tooth loss
Q/Q(M)-474249 Report a Problem

Papillon-Lefevre and Haim-Munk syndromes have which of the following symptoms?
5

Periodontitis with tooth loss
Right-ventricular cardiomyopathy is associated with Naxos syndrome, pseudoainhum is associated with
Vohwinkel syndrome, esophageal cancer is associated with Howel-Evans syndrome, and eccrine
syringofibradenomas are associated with Schopf-Schulz-Passarge syndrome. Periodontitis with tooth
24

loss is associated with Papillon-Lefevre and Haim-Munk syndromes, which are caused by mutations in
Cathepsin C.
Q/Q(M)-474249 Report a Problem


Menkes kinky hair syndrome is associated most commonly with which of the following hair
abnormalities?
1

Pili torti
2

Trichorrhexis nodosa
3

Trichorrhexis invaginata
4

Pili triangulati et canaliculati
5

None of these options are correct
Q/Q(M)-477943 Report a Problem

Menkes kinky hair syndrome is associated most commonly with which of the following hair
abnormalities?
1

Pili torti
While pili torti is not exclusively found in Menkes kinky hair syndrome, this is the most common hair
abnormality found. Trichorrhexis nodosa can be also seen. Trichorrhexis invaginata is commonly found
in Netherton syndrome. Pili triangulati et canaliculati is the finding seen in Uncombable hair syndrome.
Q/Q(M)-477943 Report a Problem
The most common cardiovascular defect in patients with Noonan syndrome is:
1

Atrial septal defect
2

Ventricular septal defect
3

Enlarged aorta
4

Pulmonic valve stenosis
5

Aortic stenosis
Q/Q(M)-474019 Report a Problem
The most common cardiovascular defect in patients with Noonan syndrome is:
4

Pulmonic valve stenosis
Noonans syndrome is also known as cardiofaciocutaneous syndrome. Patients have short stature,
ptosis, hypertelorism, low-set ears, thick lips and curly hair. Pulmonic valve stenosis is the most
common cardiovascular defect, with atrial septal defects also seen.
Q/Q(M)-474019 Report a Problem
Patients with homocystinuria are deficient in:
1

Alpha galactosidase A
2

Transglutaminase
3

Lysyl oxidase
4

Cystathionine synthetase
5

Steroid sulfatase
Q/Q(M)-477118 Report a Problem
25


Patients with homocystinuria are deficient in:
4

Cystathionine synthetase
Homocystinuria is an autosomal recessive disease caused by a deficiency of cystathionine synthetase.
Manifestations of this condition may include mental retardation, marfanoid habitus, malar flush,
ectopic lentis, embolic phenomena and leg ulcers.
Q/Q(M)-477118 Report a Problem
Mucosal malignancy is a complication of:
1

Oral hairy leukoplakia
2

White sponge nevus
3

Dyskeratosis congenital
4

Chronic candidiasis
5

Focal epithelial hyperplasia
Q/Q(M)-477157 Report a Problem
Mucosal malignancy is a complication of:
3

Dyskeratosis congenital
Dyskeratosis congenital, also called Zinsser-Engman-Cole syndrome, is an X-linked recessive
genodermatosis caused by a mutation in DKC1. DKC1 encodes for dyskerin, which helps to maintain
telomeres through the pseudouridylation of rRNA. Features of this condition include reticulate gray-
brown hyperpigmentation, dystrophic nails, alopecia and Fanconis type pancytopenia. Patients may
have premalignant leukoplakia which should be followed closely.
Q/Q(M)-477157 Report a Problem
Which of the following bony defect is found in CHILD syndrome?
1

Polyostotic fibrous dysplasia
2

Stippled epiphyses
3

Calcification of falx cerebri
4

Osteopoikilosis
5

Sphenoid wing dysplasia
Q/Q(M)-477743 Report a Problem
Which of the following bony defect is found in CHILD syndrome?
2

Stippled epiphyses
CHILD syndrome is an X-linked dominant syndrome which is lethal in males. It is caused by a
peroxisomal biogenesis disorder. It is characterized by unilateral ichthyosiform erythroderma,
limb/visceral hypoplasia, and stippled epiphyses. Stippled epiphyses can also be seen in
chondrodysplasia punctata. Polyostotic fibrous dysplasia is found in McCune-Albright syndrome,
calcification of falx cerebri in Gorlin's syndrome, osteopoikilosis in seen in Buschke-Ollendorf
syndrome.
Q/Q(M)-477743 Report a Problem


Dyshcromatosis symmetrica hereditaria (DSH) is a disorder characterized by asymptomatic hypo- and
hyperpigmented macules on the face and dorsal extremities. Mutations in which of the following genes
is associated with this condition?
26

1

DKC
2

PTEN
3

ADAR1
4

ERCC4
5

STK11
Q/Q(M)-482538 Report a Problem

Dyshcromatosis symmetrica hereditaria (DSH) is a disorder characterized by asymptomatic hypo- and
hyperpigmented macules on the face and dorsal extremities. Mutations in which of the following genes
is associated with this condition?
3

ADAR1
Dyshcromatosis symmetrica hereditaria (DSH) is a rare autosomal dominant pigmentary disease
characterized by asymptomatic hypopigmented and hyperpigmented macules distributed on the face
and dorsal extremities. It is most common in patients of Asian descent but has also been reported in
patients of European and South American descent. The condition is typically limited to the skin but has
been associated in rare instances with neurological deterioration and brain calcificiations. The lesions
tend to appear in infancy and childhood and stabilize during adolescence. Mutations in the double-
stranded RNA-specific adenosine deaminase gene (ADAR1 or DSRAD) underlie DSH. The ADAR1
protein catalyzes the deamination of adenosine to inosine in double-stranded RNA which is important
for alternative splicing or alternations of codons for protein translation. However, the precise
mechanism by which these mutations lead to actual disease remain unknown. There are two diseases
that are phenoptypically similar to DSH and previously thought to be related. The first is
dyschromatosis universalis hereditaria (DUH) which is characterized by a mixture of hyperpigmented
and hypopigmented macules that are widespread. DSH can be distinguished from DUH by its
localization to the face and distal acral sites. The second disease is acropigmentatio retiticularis (AR,
also known as acropigmentation of Kitamura). AR is also characterized by hyperpigmentated macules
on the dorsal hands and feet as well as palmar pits. However, in contrast to DSH, it is notably devoid of
hypopigmented macules. Suzuki et al. sequenced 20 cases of patients presenting with DSH, DUH, and
AR. All patients with DSH demonstrated mutations in ADAR1, however, none of the patients with
DUH or AR demonstrated mutations. Based on these studies, they proposed that these diseases are in
fact distinct and separate from DSH. There are no effective therapies for DSH. However, it is important
to distinguish DSH from other more serious disorders that can present with pigment alterations such as
dyskeratosis congenita and xeroderma pigmentosum. Both of these conditions present with
photosensitivity and squamous cell carcinomas which are not present in DSH.
Q/Q(M)-482538 Report a Problem


What is the most likely syndrome that this woman has in this photograph?
1

A. Klippel Trenaunay Weber
2

Filariasis
3

Neurofibromatosis
4

Proteus syndrome
5

Turner syndrome
Q/Q(M)-482074 Report a Problem
27



What is the most likely syndrome that this woman has in this photograph?
1

A. Klippel Trenaunay Weber
Klippel-Trenaunay Weber syndrome is characterized by a triad of port-wine stain, varicose veins, and
bony and soft tissue hypertrophy involving an extremity. Filariasis is an acquired infection that leads to
obstruction and scarring of the lymphatics. Neurofibromatosis is associated with neurofibromas, caf-
au-lait macules and Lisch nodules, but not with lymphedema and vascular malformations. Proteus
syndrome is associated with vascular abnormalities, lipomas and asymmetric limb hyperplasia, and
plantar connective tissue nevi, but usually not lymphedema. Lymphedema of the dorsal hands and feet
can be seen in Turner syndrome, but typically not this extensive.
Q/Q(M)-482074 Report a Problem

Menkes kinky hair syndrome is caused by a defect in:
1

Mitochondrial gene
2

DNA helicase
3

Gap junction protein
4

Copper Transporting ATPase
5

Proto-oncogene
Q/Q(M)-474031 Report a Problem

Menkes kinky hair syndrome is caused by a defect in:
4

Copper Transporting ATPase
Menkes kinky hair syndrome is an x-linked recessive disorder caused by a mutation at Xq12 leading
to defective intestinal copper transport.
Q/Q(M)-474031 Report a Problem

Which genetic disease is characterized by round opacities seen on radiographs of the long bones?
1

Buschke-Olendorf syndrome
2

Chondrodysplasia punctata
3

Netherton syndrome
4

Osteogenesis imperfecta
5

Focal dermal hypoplasia
Q/Q(M)-482701 Report a Problem
28


Which genetic disease is characterized by round opacities seen on radiographs of the long bones?
1

Buschke-Olendorf syndrome
Buschke-Ollendorf syndrome is an autosomal dominant disorder with dermatofibrosis lenticularis
disseminata (elastomas) and osteopoikilosis, or round opacities of the bones. It is caused by a defect in
LEMD 3 (aka MAN1), which codes an inner nuclear membrane protein. Focal dermal hypoplasia can
cause osteopathia striata; osteogenesis imperfecta can have bone fractures; and chondrodysplasia
punctata can have stippled epiphyses.
Q/Q(M)-482701 Report a Problem

Which of the following conditions is inherited in an X-linked recessive manner?
1

Epidermolysis bullosa simplex
2

Ichthyosis vulgaris
3

Sjogren-Larsson syndrome
4

Wiskott-Aldrich Syndrome
5

Nethertons Syndrome
Q/Q(M)-477707 Report a Problem

Which of the following conditions is inherited in an X-linked recessive manner?
4

Wiskott-Aldrich Syndrome
Wiskott-Aldrich Syndrome is inherited in an X-linked recessive manner. Epidermolysis bullosa
simplex and ichthyosis vulgaris are inherited in an autosomal dominant (AD) manner. Sjogren-Larsson
and Nethertons syndrome are inherited in an autosomal recessive manner.
Q/Q(M)-477707 Report a Problem
What is the characteristic radiographic finding in type I Gaucher disease?
1

Enchondromas
2

Osteopoikilosis
3

Melorheostosis
4

Ehrlenmeyer flask deformity
5

Supernumerary vertebrae with extra ribs
Q/Q(M)-478046 Report a Problem

What is the characteristic radiographic finding in type I Gaucher disease?
4

Ehrlenmeyer flask deformity
The Ehrlenmeyer flask deformity is found in the femoral midshaft as well as aseptic necrosis of the
femoral head and widening of the distal femur. Endochondromas are seen in Maffucci syndrome,
Osteopoikilosis in Buschke-Ollendorf syndrome, Melorheostosis (linear hyperostosis under affected
skin) in linear scleroderma and supernumerary vertebrae with extra ribs in incontinentia pigmenti.
Q/Q(M)-478046 Report a Problem

A child presents with the hair finding seen in the image in addition to brittle nails, keratosis pilaris,
abnormal teeth and cataracts. Which of the following abnormalities is the most likely mutated?
29

1

Keratin 1/10
2

Keratin hHb1/hHb6
3

Keratin 6/16
4

Keratin 6/17
5

Keratin 2e
Q/Q(M)-477948 Report a Problem



A child presents with the hair finding seen in the image in addition to brittle nails, keratosis pilaris,
abnormal teeth and cataracts. Which of the following abnormalities is the most likely mutated?
2

Keratin hHb1/hHb6
Keratin hHb1/hHb6 is defective in monilethrix, which is described above. Keratin 1/10 defects are
found in epidermolytic hyperkeratosis, K6/16 in inflamed skin and pachyonychia congenita type I,
K6/17 in pachyonychia congenita type II and K2e in Ichythosis bullosa of Siemens.
Q/Q(M)-477948 Report a Problem

Which one of the following is the most common oncogenic virus in patients with epidermodysplasia
verruciformis?
1

HPV-5
2

HPV-8
3

HPV-13
4

HPV-16
5

HPV-33
Q/Q(M)-477317 Report a Problem

Which one of the following is the most common oncogenic virus in patients with epidermodysplasia
verruciformis?
1

HPV-5
Epidermodysplasia verruciformis is a rare autosomal recessive disorder in which an impaired cellular
immunity allows widespread infection with certain subtypes of the human papilloma virus (HPV).
Some of these lesions have a tendency for malignant transformation, most commonly those verruca
caused by HPV type 5.
Q/Q(M)-477317 Report a Problem

Pseudoxanthoma elasticum is caused by mutations in which of the following genes?
30

1

Fibrillin 1
2

Elastin gene
3

Lysyl oxidase
4

ABCC6 gene
5

Collagen 5
Q/Q(M)-474279 Report a Problem

Pseudoxanthoma elasticum is caused by mutations in which of the following genes?
4

ABCC6 gene
Pseudoxanthoma elasticum is caused by mutations in the ABCC6 gene, which is an ATP-using cell
transporter. Elastin and lysyl oxidase mutations cause cutis laxa, fibrillin 1 mutations cause Marfan
syndrome, and collagen 5 mutations cause Ehlers-Danlos syndrome.
Q/Q(M)-474279 Report a Problem
Which of the following features is not associated with Cornelia de Lange Syndrome?
1

Normal intelligence
2

Characteristic facies with downturned mouth, hirsutism, synophrys, trichomegaly, anteverted
nostrils, long philtrum and low set ears
3

Cryptorchidism
4

Fifth finger clinodactyly
5

Recurrent lung infections
Q/Q(M)-477815 Report a Problem

Which of the following features is not associated with Cornelia de Lange Syndrome?
1

Normal intelligence
Children with Cornelia de Lange are usually severly retarded with an IQ <35. In addition to the features
listed above, other features include cutis marmorata, hypoplastic nipples and umbilicus, low-pitched
cry in infancy and congenital heart defects. While most cases are inherited in a sporadic manner, those
cases which are familial are thought to be autosomal dominant and associated with the NIPBL (nipped-
beta-like) gene. Prognosis is poor with premature death often secondary to sspiration or recurrent
pulmonary infection.
Q/Q(M)-477815 Report a Problem

A child presents with pretibial hyperpigmentation, ataxia, decreased motor coordination, cirrhosis, and
decreased motor coordination. The physical exam which would reveal the most specific finding for this
disease is:
1

Hearing test
2

Slit-lamp eye exam
3

EKG
4

Colonoscopy
5

Renal ultrasound
Q/Q(M)-474029 Report a Problem

31

A child presents with pretibial hyperpigmentation, ataxia, decreased motor coordination, cirrhosis, and
decreased motor coordination. The physical exam which would reveal the most specific finding for this
disease is:
2

Slit-lamp eye exam
Wilsons disease (also known as hepatolenticular degeneration) is an autosomal recessive disorder
result in defective biliary excretion of copper, leading to copper accumulation in the liver, brain and
cornea. Clinical features include hepatomegaly, cirrhosis, ataxia, dysarthria, decreased motor
coordination, pretibial hyperpigmentation, blue lunulae, and copper deposition in the corneaKayser-
Fleisher ring, which can be diagnosed using a slit-lamp.
Q/Q(M)-474029 Report a Problem
Patients with Chondrodysplasia punctata can have findings of stippled epiphyses on X-ray examination.
Which other x-linked dominant condition can have stippled epiphyses?
1

CHILD syndrome
2

Incontinentia Pigmenti
3

Focal Dermal Hypoplasia
4

Goltz syndrome
5

Bazex syndrome
Q/Q(M)-477744 Report a Problem

Patients with Chondrodysplasia punctata can have findings of stippled epiphyses on X-ray examination.
Which other x-linked dominant condition can have stippled epiphyses?
1

CHILD syndrome
All of the syndromes listed have X-linked dominant inheritance. CHILD syndome also has findings of
stippled epiphyses. Incontinentia pigmenti is caused by defecdts in the NEMO gene. Findings include
peg/conical teeth, eye and CNS defects and alopecia. There are no bone abnormalities. Focal Dermal
Hypoplasia, otherwise known as Goltz syndrome has findings of linear atrophy following Blaschko's
lines with areas of fat herniation, mucocutaneous papillomas and pits, alopecia, nail dystrophy, tooth
abnormalities and osteopathia striata (striations of the long bones). Bazex syndrome is associated with
follicular atrophoderma, hypohidrosis, hypotrichosis and multiple basal cell carcinomas. There are no
bone abnormalities associated.
Q/Q(M)-477744 Report a Problem
The gene defect in Griscelli Syndrome is:
1

Myosin Va or Rab27a
2

LYST or CHS1
3

P gene
4

TRP1
5

None of these answers are correct
Q/Q(M)-477735 Report a Problem
The gene defect in Griscelli Syndrome is:
1

Myosin Va or Rab27a
Myosin Va or Rab27a are defective in Griscelli syndrome, an AR syndrome with mild albinism,
pancytopenia, immunodeficiency, neurologic symptoms and an accelerated phase similar to Chediak-
Higashi syndrome. LYST/CHS1 is defective in Chediak-Higashi syndrome. The P-gene is mutated in
32

oculocutaneous albinism type 2 and TRP in oculocutaneous albinism type 3.
Q/Q(M)-477735 Report a Problem
The most common cutaneous neoplasm associated with Muir Torre Syndrome is:
1

Sebaceous carcinoma
2

Sebaceous adenoma
3

Keratoacanthoma
4

Basal cell carcinoma with sebaceous differentiation
5

Squamous cell carcinomas
Q/Q(M)-474041 Report a Problem

The most common cutaneous neoplasm associated with Muir Torre Syndrome is:
2

Sebaceous adenoma
Muir-Torre syndrome is an autosomal dominant disorder due to a DNA mismatch repair gene (MSH2).
Patients present with sebaceous tumors, with adenomas being the most common (sebaceous carcinomas
next common) and associated visceral malignancies such as colon cancer (most common).
Q/Q(M)-474041 Report a Problem

Erythematous keratotic plaques of KID Syndrome most commonly occur in which location?
1

Face
2

Neck
3

Chest
4

Back
5

Abdomen
Q/Q(M)-477885 Report a Problem

Erythematous keratotic plaques of KID Syndrome most commonly occur in which location?
1

Face
The erythematous keratotic plaques of KID Syndome occur on face, extremities > trunk. KID
Syndrome is characterized by keratitis, icthyosis, and deafness. It is inherited in an autosomal dominant
fashion and caused by a mutation in the GJB2 gene which encodes connexin 26.
Q/Q(M)-477885 Report a Problem

Comma shaped corneal opacities are seen in what disease?
1

Refsum Syndrome
2

Sjogren-Larson Syndrome
3

Pseudoxanthoma elasticum
4

X-linked ichthyosis
5

Proteus syndrome
Q/Q(M)-477851 Report a Problem
33


Comma shaped corneal opacities are seen in what disease?
4

X-linked ichthyosis
X-linked ichthyosis is a X-linked recessive disorder secondary to steroid sulfatase deficiency
characterized by brown adherent scale. Additional findings include comma-shaped corneal opacities,
cryptorchisdism, and failure to progress during labor.
Q/Q(M)-477851 Report a Problem
A 12 year-old boy with pits on his palms and lateral fingers may have:
1

Arsenic exposure
2

A hereditary keratoderma
3

A corynebacteria infection
4

An inherited cancer syndrome
5

Secondary syphilis
Q/Q(M)-477232 Report a Problem

A 12 year-old boy with pits on his palms and lateral fingers may have:
4

An inherited cancer syndrome
Basal cell nevus syndrome is an autosomal dominant disease caused by mutations in the PTCH1 gene.
Clinically, patients may have numerous basal cell carcinomas, palmoplantar pits, jaw cysts, frontal
bossing, bifid ribs, calcification of falx cerebri, medulloblastoma, ovarian fibromas and fibrosarcomas.
Q/Q(M)-477232 Report a Problem

A patient with port wine stain on a lower extremity, hemihypertrophy of the limb and lymphatic and
deep venouse insufficiency of the affected limb would be considered to have Klippel-Trenaunay-Weber
syndrome. What additional feature would need to be present to define the patient as having Parkes-
Weber syndrome?
1

Arteriovenous fistulas
2

Multiple cafe-au-lait macules
3

Macroglossia
4

Cutis marmorata
5

Distichiasis
Q/Q(M)-477779 Report a Problem


A patient with port wine stain on a lower extremity, hemihypertrophy of the limb and lymphatic and
deep venouse insufficiency of the affected limb would be considered to have Klippel-Trenaunay-Weber
syndrome. What additional feature would need to be present to define the patient as having Parkes-
Weber syndrome?
1

Arteriovenous fistulas
Parkes-Weber syndrome has the additional feature of arteriovenous fistulas. The remaining features are
not part of these syndromes.
Q/Q(M)-477779 Report a Problem
34


A patient with multiple deeply pigmented papules has a skin biopsy which reveals an epitheloid blue
nevus. The next appropriate step is:
1

Reassure the patient and follow up as needed
2

Schedule prophylactic excision of the lesion
3

Begin a malignancy work-up
4

Refer to genetics
5

Order an echocardiogram
Q/Q(M)-477378 Report a Problem

A patient with multiple deeply pigmented papules has a skin biopsy which reveals an epitheloid blue
nevus. The next appropriate step is:
5

Order an echocardiogram
Epithelioid blue nevi have been reported with and without association with cardiac myxomas as a
component of the Carney complex (NAME/LAMB syndrome). Carney complex is an autosomal
dominant disorder caused by mutations in PRKAR1A. Patients have cutaneous and atrial myxomas,
blue nevi, ephelides, adrenocortical disease, and testicular tumors.
Q/Q(M)-477378 Report a Problem

Maffucci syndrome is has characteristic venous malformations of the distal extremities and benign
endochondromas which can compromise bone strength and lead to chondrosarcomas. The defect
causing this believed to be the PTH/PTHrP type I receptor which is inherited in which manner?
1

Sporadic
2

Autosomal dominant
3

Autosomal recessive
4

X-linked dominant
5

X-linked recessive
Q/Q(M)-477785 Report a Problem
Maffucci syndrome is has characteristic venous malformations of the distal extremities and benign
endochondromas which can compromise bone strength and lead to chondrosarcomas. The defect
causing this believed to be the PTH/PTHrP type I receptor which is inherited in which manner?
1

Sporadic
Maffucci syndrome is inherited in a sporadic manner.
Q/Q(M)-477785 Report a Problem

Which of the following syndromes is associated with cutis marmorata?
1

Nethertons
2

Papillon-Lefevre syndrome
3

Hemansky-Pudlak syndrome
4

Maffucci syndrome
5

Cornelia de Lange syndrome
35

Q/Q(M)-474017 Report a Problem
Which of the following syndromes is associated with cutis marmorata?
5

Cornelia de Lange syndrome
Cornelia de Lange is also known as Brachmann-de Lange syndrome. Cutaneous manifestations include
cutis marmorata, hirsutism, hypoplastic nipples and umbilicus. Patients also have small hands and feet.
They have characteristic facies which include hirsutism on the forehead, trichomegaly, synophrys,
anteverted nostrils, long philtrum, and low-set ears.
Q/Q(M)-474017 Report a Problem
Which syndrome is due to a defective secreted mammilian Ly6/uPAR-related protein-1?
1

Netherton's syndrome
2

Refsum's syndrome
3

Sjogren-Larsson syndrome
4

Mal de Meleda syndrome
5

Haim-Munk syndrome
Q/Q(M)-477499 Report a Problem

Which syndrome is due to a defective secreted mammilian Ly6/uPAR-related protein-1?
4

Mal de Meleda syndrome
Mal de Meleda, also known as keratoderma palmoplantaris transgrediens, is due to a defect in secreted
mammilian Ly6/uPAR-related protein or SLURP-1.
Q/Q(M)-477499 Report a Problem

The hair abnormality shown in the image is characteristic of which of the following diseases?
1

Naxos disease
2

Trichothiodystrophy
3

Bjornstad syndrome
4

Monilethrix
5

All of the options are correct
Q/Q(M)-477950 Report a Problem

The hair abnormality shown in the image is characteristic of which of the following diseases?
36

2

Trichothiodystrophy
The hair abnormality shown is trichoschisis, clean breaks of the hair shaft which occurs in
trichothiodystrophy. Also commonly seen is the "tiger-tail" banding pattern of the hair when placed
under polarized light. The other options do have hair shaft abnormalities, but not trichoschisis.
Q/Q(M)-477950 Report a Problem

Which of the following metals is deficient in the serum of patients with Menkes kinky hair syndrome?
1

Copper
2

Iron
3

Selenium
4

Zinc
5

Biotin
Q/Q(M)-477944 Report a Problem

Which of the following metals is deficient in the serum of patients with Menkes kinky hair syndrome?
1

Copper
Menkes kinky hair syndrome is transmitted in an X-linked recessive manner and is caused by a
mutation in ATP7A, an ATP-dependent copper tranporter. This defect results in low serum levels of
copper. These individuals will have hair abnormalities such as sparse, hypopigmented brittle hair,
eyelashes and eyebrows, lax skin, a "cupid's bow" upper lip, CNS progressive deterioration, seizures,
skeletal abnormalities and tortuous arteries. The other listed items are not associated with Menkes
syndrome.
Q/Q(M)-477944 Report a Problem

Which of the following syndromes demonstrate atrophoderma vermiculatum?
1

Tuzun Syndrome
2

ROMBO Syndrome
3

Nicolau-Balus Syndrome
4

Braun-Falco-Marghescu Syndrome
5

All of these answers are correct
Q/Q(M)-477794 Report a Problem

Which of the following syndromes demonstrate atrophoderma vermiculatum?
5

All of these answers are correct
All of the listed syndromes include atrophoderma vermiculatum as part of their constellation of
symptoms. Atrophoderma vermiculatum is characterized by honeycomb pattern of atrophic scars on the
face. Tuzun Syndrome also has scrotal tongue. ROMBO has BCCs, milia, peripheral vasodilation,
trichoepitheliomas. Nicolau-Balus has eruptive syringomas and milia. Braun-Falco-Marghescu has
keratosis pilaris and palmoplantar hyperkeratosis.
Q/Q(M)-477794 Report a Problem
A patient with Bloom Syndrome is most likely to have which laboratory abnormalities:
1

Decreased immunoglobulins
2

Macrocytic anemia
37

3

Elevated IgE
4

Thrombocytopenia
5

Positive ANA
Q/Q(M)-474039 Report a Problem

A patient with Bloom Syndrome is most likely to have which laboratory abnormalities:
1

Decreased immunoglobulins
Bloom syndrome is an autosomal recessive disorder due to a mutation in the BLM gene which codes
for a DNA helicase. Patients have impaired DNA repair after UV exposure and increased
photosensitivity. Clinical features include photodistributed erythema, cheilitis, high-pitched voice,
hypogonadism, and increased risk for leukemia, lymphoma and GI adenocarcinoma. Laboratory
evaluation reveals decreased IgA, IgM and IgG leading to increased risk of respiratory infections
Q/Q(M)-474039 Report a Problem

The syndrome characterized by generalized mild hyperkeratosis, erythematous keratotic plaques,
palmoplantar keratoderma, non-progressive sensorineural deafness, progressive bilateral keratitis with
secondary blindness is:
1

KID syndrome
2

Vohwinkel syndrome
3

Erythrokeratoderma variabilis
4

CHILD syndrome
5

Refsum syndrome
Q/Q(M)-477722 Report a Problem

The syndrome characterized by generalized mild hyperkeratosis, erythematous keratotic plaques,
palmoplantar keratoderma, non-progressive sensorineural deafness, progressive bilateral keratitis with
secondary blindness is:
1

KID syndrome
KID syndrome is described above. It is an autosomal dominant mutation in connexin 26. Vohwinkel
syndrome is also a connexin 26 mutation, but is characterized by diffuse honeycombed palmoplantar
keratoderma, pseudoainhum, starfish-shaped keratotic plaques over joints and deafness.
Erythrokeratoderma variabilis is an autosomal dominant mutation in connexin 31 and 30.3
characterized by erythematous migratory patches, fixed hyperkeratotic plaques and a palmoplantar
keratoderma. CHILD syndrome is an X-linked dominant mutation condition due to a mutation in
NAD(P)H Steroid dehydrogenas-like protein, lethal in males. Unilateral ichthyosiform erythroderma,
limb/visceral hypoplasias are characteristic. Refsum syndrome is an autosomal recessive condition with
a mutation in phytanoyl coenzyme A hydroxylase characterized by mild ichthyosis, cerebellar ataxia,
peripheral neuropathy, retinitis pigmentosa (salt & pepper) and deafness.
Q/Q(M)-477722 Report a Problem


Painful crises and 'whorled' corneal opacities are seen with which of the following enzyme
abnormalities?
1

Homogentisic acid oxidase
38

2

Alpha-galactosidase A
3

Glucocerebrosidase
4

Iduronate sulfatase
5

Glucoronidase
Q/Q(M)-478045 Report a Problem


Painful crises and 'whorled' corneal opacities are seen with which of the following enzyme
abnormalities?
2

Alpha-galactosidase A
Painful crises and whorled corneal opacities are found in Fabry disease which is caused by a defect in
alpha-galactosidase A. The remaining conditions do not have these findings.
Q/Q(M)-478045 Report a Problem

The coast of Maine cafe au lait macule is a common finding in which of the following syndromes?
1

Tuberous Sclerosis
2

Neurofibromatosis type I
3

McCune-Albright syndrome
4

Watson syndrome
5

Russell-Silver syndrome
Q/Q(M)-477741 Report a Problem

The coast of Maine cafe au lait macule is a common finding in which of the following syndromes?
3

McCune-Albright syndrome
McCune-Albright syndrome is due to a sporadic somatic mutation in Gs subunit of adenylate cyclase.
The "coast of Maine" cafe au lait macule is a characteristic finding. Other findings in clude polyostotic
fibrous dysplasia and precocious puberty. Tuberous sclerosis, Neurofibromatosis type I, Watson
syndrome (pulmonic stenosis and CALM's) and Russell-Silver syndrome (CALMs, short stature,
precocious puberty, cryptorchidism and musculoskeletal/craniofacial defects) all have CALMs as a
feature, but not the jagged type seen in McCune-Albright syndrome.
Q/Q(M)-477741 Report a Problem

Electron microscopic examination of a hair shaft reveals a canal-like groove along the shaft of a
triangular-shaped hair. This patient has:
1

Nethertons syndrome
2

Menkes Kinky Hair syndrome
3

Spun-glass hair
4

Trichothiodystrophy
5

Bjornstad syndrome
Q/Q(M)-474046 Report a Problem
39



Electron microscopic examination of a hair shaft reveals a canal-like groove along the shaft of a
triangular-shaped hair. This patient has:
3

Spun-glass hair
Pili trianguli et canaliculi is also known as Spun-glass hair or Uncombable Hair Syndrome. Netherton
patients have trichorexis invaginata, Menkes kinky hair patients have short, brittle sparse hairs, tiger
tail hair is seen in trichothiodystrophy, and pili torti is seen in bjornstad syndrome.
Q/Q(M)-474046 Report a Problem

Hereditary Hemorrhagic Telangiectasia syndrome is transmitted in an autosomal dominant fashion and
can have two variants. Type I is linked to defects in HHT1, the endoglin gene. Type II is linked to
defects in HHT2, the ALK1 gene. What feature that differentiates type I from type II clinically?
1

Type I families have an increase incidence of pulmonary arteriovenous fistulas
2

Type II families have an increased incidence of pulmonary arteriovenous fistulas
3

Type I families have an increased incidence of hepatic arteriovenous malformations
4

Type II families have a decreased incidence of hepatic arteriovenous malformations
5

None of the answers are correct
Q/Q(M)-477811 Report a Problem

Hereditary Hemorrhagic Telangiectasia syndrome is transmitted in an autosomal dominant fashion and
can have two variants. Type I is linked to defects in HHT1, the endoglin gene. Type II is linked to
defects in HHT2, the ALK1 gene. What feature that differentiates type I from type II clinically?
1

Type I families have an increase incidence of pulmonary arteriovenous fistulas
There is an increased incidence of pulmonary arteriovenous fistulas in HHT type I. Type II has an
increased incidence of hepatic arteriovenous malformations.
Q/Q(M)-477811 Report a Problem


An infant with doughy, redundant skin and short sparse hairs is likely to show which features on x-ray?
1

Metaphyseal widening in the long bones
2

Sphenoid wing dysplasia
3

Periosteal thickening
4

Osteopoikilosis
5

Stippled epiphyses
Q/Q(M)-474044 Report a Problem

An infant with doughy, redundant skin and short sparse hairs is likely to show which features on x-ray?
1

Metaphyseal widening in the long bones
The patient described has Menkes Kinky Hair syndrome, an X-linked recessive disease due to a defect
in an intestinal copper transport protein. Clinical features include pili torti, short, brittle steel-wool
hair, and spare eyelashes and sparse broken eyebrows. The skin is often hypopigmented with a soft,
doughy consistency and redundancy. Musculoskeletal manifestations include metaphyseal widening
with spurs in the long bones
40

Q/Q(M)-474044 Report a Problem

Patients with junctional epidermolysis bullosa have been found to have mutations in:
1

Laminin 5
2

Bullous pemphigoid antigen 2
3

Collagen 17
4

BP180
5

All of the answers are correct
Q/Q(M)-477712 Report a Problem

Patients with junctional epidermolysis bullosa have been found to have mutations in:
5

All of the answers are correct
All of the answers are correct. Laminin 5 is a protein integral in the adhesion of the dermis to the
epidermis. Also involved in junctional epidermolysis bullosa is bullous pemphigoid antigen 2, collagen
17 and BP180, which are synonymous for the same structure.
Q/Q(M)-477712 Report a Problem
A patient with multiple lentigines and blue nevi may also have:
1

Deafness
2

Pulmonary valve stenosis
3

Atrial myxoma
4

Mental retardation
5

GI malignancy
Q/Q(M)-475878 Report a Problem

A patient with multiple lentigines and blue nevi may also have:
3

Atrial myxoma
This patient may have a constellation of features associated with NAME syndrome, otherwise known
as Carney complex or LAMB syndrome. This condition is inherited in an autosomal dominant pattern
and is due to a defect in the PRKAR1A gene. This condition is characterized by the following features:
blue Nevi, Atrial myxomas, cutaneous Myxomas, and Ephelides. In addition, testicular tumors are seen
as well as sexual precocity. Finally, patients may have endocrine abnormalities including pigmented
nodular adrenocorticoal disease and Cushing syndrome as well as pituitary adenomas. Deafness,
pulmonary stenosis, GI malignancies, and mental retardation are not features of this condition.
Q/Q(M)-475878 Report a Problem

The main cause of death in patients with dyskeratosis congenita is which of the following?
1

Oral squamous cell carcinoma
2

Leukemia
3

Renal cell carcinoma
4

Pancytopenia
5

Atherosclerotic heart disease
41

Q/Q(M)-474288 Report a Problem

The main cause of death in patients with dyskeratosis congenita is which of the following?
4

Pancytopenia
Dyskeratosis congenita is usually inherited in an X-recessive fashion due to mutations in the dyskerin
gene, which is involved in ribosomal RNA synthesis. The less common autosomal dominant form is
caused by mutations in the telomerase gene. Clinically, there is reticulated pigmentation of skin,
poikiloderma, alopecia, nail atrophy, premalignant oral leukoplakia, and Fanconi-type pancytopenia
resulting in early death.
Q/Q(M)-474288 Report a Problem

Reticulate pigmentation of skin, poikiloderma, alopecia, nail atrophy, premalignant oral leukoplakia,
and a Fanconi-type pancytopenia resulting in early death in addition to posterior fossa malformations is
characteristic of which of the following syndromes.
1

Dyskeratosis congenita
2

Hoyeraal-Hreidarsson syndrome
3

Bloom syndrome
4

Cockayne syndrome
5

Wiskott-Aldrich syndrome
Q/Q(M)-478069 Report a Problem

Reticulate pigmentation of skin, poikiloderma, alopecia, nail atrophy, premalignant oral leukoplakia,
and a Fanconi-type pancytopenia resulting in early death in addition to posterior fossa malformations is
characteristic of which of the following syndromes.
2

Hoyeraal-Hreidarsson syndrome
Hoyeraal-Hreidarsson syndrome is has all of the features of dyskeratosis congenita plus posterior fossa
malformations. Bloom syndrome and Cockayne syndrome both have poikiloderma as features, but do
not include posterior fossa malformations as part of the syndrome. Wiskott-Aldrich syndrome does not
include any of these findings.
Q/Q(M)-478069 Report a Problem

What is the genetic defect of this autosomal dominant disorder?
1

PTEN
2

PTPN11
3

Calcium ATPase 2A2
4

Calcium ATPase 2C1
5

SPINK5
Q/Q(M)-476537 Report a Problem
42




What is the genetic defect of this autosomal dominant disorder?
4

Calcium ATPase 2C1
Hailey-Hailey is an autosomal dominant disorder that usually affects the intertriginous areas. Clinically,
there is erythema and linear fissures of the axilla and groin. On pathology, the characteristic finding is
dyskeratosis in a "dilapidated brick wall" pattern. The gene defect responsible is calcium ATPase 2C1.
Q/Q(M)-476537 Report a Problem

Ichthyosis bullosa of Siemens is a condition characterized by fragile blisters at birth, hyperkeratotic
plaques on elbows/knees later in life, and a gene mutation in:
1

Keratin 2e
2

Keratin 5/14
3

Keratin 6a/16
4

Keratin 1/10
5

None of these options are correct
Q/Q(M)-477716 Report a Problem

Ichthyosis bullosa of Siemens is a condition characterized by fragile blisters at birth, hyperkeratotic
plaques on elbows/knees later in life, and a gene mutation in:
1

Keratin 2e
Keratin 2e is mutated in ichythosis bullosa of Siemens. Keratin 5 and 14 are defective in epidermolysis
bullosa simplex, 6a/16 in Pachyonychia congenital type I (Jadassohn-Lewandowsky), 1/10 in
epidermolytic hyperkeratosis and Unna-Thost PPK.
Q/Q(M)-477716 Report a Problem

Which syndrome is characterized by hyperhidrosis, lack of pain sensation, hypersalivation, and absent
fungiform papillae?
1

Turner Syndrome
43

2

Noonan Syndrome
3

Riley-Day
4

Rubinstein-Taybi syndrome
5

Cornelia de lange Syndrome
Q/Q(M)-474014 Report a Problem


Which syndrome is characterized by hyperhidrosis, lack of pain sensation, hypersalivation, and absent
fungiform papillae?
3

Riley-Day
Riley-Day syndrome is also known as Familial Dysautonomia. It is an autosomal recessive disorder
with the gene defect on the long arm of chromosome 9. Patients have unmyelinated sensory and
sympathetic neurons and autonomic dysfunction, leading to hyperhidrosis, decreased corneal sensation
and tear flow, hypersalivation, gastroesophageal reflux, decreased deep tendon reflexes, and lack of
pain sensation. They also exhibit abnormal histamine skin test.
Q/Q(M)-474014 Report a Problem

Which of the following is not classically associated with pheochromoctyoma?
1

Multiple Endocrine Neoplasia Type IIA
2

Multiple Endocrine Neoplasia Type IIB
3

Von-Hippel-Lindau Syndrome
4

Cobb Syndrome
5

Neurofibromatosis
Q/Q(M)-476747 Report a Problem

Which of the following is not classically associated with pheochromoctyoma?
4

Cobb Syndrome
Cobb syndrome is a sporadic disease characterized by cutaneous vascular malformations associated
with malformations of the spinal cord.
Q/Q(M)-476747 Report a Problem

Patients with Hermansky-Pudlak syndrome may experience which of the following systemic
complications as a result of their disease?
1

Arteriovenous malformations
2

Pulmomary fibrosis
3

Gastroesophageal reflux disease
4

Aortic stenosis
5

Rectal abscesses
Q/Q(M)-474056 Report a Problem

44

Patients with Hermansky-Pudlak syndrome may experience which of the following systemic
complications as a result of their disease?
2

Pulmomary fibrosis
Patients with Hermansky-Pudlak syndrome are tyrosinase positive albinos. In addition to their
pigmentary dilution and increased risks for cutaneous malignancies, patients lack platelet granules
leading to impaired platelet aggregation (and therefore a bleeding diathesis). There is also a lysosomal
membrane defect which leads to the accumulation of ceroid lipofuscion in macrophages within the lung
(leading to pulmonary fibrosis), gastrointestinal tract (leading to granulomatous colitis), and heart
(cardiomyopathy).
Q/Q(M)-474056 Report a Problem

A Puerto Rican woman is seen in clinic for a pruritic rash on her trunk. A punch biopsy is performed.
The biopsy site continues to bleed, with hematoma formation. The bleeding is eventually controlled.
On further exam, her skin and hair are light brown. She has a history of granulomatous colitis. What it
the most likely reason she had excess bleeding with a simple procedure?
1

Her platelets lack dense bodies, causing excess bleeding
2

Her intrinsic factor is deficient
3

Her Factor VIII levels are low
4

She is congentially deficient in platelets
5

None of the answers are correct
Q/Q(M)-477736 Report a Problem

A Puerto Rican woman is seen in clinic for a pruritic rash on her trunk. A punch biopsy is performed.
The biopsy site continues to bleed, with hematoma formation. The bleeding is eventually controlled.
On further exam, her skin and hair are light brown. She has a history of granulomatous colitis. What it
the most likely reason she had excess bleeding with a simple procedure?
1

Her platelets lack dense bodies, causing excess bleeding
Platelets without dense bodies cause excess bleeding in Hermansky-Pudlak syndrome. Other features of
this condition include oculocutaneous albinism, ceroid lysosomal storage disease resulting in
pulmonary fibrosis, granulomatous colitis.
Q/Q(M)-477736 Report a Problem

Acropigmentation of Dohi is characterized by:
1

Reticulated pigmentation of the axillae, neck, and groin
2

Linear palmar pits and pigmented macules on volar and dorsal hands and feet
3

Pigmented and depigmented macules on the distal dorsal extremities and face
4

Hyperpigmented macules on the lips and oral mucosa
5

Flaccid, superficial pustules that burst and leave pigmented macules
Q/Q(M)-482357 Report a Problem

Acropigmentation of Dohi is characterized by:
3

Pigmented and depigmented macules on the distal dorsal extremities and face
Patients with Acropigmentation of Dohi (dyschromatosis symmetrica hereditaria) are usually from
Europe, India, or the Carribean. They develop pigmented and depigmented macules on dorsal distal
45

extremities and face. This disorder is due to mutation in DSRAD gene. Reticulated pigmentation of the
axillae, neck, and groin is seen in Dowling-Degos' disease. Linea palmar pits and pigmented macules
on volar and dorsal hands an feet is seen in Reticulate acropigmentation of Kitamura. Hyperpigmented
macules on oral mucosa and lips can be seen in Peutz-Jeghers, Cronkite-Canada, and Laugier-Hunziker
syndromes. Flaccid, superficial pustules that burst and leave pigmented macules is seen in transient
neonatal pustulosis.
Q/Q(M)-482357 Report a Problem

What is the inheritance pattern of chronic granulomatous disease?
1

Autosomal recessive
2

Autosomal dominant
3

X-linked recessive
4

X-linked dominant
5

Sporadic
Q/Q(M)-478039 Report a Problem

What is the inheritance pattern of chronic granulomatous disease?
3

X-linked recessive
Chronic granulomatous disease is inherited in an x-linked recessive manner. There are mutations
present in CYBA (a cytochrome subunit), CYBB, and NCF1 & 2 (neutrophil cytosol factors 1 & 2).
Q/Q(M)-478039 Report a Problem

What finding is seen on brain imaging of patients with Papillon-Lefevre Syndrome?
1

Tram track calcifications
2

Calcification of the falx cerebri
3

Calcification of the hippocampus
4

Calcification of the dura
5

Agenesis of the corpus callosum
Q/Q(M)-478204 Report a Problem

What finding is seen on brain imaging of patients with Papillon-Lefevre Syndrome?
4

Calcification of the dura
Pappilon Lefevre Syndrome is an autosomal recessive syndrome characterized by transgredient PPK
and periodontitis. There is a defect in cathepsin C. One sees dural calcification at the tentorium and
choroid attachments. Tram track calcifications are seen in STurge-WEber. CAlcification of the falx
cerebri and agenesis of the corpus callosum is seen in basal cell nevus syndrome. Hippocampal
calcification is seen in lipoid proteinosis.
Q/Q(M)-478204 Report a Problem

Premature aging seen in Cockaynes syndrome is caused by a mutation in which gene?
1

Transglutaminase
2

ATM gene
46

3

Patched gene
4

NEMO gene
5

DNA helicase gene
Q/Q(M)-477323 Report a Problem

Premature aging seen in Cockaynes syndrome is caused by a mutation in which gene?
5

DNA helicase gene
Cockaynes syndrome is caused by a mutation in a DNA helicase gene, CSA or ERCC8. The
condition is defined by growth deficiency, premature aging, and pigmentary retinal degeneration. 75%
of patient have photosensitive eruptions and severe cataracts before the age of 3 are associated with
poor prognosis.
Q/Q(M)-477323 Report a Problem

An infant presents with poikiloderma on his face, buttocks, arms and legs. He is also noted to have a
hypoplastic thumb and no radius. Yearly ophthalmologic examination is indicated because of the infant
is at risk for developing:
1

Glaucoma
2

Cataracts
3

Subcapsular lens displacement
4

Copper deposition
5

Macular degeneration
Q/Q(M)-474038 Report a Problem

An infant presents with poikiloderma on his face, buttocks, arms and legs. He is also noted to have a
hypoplastic thumb and no radius. Yearly ophthalmologic examination is indicated because of the infant
is at risk for developing:
2

Cataracts
The patient described has Rothmund-Thomson syndrome (or poikiloderma congentiale), an autosomal
recessive disease localized to chromosome 8 and believed to be due to a DNA helicase mutation
(RECQL4). 40-50% of patients will develop juvenile cataracts before puberty. Other clinical features
include alopecia, dystrophic nails, short stature, hypogonadism and dental dysplasia.
Q/Q(M)-474038 Report a Problem
Which of the following mutations has been linked to an NF-1 like presentation?
1

SLURP-1
2

Plakophilin-1
3

SPRED-1
4

ALK-1
5

TSC-1
Q/Q(M)-482516 Report a Problem

Which of the following mutations has been linked to an NF-1 like presentation?
47

3

SPRED-1
SPRED-1 acts as a suppressor of the Ras/MAPK pathway, the same pathway involved in
neurofibromatosis-1. Likewise, defects in SPRED1 are the cause of neurofibromatosis type 1-like
syndrome, also known as Legius Syndrome. Legius Syndrome has the following characteristics: cafe au
lait spots, axillary and inguinal freckling, learning disability, and macrocephaly; likewise patients can
actually meet criteria for the clinical diagnosis of NF-1. Although Legius Syndrome has many
overlapping features with NF1, neurofibromas, lisch nodules, and CNS tumors have not been reported.
(Brems H, et al. Nat Gen 2007.)
Q/Q(M)-482516 Report a Problem

A patient Buschke-Ollendorff syndrome has osteopoikilosis and which cutaneous finding?
1

Waxy papules along the eyelids
2

Caf au lait macules
3

Port wine stain
4

Juvenile elastoma
5

Epidermal nevi
Q/Q(M)-477128 Report a Problem

A patient Buschke-Ollendorff syndrome has osteopoikilosis and which cutaneous finding?
4

Juvenile elastoma
Buschke-Ollendorf syndrome is an autosomal dominant syndrome associated with increased elastic
fiber in the skin. Key features include dermatofibrosis lenticularis disseminata (also called juvenile
elastomas) and osteopoikilosis.
Q/Q(M)-477128 Report a Problem

Which of the following are cutaneous features of Marfan syndrome?
1

Loose skin and crumpled ears
2

Dermatofibrosis lenticularis and striae
3

Fat herniation and cutaneous atrophy
4

Sclerodermoid changes and dyspigmentation
5

Striae and elastosis perforans serpiginosa
Q/Q(M)-474278 Report a Problem

Which of the following are cutaneous features of Marfan syndrome?
5

Striae and elastosis perforans serpiginosa
Marfan syndrome is an autosomal dominant disorder caused by mutations in fibrillin 1 and 2. Patients
have tall stature, arachnodactyly, pectus excavatum, high-arched palate, joint laxity, ectopia lentis with
upward dislocation, dilated aorta with rupture, mitral valve prolapse, striae, and elastosis perforans
serpiginosa.
Q/Q(M)-474278 Report a Problem

Ichthyosis linearis circumflexa is one of the findings seen in the syndrome caused by which of the
following genes?
48

1

SPINK5
2

SLURP1
3

GJB2
4

LMX1B
5

CYLD
Q/Q(M)-477720 Report a Problem

Ichthyosis linearis circumflexa is one of the findings seen in the syndrome caused by which of the
following genes?
1

SPINK5
The SPINK5 gene encodes for LEKTI, a serine protease inhibitor important in the regulation of
proteolysis in epithelia formation and keratinocyte terminal differentiation, is mutated in Nethertons
Syndrome. Other findings include: trichorrhexis invaginata (bamboo, ball and socket hair), atopic
dermatitis, and anaphylaxis from food allergy. SLURP1 is mutated in Mal de Maleda, GJB2 in
Vohwinkels syndrome, LMX1B in Nail-Patella syndrome, and CYLD in Familial Cylindromiasis.
Q/Q(M)-477720 Report a Problem


Patients with hemochromatosis are at increased risk for which of the following?
1

Vibrio vulnificus infections
2

Yersenia infections
3

Polyarthritis
4

Generalized metallic-grey hyperpigmentation
5

All of these options are correct
Q/Q(M)-478062 Report a Problem


Patients with hemochromatosis are at increased risk for which of the following?
5

All of these options are correct
Patients with hemochromatosis have increased intestinal iron absorption leading to systemic iron
overload. Signs inclued a generalized metallic-grey hyperpigmentation, koilonychia, alopecia
(especially pubic/axillary hair) cardiac failure/arrythmias/heart block, hepatomegaly with crrhosis,
diabetes (bronze diabetes), polyarthritis with chondrocalcinosis and are susceptible to Vibrio vulnificus
and Yersinia infections.
Q/Q(M)-478062 Report a Problem

The best screening test for hemochromatosis is:
1

Ferritin
2

Hematocrit
3

Complete Blood Count
4

Transferrin
49

5

Copper levels
Q/Q(M)-478061 Report a Problem


The best screening test for hemochromatosis is:
1

Ferritin
The ferritin is the best screening test for hemochromatosis, an autosomal disease of increased intestinal
iron absorption. The other tests are not the best method for screening.
Q/Q(M)-478061 Report a Problem

A patient presents with starfish keratoses, pseudoainhum, honeycombed PPK, and generalized
ichthyosis. What is the most likely genetic defect?
1

Connexin 31
2

Connexin 26
3

Connexin 30
4

Loricrin
5

Connexin 33
Q/Q(M)-478196 Report a Problem

A patient presents with starfish keratoses, pseudoainhum, honeycombed PPK, and generalized
ichthyosis. What is the most likely genetic defect?
4

Loricrin
The patient has Vohwinkel syndrome. This is an autosomal dominant syndrome with 2 clinical variants.
The variant described above with generalized ichthyosis is due to a loricrin mutation. In the classic
form with nonprogressive hearing loss connexin 26 is mutated.
Q/Q(M)-478196 Report a Problem

Mutations affecting the VEGF receptor-3 cause which of the following disorders?
1

Hereditary lymphedema (Nonne-Milroy disease)
2

Lymphedema-distichiasis syndrome
3

Lymphedema and ptosis
4

Noonan syndrome
5

Hereditary hemorrhagic telangiectasias
Q/Q(M)-474270 Report a Problem

Mutations affecting the VEGF receptor-3 cause which of the following disorders?
1

Hereditary lymphedema (Nonne-Milroy disease)
Hereditary lymphedema (Nonne-Milroy disease) is an autosomal dominant condition caused by
mutations in the FLT4 gene which encodes for VEGF receptor-3. There is congenital lymphedema and
chylous ascites, scrotal swelling, intestinal tract protein loss, persistent bilateral pleural effusion, and
hypoproteinemia.
Q/Q(M)-474270 Report a Problem
50

Adenosine deaminase deficiency is associated with which of the following disorders?
1

Wiskott Aldrich syndrome
2

Gout
3

Job syndrome
4

Xeroderma pigmentosum
5

Severe combined immunodeficiency
Q/Q(M)-478041 Report a Problem

Adenosine deaminase deficiency is associated with which of the following disorders?
5

Severe combined immunodeficiency
Adenosine deaminase deficiency is associated with severe combined immunodeficiency. The most
common inheritance is x-linked recessive. It is a mixed group of disorders all sharing defects in cell-
mediated and humoral immunity. Skin findings include: candidal infections, mucocutaneous, bacterial
pyodermas, seborrheic-like dermatitis/lichen planus-like sclerodermatous changes, aplastic thymus and
pneumonias. The other listed syndromes are not associated with adenosine deaminase deficiency.
Q/Q(M)-478041 Report a Problem

Which of the following signs is not a criteria for the diagnosis of Neurofibromatosis type I?
1

Bilateral vestibular schwannomas
2

Axillary freckling
3

Optic gliomas
4

Greater than 5 caf-au-lait macules
5

Sphenoid dysplasia
Q/Q(M)-477733 Report a Problem
Which of the following signs is not a criteria for the diagnosis of Neurofibromatosis type I?
1

Bilateral vestibular schwannomas
Bilateral vestibular schwannomas are related to type II neurofibromatosis. The remaining options are
diagnostic criteria for NF-1.
Q/Q(M)-477733 Report a Problem

A patient with melanoma and a malignant glioma is diagnosed with Li-Fraumeni syndrome. Which of
the following tumors occurs most frequently in this disease?
1

Rhabdomyosarcoma
2

Adrenocortical carcinoma
3

Lung carcinoma
4

Breast carcinoma
5

Leukemia
Q/Q(M)-477233 Report a Problem

51

A patient with melanoma and a malignant glioma is diagnosed with Li-Fraumeni syndrome. Which of
the following tumors occurs most frequently in this disease?
1

Rhabdomyosarcoma
Li-Fraumeni syndrome is a familial tumor syndrome caused by mutations in the tumor suppressor gene
p53. They are at risk for a wide range of malignancies with particularly high occurrences of soft tissue
sarcomas, breast caner, brain tumors, acute leukemia, and adrenal cortical carcinoma. Soft tissue
sarcomas are among the most common reported with this disease.
Q/Q(M)-477233 Report a Problem


A 13 year old girl has woolly hair, keratoderma of the soles, edema of the lower extremities, and
swelling of the abdomen. A cardiac evaluation reveals right-sided cardiomyopathy. What is the most
likely defective protein?
1

Plakoglobin
2

Plectin
3

Desmocollin
4

Desmoglein
5

Desmoplakin
Q/Q(M)-482707 Report a Problem

A 13 year old girl has woolly hair, keratoderma of the soles, edema of the lower extremities, and
swelling of the abdomen. A cardiac evaluation reveals right-sided cardiomyopathy. What is the most
likely defective protein?
1

Plakoglobin
Naxos disease is caused by a defect in the plakoglobin protein. It is characterized by woolly hair,
keratoderma, and right-sided cardiomyopathy.
Q/Q(M)-482707 Report a Problem
Christ-Siemens-Touraine Syndrome is most commonly linked with defects in which of the following
genes?
1

NEMO
2

Ectodysplasin (EDA)
3

ERCC2
4

ATP7A
5

None of these options are correct
Q/Q(M)-477952 Report a Problem

Christ-Siemens-Touraine Syndrome is most commonly linked with defects in which of the following
genes?
2

Ectodysplasin (EDA)
Ectodysplasin (EDA) on Xq12-q13 is transmitted in an X-linked recessive fashion and is the most
common cause of anhidrotic ectodermal dysplasia (Christ-Siemens-Touraine syndrome or hypohidrotic
ectodermal dysplasia). NEMO can be linked to this syndrome and is associated with
immunodeficiency, but this is a rare association. ERCC2 is associated with trichothiodystrophy and
ATP7A with Menkes kinky hair syndrome. They are not associated with anhidrotic ectodermal
52

dysplasia.
Q/Q(M)-477952 Report a Problem

A 3 year-old boy presents with the findings seen in the image. He also has thrombocytopenia with
purpura and a history of recurrent pyogenic bacterial infections. What is the most likely diagnosis in
this child?
1

Chronic Granulomatous disease
2

Wiskott-Aldrich syndrome
3

Hyper-IgE syndrome
4

Severe combined immunodeficiency
5

Leiner syndrome
Q/Q(M)-478024 Report a Problem


A 3 year-old boy presents with the findings seen in the image. He also has thrombocytopenia with
purpura and a history of recurrent pyogenic bacterial infections. What is the most likely diagnosis in
this child?
2

Wiskott-Aldrich syndrome
The findings described are consistent with Wiskott-Aldrich syndrome. The characteristic triad can be
simplified to the 3 P's - Pruritus (atopic dermatitis), Purpura (thrombocytopenia leading to purpura and
other bleeding) and pyogenic infections. The remaining options are related to Wiskott-Aldrich in that
they all have immunodeficiency as a feature, but not the same spectrum of disease.
Q/Q(M)-478024 Report a Problem

Beare-Stevenson cutis gyrata syndrome is linked with mutations in:
1

Fibroblast growth factor receptor 2
2

BSCL2
3

LMNA
4

ATP7A
5

None of these answers are correct
Q/Q(M)-477902 Report a Problem

Beare-Stevenson cutis gyrata syndrome is linked with mutations in:
1

Fibroblast growth factor receptor 2
Beare-Stevenson ciutis gyrata syndrome has been linked to mutations in fibroblast growth factor
53

receptor 2. This syndrome is characterized by: craniosynostosis, ciutis gyrata, acanthosis nigricans,
anogenital anomalies, skin tags, prominent umbilical stump, furrowed palms and soles. Apert syndrome
is also linked to this mutation. BSCL2 is linked to Berardinelli-Seip congenital lipodystrophy, LMNA
to Familial partial lipodystrophy and ATP7A to Menkes kinky hair syndrome.
Q/Q(M)-477902 Report a Problem

Which type of epidermolysis bullosa is associated with mitten deformities of the hands?
1

Dominant dystrophic
2

Recessive dystrophic
3

Weber-Cockayne
4

Herlitz type
5

Generalized atrophic benign epidermolysis bullosa (GABEB)
Q/Q(M)-474241 Report a Problem


Which type of epidermolysis bullosa is associated with mitten deformities of the hands?
2

Recessive dystrophic
The recessive dystrophic type of epidermolysis bullosa is associated with chronic hand ulcers resulting
in scarring that causes mitten deformities. These chronic scars often result in the formation of fatal
squamous cell carcinomas.
Q/Q(M)-474241 Report a Problem

A 16 year-old girl presents with a family history of Gardner syndrome. Her mother is very concerned
that her daughter may have the syndrome as it runs in her family and she has many skin complaints.
Gardner syndrome has been linked to defects in beta-catenin mediated transcription. Which of the
following ocular finding is diagnostic for Gardner syndrome?
1

Lisch nodules
2

Lester iris
3

Congenital Hypertrophy of the Retinal Pigment Epithelium
4

Angioid streaks
5

Retinal detachment
Q/Q(M)-481630 Report a Problem


A 16 year-old girl presents with a family history of Gardner syndrome. Her mother is very concerned
that her daughter may have the syndrome as it runs in her family and she has many skin complaints.
Gardner syndrome has been linked to defects in beta-catenin mediated transcription. Which of the
following ocular finding is diagnostic for Gardner syndrome?
3

Congenital Hypertrophy of the Retinal Pigment Epithelium
CHRPE (Congenital Hypertrophy of the Retinal Pigment Epithelium) is the characteristic eye finding
for Gardner syndrome. Lisch nodules are seen in Neurofibromatosis I, Lester iris in Nail-Patella
syndrome and angioid streaks are present in Pseudoxanthoma elasticum. Retinal detachment is not part
of Gardner syndrome.
Q/Q(M)-481630 Report a Problem
54


Which of the following syndromes is associated with tricholemmomas?
1

Birt-Hogg-Dube
2

Brooke-Spiegler
3

Bannayan-Riley-Ruvacalba
4

Basex
5

Rasmussen's
Q/Q(M)-477793 Report a Problem


Which of the following syndromes is associated with tricholemmomas?
3

Bannayan-Riley-Ruvacalba
Tricholemmomas are seen in Bannayan-Riley-Ruvacalba syndrome. This is an autosomal dominant
condition with macrocephaly, lipomas, hemangiomas, skeletal abnormalities, lymphangioma
circupscriptum, angiokeratomas, penile lentigines, acanthosis nigricans, and achrocrodons. There is an
increased incidence of breast, thyroid, and GI cancers. Tricholemmomas are also seen in Cowden
syndrome.
Q/Q(M)-477793 Report a Problem

Odontogenic cysts and palmoplantar pits are seen in:
1

Gardners syndrome
2

Gorlin Syndrome
3

Blooms Syndrome
4

Goltz Syndrome
5

Refsum syndrome
Q/Q(M)-474040 Report a Problem


Odontogenic cysts and palmoplantar pits are seen in:
2

Gorlin Syndrome
Gorlin syndrome (Basal Cell Nevus Syndrome)is an autosomal dominant disorder due to a defect in the
PTCH gene whose function normally inhibits SMOOTHENED signaling. Odontogenic cysts
and palmoplantar pits are characteristic features, in addition to multiple basal cell carcinomas. Other
features include: frontal bossing, kyphoscoliosis, calcification of falx cerebri, hypertelorism, ovarian
fibromas and rarely mental retardation.
Q/Q(M)-474040 Report a Problem

A child presents with macroglossia, exopthalmos and gigantism. He has a history of omphalocele repair
and has circular depressions on the rim of the posterior helices. Although this syndrome is most often
transmitted in a sporadic manner, 15% of cases have defects in which gene?
1

KIP2
2

ATM
55

3

VHL
4

FLT4
5

FOXC2
Q/Q(M)-477780 Report a Problem

A child presents with macroglossia, exopthalmos and gigantism. He has a history of omphalocele repair
and has circular depressions on the rim of the posterior helices. Although this syndrome is most often
transmitted in a sporadic manner, 15% of cases have defects in which gene?
1

KIP2
KIP2 can be mutated in 15% of cases of Beckwith-Wiedemann Syndrome. KIP2 is a cyclin-dependent
kinase inhibitor gene which acts as a negative regulator of cell proliferation. These children have an
increase risk of Wilms' tumor and organomegaly. In addition to the circular ear depression, they can
also have a linear earlobe crease. The remaining genes are mutated in other syndromes with vascular
disorders as a component: ATM in ataxia telangectasia, VHL in Von Hippel-Lindau, FLT4 in
Hereditary lymphedema and FOXC2 in Lymphedema-distichiasis syndrome.
Q/Q(M)-477780 Report a Problem

Which gene is defective in Wiskott-Aldrich syndrome?
1

WAS
2

CYBA
3

CYBB
4

NCF1
5

NCF2
Q/Q(M)-478025 Report a Problem

Which gene is defective in Wiskott-Aldrich syndrome?
1

WAS
The WAS gene is defective in Wiskott-Aldrich syndrome. WAS is an Arp2/3 complex interacting
protein. The remaining options are genes related to Chronic Granulomatous Disease and are not active
in the pathogenesis of Wiskott-Aldrich syndrome.
Q/Q(M)-478025 Report a Problem


What is the most likely nail findings in a patient who has this autosomal dominant disease with these
keratotic papules and cobblestoning of the oral mucosa?
1

Koilonychia
2

Red and white longitudinal bands
3

Melanonychia
4

Half and half nails
5

Pincer nails
Q/Q(M)-476639 Report a Problem
56



What is the most likely nail findings in a patient who has this autosomal dominant disease with these
keratotic papules and cobblestoning of the oral mucosa?
2

Red and white longitudinal bands
Darier's disease is an autosomal dominant disorder characterized by greasy hyperkeratotic papules. The
papules often coalesce into a warty plaque and have a tendency for secondary viral or bacterial
infection. The mutation is in calcium ATPase 2A2. The classic nail finding is red and white
longitudinal bands with V-shaped nicking.
Q/Q(M)-476639 Report a Problem

Hyper IgE syndromes are associated with genetic deficiency of which of the following?
1

WASp
2

IL-17 receptor
3

AIRE
4

FOXP3
5

Dock8
Q/Q(M)-482536 Report a Problem

Hyper IgE syndromes are associated with genetic deficiency of which of the following?
5

Dock8
To date, three genetic etiologies of hyper IgE syndromes have been identified: STAT3, DOCK8, and
Tyk2. All of these hyper IgE syndromes are characterized by eczema, sinopulmonary infections, and
greatly elevated serum IgE. However, each has distinct clinical manifestations. Mutations in STAT3
cause autosomal dominant HIES (Job\'s syndrome), which is unique in its diversity of connective
tissue, skeletal, and vascular abnormalities. DOCK8 deficiency is characterized by severe cutaneous
viral infections such as warts, and a predisposition to malignancies at a young age. Hyper IgE
syndrome associated with Tyk2 deficiency is characterized by nontuberculous mycobacterial infections.
Q/Q(M)-482536 Report a Problem

Which eye findings would be expected in an individual with this disorder associated with
atherosclerosis?
1

Dendritic corneal ulcerations
2

Salt and pepper retinitis pigmentosa
3

Ectopia lentis
57

4

Angoid streaks
5

Keratoconus
Q/Q(M)-476539 Report a Problem



Which eye findings would be expected in an individual with this disorder associated with
atherosclerosis?
4

Angoid streaks
Pseudoxanthoma elasticum is an autosomal rescessive or autosomal domminant disease caused by a
mutation in ABCC6 (adenosine triphosphate-binding cassette subfamily C member 6). Associated
findings include gastric artery hemorrhage, angiod streaks, retinal hemorrhage, atherosclerotic disease,
and a possible increased risk of first trimester miscarriage.
Q/Q(M)-476539 Report a Problem

The gene defect in LEOPARD syndrome is:
1

PTPN11
2

PRKAR1A
3

KIP2
4

ATM
5

Neurofibromin
Q/Q(M)-477746 Report a Problem

The gene defect in LEOPARD syndrome is:
1

PTPN11
PTPN11 gene is mutated in LEOPARD syndrome. This syndrome consists of the complex of
Lentigines, EKG abnormalities, Ocular hypertelorism, Pulmonary stenosis, Abnormal genitalia,
Retardation of growth and Deafness. PRKAR1A is associated with the Carney complex of diseases.
KIP2 is found in 15% of cases of Beckwith-Wiedermann syndrome. ATM is mutated in ataxia
telangectasia syndrome and Neurofibromin in Neurofibromatosis type I.
Q/Q(M)-477746 Report a Problem

An infant presents with multiple congenital hemangomas in an generalized distribution. What is the
most serious associated condition?
58

1

Congestive Heart Failure
2

Obstructive jaundice
3

Portal hypertension
4

All of the answers are correct
5

None of the answers are correct
Q/Q(M)-477817 Report a Problem

An infant presents with multiple congenital hemangomas in an generalized distribution. What is the
most serious associated condition?
1

Congestive Heart Failure
High output congestive heart failure can lead to death in these children. Obstructive jaundice and portal
hypertension both occur, but are less likely to cause death. The hemangiomas will undergo spontaneous
regression.
Q/Q(M)-477817 Report a Problem


Refsum syndrome is due to a deficiency in phytanyl coenzyme A hydroxylase. Treatment for this
condition is:
1

Diet low in green vegetables, dairy and ruminant fats
2

Diet high in green vegetables, dairy and ruminant fats
3

Enzyme replacement
4

No treatment is available at this time
5

Avoid phenylalanine
Q/Q(M)-477723 Report a Problem

Refsum syndrome is due to a deficiency in phytanyl coenzyme A hydroxylase. Treatment for this
condition is:
1

Diet low in green vegetables, dairy and ruminant fats
Treatment is with a diet low in green vegetables, dairy and ruminant fats is the treatment of choice for
Refsum syndrome. Avoidance of specific amino acids is not helpful.
Q/Q(M)-477723 Report a Problem

Which type of porphyria is associated with hyponatremia?
1

Acute intermittent porphyria
2

Porphyria cutanea tarda
3

Variegate porphyria
4

Hereditary coproporphyria
5

Erythropoietic protoporphyria
Q/Q(M)-482617 Report a Problem
59


Which type of porphyria is associated with hyponatremia?
1

Acute intermittent porphyria
Acute intermittent porphyria can cause hyponatremia due to the syndrome of inappropriate antidiuretic
hormone secretion.
Q/Q(M)-482617 Report a Problem


A 16 year-old girl presents with a family history of Gardner syndrome. Her mother is very concerned
that her daughter may have the syndrome as it runs in her family and she has many skin complaints.
Gardner syndrome has been linked to defects in beta-catenin mediated transcription. Which of the
following genes dysfunction is responsible?
1

APC
2

STK11
3

CYLD
4

PTCH
5

ABCC6
Q/Q(M)-481629 Report a Problem

A 16 year-old girl presents with a family history of Gardner syndrome. Her mother is very concerned
that her daughter may have the syndrome as it runs in her family and she has many skin complaints.
Gardner syndrome has been linked to defects in beta-catenin mediated transcription. Which of the
following genes dysfunction is responsible?
1

APC
APC gene has been associated with defects in beta-catenin mediated transcription. The remaining genes
are not associated with Gardner syndrome.
Q/Q(M)-481629 Report a Problem

Medulloblastomas are seen in which of the following syndromes?
1

Bazex syndrome
2

Familial cylindromatosis
3

Nicolau-Balus syndrome
4

Birt-Hogg-Dube syndrome
5

Nevoid basal cell carcinomas syndrome
Q/Q(M)-474286 Report a Problem

Medulloblastomas are seen in which of the following syndromes?
5

Nevoid basal cell carcinomas syndrome
Nevoid basal cell carcinomas syndrome (Gorlin syndrome) is an autosomal dominant condition caused
by mutations in the patched gene, which is involved in the hedgehog signaling pathway. Patients
develop innumerable BCCs, palmoplantar pits, painful odontogenic jaw keratocysts, frontal bossing,
bifid ribs, and calcification of the falx cerebri. Medulloblastomas and ovarian fibromas and
fibrosarcomas are associated with this condition.
Q/Q(M)-474286 Report a Problem
60


Which of the following medications is a teratogen associated with a aplasia cutis congenita?
1

Propranolol
2

Methimazole
3

Lithium
4

Alcohol
5

Warfarin
Q/Q(M)-474284 Report a Problem

Which of the following medications is a teratogen associated with a aplasia cutis congenita?
2

Methimazole
Aplasia cutis congenita is characterized by well-demarcated erosions at birth that heal with atrophic,
alopecic scars. Some cases are caused by medications, with methimazole considered a teratogen
particularly associated with this condition.
Q/Q(M)-474284 Report a Problem

Which of the following conditions is inherited in an X-linked dominant (XD) manner?
1

Chodrodysplasia punctata
2

CHILD Syndrome
3

Focal dermal hypoplasia
4

All of the answers are correct
5

None of the answers are correct
Q/Q(M)-477708 Report a Problem

Which of the following conditions is inherited in an X-linked dominant (XD) manner?
4

All of the answers are correct
All of the syndromes listed are XD. Other XD syndromes are: Incontinentia pigmenti and Bazex
syndrome.
Q/Q(M)-477708 Report a Problem

Which of the following diseases is seen only in females?
1

Griscelli syndrome
2

Piebaldism
3

Hypomelanosis of Ito
4

Incontinentia pigmenti
5

Carney complex
Q/Q(M)-474254 Report a Problem


61

Which of the following diseases is seen only in females?
4

Incontinentia pigmenti
Incontinentia pigmenti (Bloch-Sulzberger syndrome) is an X-linked dominant disease that is lethal in
males. There are four stages: vesicular, verrucous, hyperpigmented and hypopigmented. Also seen are
peg and conical teeth, eye abnormalities, CNS defects, and alopecia. This condition is caused by
mutations in the NEMO gene.
Q/Q(M)-474254 Report a Problem

Patients with Dariers disease are at increased risk for:
1

Kaposis varicelliform eruption
2

Melanoma
3

Decreased life span
4

Basal cell carcinoma
5

Lipid abnormalities
Q/Q(M)-477166 Report a Problem

Patients with Dariers disease are at increased risk for:
1

Kaposis varicelliform eruption
Kaposis varicelliform eruption is the condition in which viral infection occurs in a patient with a pre-
existing chronic dermatitis. Dariers disease is an autosomal dominant genodermatosis caused by a
mutation in ATP2A2 which encodes SERCA2. Cutaneous manifestations of warty, hyperkeratotic
papules in a seborrheic dermatitis, which may be infected with HSV or bacteria.
Q/Q(M)-477166 Report a Problem
Junctional epidermolysis bullosa with pyloric atresia is associated with mutations in:
1

The alpha-6 subunit of integrin
2

The beta-4 subunit of integrin
3

Both subunits of integrin can have mutations causing this type of junctional epidermolysis
bullosa
4

Plectin
5

Laminin 5
Q/Q(M)-477713 Report a Problem

Junctional epidermolysis bullosa with pyloric atresia is associated with mutations in:
3

Both subunits of integrin can have mutations causing this type of junctional epidermolysis
bullosa
Both subunits of integrin can have mutations causing this type of junctional epidermolysis bullosa.
Plectin is associated with epidermolysis bullosa simplex with muscular dystrophy. Laminin 5 is
mutated in Herlitz and non-Herlitz types of junctional epidermolysis bullosa.
Q/Q(M)-477713 Report a Problem

A sporadic syndrome affecting transcriptional coactivator CREB-binding protein is:
1

Rubinstein-Taybi syndrome
62

2

Cornelia de Lange syndrome
3

Nonne-Milroy disease
4

Maffucci syndrome
5

Blue rubber bleb nevus syndrome
Q/Q(M)-477784 Report a Problem
A sporadic syndrome affecting transcriptional coactivator CREB-binding protein is:
1

Rubinstein-Taybi syndrome
Rubinstein-Taybi syndrome is caused by a sporadically transmitted defect in transcriptional coactivator
CREB-binding protein. This gene is responsible for encoding a nuclear protein which acts as a co-
activator of cAMP regulated gene expression. Findings of this syndrome include: capillary
malformation, short stature, broad thumbs, craniofacial abnormalities including beaked nose, mental
retardation, strabismus, congenital heart defects and cryptorchidism. The other listed conditions are not
related to this defect.
Q/Q(M)-477784 Report a Problem
Premalignant leukoplakia of the oral mucosa is associated with:
1

Bloom syndrome
2

Werner Syndrome
3

Xeroderma Pigmentosum
4

Dyskeratosis Congenita
5

Rothmund-Thomson syndrome
Q/Q(M)-474043 Report a Problem

Premalignant leukoplakia of the oral mucosa is associated with:
4

Dyskeratosis Congenita
Dyskeratosis Congenita (also known as Zinsser-Engman-Cole syndrome) is thought to have two modes
of inheritance. The more common X-linked disorder is due to a mutation in the Dyskerin gene, while
the autosomal dominant form is due to a mutation in TERC, a telomerase RNA component. Clinical
features include reticulated gray-brown hyperpigmentation, paloplantar hyperkeratosis, alopecia,
onychodystrophy, premalignant leukoplakia of any mucosal surface, and mental retardation.
Q/Q(M)-474043 Report a Problem


Cutaneous osteomas are seen in which syndrome?
1

Waardenburg syndrome
2

LEOPARD syndrome
3

Carney complex
4

Albright hereditary osteodystrophy
5

Gauchers syndrome
Q/Q(M)-474257 Report a Problem

Cutaneous osteomas are seen in which syndrome?
63

4

Albright hereditary osteodystrophy
Albright hereditary osteodystrophy is caused by mutations in the Gs subunit of adenylate cyclase.
There is calcification and ossification due to pseudohypoparathyroidism, absent 4th knuckle, and
hypogonadism.
Q/Q(M)-474257 Report a Problem

A 18 yo man presents for evaluation of foot lesions. There are thick hyperkeratotic plaques
symmetrically on only the weight bearing plantar surfaces. What test(s) should this patient be referred
for?
1

Head CT
2

Thoracic CT
3

Hepatic ultrasound
4

Endoscopy
5

Knee films
Q/Q(M)-477887 Report a Problem

A 18 yo man presents for evaluation of foot lesions. There are thick hyperkeratotic plaques
symmetrically on only the weight bearing plantar surfaces. What test(s) should this patient be referred
for?
4

Endoscopy
The patient likley has Howel-Evans Syndrome. These patients present with symmetric focal weight
bearing PPK in the second decade to adulthood. After the third decade, esophageal carcinoma can
occur. These patients should have periodic endoscopic evaluation.
Q/Q(M)-477887 Report a Problem

Regarding the inheritance of Ehlers-Danlos syndrome, which subtype is inherited in an X-linked
recessive manner?
1

The type with the lysyl oxidase deficiency
2

The type with the lysyl hydroxylase deficiency
3

The type with a collagen 5 defect
4

The type with a collagen 3 defect
5

The type with a defect in procollagen aminopeptidase
Q/Q(M)-477709 Report a Problem

Regarding the inheritance of Ehlers-Danlos syndrome, which subtype is inherited in an X-linked
recessive manner?
1

The type with the lysyl oxidase deficiency
Lysyl oxidase deficiency is related to type V or X-linked Ehlers-Danlos syndrome (EDS). Type IX,
also has x-linked recessive inheritance with mild symptoms of EDS with occipital exostoses and
hernias. Lysyl hydroxylase deficiency is seen in type VI EDS, linked with severe kyphoscoliosis,
retinal detachment and other eye abnormalities. Collagen V deficiencies are seen in type I (Gravis) and
64

type II (Mitis) EDS, associated with skin elasticity, gaping wounds, hypermobile joints, Gorlins sign,
blue sclera and mitral valve prolapse.
Q/Q(M)-477709 Report a Problem

A 27 year old presents with some mildly itchy spots on his posterior neck. Biopsy reveals elastosis
perforans serpiginosa. Other notable exam findings include tall stature, joint laxity, pectus excavatum,
and a high-arched palate. Which gene is most likely mutated in this patient?
1

Fibrillin 1
2

Fibronectin
3

Collagen 5
4

ABCC6
5

Fibulin 4
Q/Q(M)-482700 Report a Problem

A 27 year old presents with some mildly itchy spots on his posterior neck. Biopsy reveals elastosis
perforans serpiginosa. Other notable exam findings include tall stature, joint laxity, pectus excavatum,
and a high-arched palate. Which gene is most likely mutated in this patient?
1

Fibrillin 1
Marfan syndrome is an autosomal dominant disorder characterized by tall stature, arachnodactyly,
pectus excavatum, high-arched palate, joint laxity, ectopia lentis, aortic dilatation, striae, and elastosis
perforans serpiginosa. The syndrome is caused by a defect in fibrillin 1 or 2. Fibronectin is defected in
Type 10 Ehlers-Danlos syndrome. Collagen 5 defects can cause Ehlers-Danlos types 1 or 2. ABCC6 is
mutated in pseudoxanthoma elasticum. Cutis laxa is caused by defects in fibulin 4.
Q/Q(M)-482700 Report a Problem

In patients with diffuse congenital hemangiomatosis, the most common site for extracutaneous
involvement is the :
1

Liver
2

Thyroid
3

Lungs
4

Colon
5

Brain
Q/Q(M)-474057 Report a Problem

In patients with diffuse congenital hemangiomatosis, the most common site for extracutaneous
involvement is the :
1

Liver
Diffuse congenital hemangiomatosis is characterized by multiple hemangiomas with the liver being the
most common extracutaneous site, followed by the lungs. Liver hemangioma may be complicated by
hepatomegaly, obstructive jaundice, and portal hypertension.
Q/Q(M)-474057 Report a Problem

Collagen III is mutated in which type(s) of Ehlers-Danlos syndrome (EDS)?
65

1

All of the answers are correct
2

None of the answers are correct
3

Benign Hypermobile (type III)
4

Vascular (type IV)
5

Periodontitis (type VIII)
Q/Q(M)-477878 Report a Problem

Collagen III is mutated in which type(s) of Ehlers-Danlos syndrome (EDS)?
1

All of the answers are correct
Collagen III is mutated in all three types of EDS listed. Benign hypermobile type EDS is associated
with hypermobile joints and is autosomal dominant (AD) in transmission. Vascular type EDS is
associated with arterial and visceral rupture leading to early death, and visible venous patterns. It is
transmitted autosomal recessive (AR) or AD. Periodontitis type EDS is associated with mild EDS
symptoms and periodontitis.
Q/Q(M)-477878 Report a Problem

A patient with a white, spongy overgrowth of the buccal mucosa that has passed in an autosomal
dominant fashion is most likely related to a mutation in which of the following?
1

Keratin 1/10
2

Keratin 6b/17
3

Keratin 4/13
4

Keratin 6a/16
5

None of these options are correct
Q/Q(M)-477963 Report a Problem


A patient with a white, spongy overgrowth of the buccal mucosa that has passed in an autosomal
dominant fashion is most likely related to a mutation in which of the following?
3

Keratin 4/13
This description is most likely a white sponge nevus, an autosomal dominant defect in keratin 4/13.
Keratin 1/10 is mutated in epidermolytic hyperkeratosis and Unna-Thost PPK, Keratin 6a/16 in
pachyonychia congenita type I, and keratin 6b/17 in pachyonychia congenita type II.
Q/Q(M)-477963 Report a Problem

Nevoid basal carcinoma syndrome (Gorlin syndrome) is autosomal dominant transmitted mutation of
the patched gene. Symptoms include innumberable basal cell carcinomas, painful odontogenic jaw
keratocysts, palmoplantar pits, frontal bossing, bifid ribs and what other bony abnormality?
1

Polyostotic fibrous dysplasia
2

Stippled epiphyses
3

Calcification of falx cerebri
66

4

Osteopoikilosis
5

Sphenoid wing dysplasia
Q/Q(M)-477745 Report a Problem

Nevoid basal carcinoma syndrome (Gorlin syndrome) is autosomal dominant transmitted mutation of
the patched gene. Symptoms include innumberable basal cell carcinomas, painful odontogenic jaw
keratocysts, palmoplantar pits, frontal bossing, bifid ribs and what other bony abnormality?
3

Calcification of falx cerebri
Calcification of falx cerebri is seen in Gorlin's syndrome. CHILD syndrome and chondrodysplasia
punctata both can exhibit stippled epiphyses. Polyostotic fibrous dysplasia is found in McCune-
Albright syndrome, osteopoikilosis in seen in Buschke-Ollendorf syndrome. Sphenoid wing dysplasia
is seen in neurofibromatosis type I.
Q/Q(M)-477745 Report a Problem

Cronkhite-Canada is associated with gastrointestinal polyposis, nail atrophy, alopecia, generalized
pigmentation of skin and melanotic macules on the fingers. Which of the following describes its
inheritance pattern?
1

Autosomal recessive
2

Autosomal dominant
3

X-linked recessive
4

X-linked dominant
5

Sporadic
Q/Q(M)-478001 Report a Problem

Cronkhite-Canada is associated with gastrointestinal polyposis, nail atrophy, alopecia, generalized
pigmentation of skin and melanotic macules on the fingers. Which of the following describes its
inheritance pattern?
5

Sporadic
Cronkhite-Canada is associated with gastrointestinal polyposis, nail atrophy, alopecia, generalized
pigmentation of skin and melanotic macules on the fingers. It is inherited in a sporadic manner.
Q/Q(M)-478001 Report a Problem

Epistaxis in early childhood to teens followed by multiple cutaneous and gastrointestinal telangectasias
describes which of the following syndromes?
1

Hereditary Hemorrhagic Telangiectasia syndrome
2

Maffucci syndrome
3

CREST syndrome
4

Ataxia telangectasia
5

Fabry disease
Q/Q(M)-477810 Report a Problem

67

Epistaxis in early childhood to teens followed by multiple cutaneous and gastrointestinal telangectasias
describes which of the following syndromes?
1

Hereditary Hemorrhagic Telangiectasia syndrome
Hereditary Hemorrhagic Telangiectasia syndrome is described above. The first signs in over 50% of
cases is epistaxis in childhood to young adulthood. Telangectiasias develop in the 30's and 40's. Other
findings include gastrointestinal telangiectasia, hepatic and pulmonary arteriovenous malformations.
The other syndromes listed can have cutaneous vascular lesions and should be considered on the
differential for hereditary hemorrhagic telangiectasia syndrome.
Q/Q(M)-477810 Report a Problem

A patient with colon cancer is diagnosed with Muir-Torre syndrome. Which of the following cutaneous
lesions might the patient have?
1

Keratoacanthomas
2

Seborrheic keratoses
3

Tricholemmomas
4

Arsenical keratoses
5

Basal cell carcinomas
Q/Q(M)-477363 Report a Problem

A patient with colon cancer is diagnosed with Muir-Torre syndrome. Which of the following cutaneous
lesions might the patient have?
1

Keratoacanthomas
Muir-Torre syndrome is an autosomal dominant disease caused by mutations in MSH2 and MLH1,
DNA mismatch repair genes. Clinically, patients have multiple sebaceous tumors (adenomas are most
common), keratoacanthomas, and are at risk for adenocarcinoma of the colon.
Q/Q(M)-477363 Report a Problem

What medication may exacerbate this autosomally dominant, acnatholytic disorder?
1

Phenytoin
2

Lithium
3

Oral contraceptives
4

Anti-malarials
5

Corticosteroids
Q/Q(M)-476828 Report a Problem
68



What medication may exacerbate this autosomally dominant, acnatholytic disorder?
2

Lithium
Darier's disease is autosomal dominant condition characterized by hyperkeratotic papules coalescing
into warty plaques and cobblestoned papules on mucosal surfaces. The cutaneous manifestations may
be exacerbated by lithium.
Q/Q(M)-476828 Report a Problem

Keratins 1 and 10 are important intermediate filaments in suprabasal keratinocytes. Genetic mutation of
these keratins can lead to which of the following disorders?
1

Netherton syndrome
2

Sjogren-Larsson syndrome
3

Pachyonychia congenita type 1
4

Epidermolytic hyperkeratosis / Bullous ichthyosiform erythroderma
5

White sponge nevus
Q/Q(M)-482406 Report a Problem

Keratins 1 and 10 are important intermediate filaments in suprabasal keratinocytes. Genetic mutation of
these keratins can lead to which of the following disorders?
4

Epidermolytic hyperkeratosis / Bullous ichthyosiform erythroderma
Epidermolytic hyperkeratosis is caused by a genetic mutation in keratins 1 and 10. White sponge nevus
is caused by a genetic mutation in keratins 4 and 13. Pachyonychia congenita is caused by a genetic
mutation in keratins 6a and 16. Netherton syndrome and Sjogren-Larsson syndrome are not disorders of
keratins.
Q/Q(M)-482406 Report a Problem

In ataxia telangectasia, the ATM gene is mutated. The product of the ATM gene is an enzyme which:
1

Responds to DNA damage by phosphorylating key DNA repair substrates
2

Binds transforming growth factor beta protein
3

Is the VEGF receptor 3
4

Participates in NF-kB activation
5

Is an inhibitor of G1 cyclin/Cdk complexes
69

Q/Q(M)-477808 Report a Problem

In ataxia telangectasia, the ATM gene is mutated. The product of the ATM gene is an enzyme which:
1

Responds to DNA damage by phosphorylating key DNA repair substrates
The ATM gene is a member of the phosphatidylinositol-3 family of proteins that respond to DNA
damage by phosphorylating key substrates involved in DNA repair according to OMIM. Defects in
endoglin (TGF beta3 binding protein) is deficient in Osler-Weber-Rendu syndrome. The VEGF
receptor 3 is defective in hereditary lymphedema. The NEMO gene is defective in Incontinentia
Pigmenti. Its product, NF-kB essential modulator (NEMO) is a key activator in the NF-kB pathway.
KIP2 is involved in AD Beckwith-Wiedemann syndrome and is an inhibitor of G1 cyclin/Cdk
complexes.
Q/Q(M)-477808 Report a Problem

Which malignancy is associated with Cowden syndrome?
1

Colon cancer
2

Renal cancer
3

Lung cancer
4

Ovarian cancer
5

Basal cell cancer
Q/Q(M)-477792 Report a Problem

Which malignancy is associated with Cowden syndrome?
1

Colon cancer
Cowden syndrome is a autosomal dominant syndome with tricholemmomas, oral mucosal papules,
acral keratotic papules, thyroid goiter, GI polyps, and fibrocystic breast disease. Malignant associations
include breast, thyroid follicular carcinoma, and colon adenocarcinoma.
Q/Q(M)-477792 Report a Problem

Which cutaneous finding is seen in patients with phenylketonuria?
1

Angular stomatitis
2

Ichthyosis
3

Pigment dilution of hair and skin
4

Phyrnoderma
5

Erosive diaper dermatitis
Q/Q(M)-474033 Report a Problem

Which cutaneous finding is seen in patients with phenylketonuria?
3

Pigment dilution of hair and skin
Phenylketonuria is an autosomal recessive condition caused by a mutation in the gene coding for
phenylalanine hydroxylase. Defect in this enzyme results in accumulation of phenylalanine and its
metabolites. Increased phenylalanine has toxic effects on the central nervous system in addition to
competitively inhibiting tyrosine in melanogenesis. Inhibition of melanogenesis results in pigmentary
dilution of the hair and skin. Other features of this condition include a predisposition to eczema,
sclerodermoid changes of the skin, urine that has a distinctive "mousy" odor, psychomotor delay,
mental retardation, seizures and hyperreflexia. A low-phenylalanine diet instituted early on can prevent
70

these manifestations of the disease. The morbidity of phenylketonuria has improved since the advent of
routine neonatal screening for this condition.
Q/Q(M)-474033 Report a Problem

Dermatofibrosis lenticularis disseminata is seen in which of the following conditions?
1

Ehlers-Danlos syndrome
2

Marfan syndrome
3

Pseudoxanthoma elasticum
4

Focal dermal hypoplasia
5

Buschke-Ollendorf syndrome
Q/Q(M)-474280 Report a Problem

Dermatofibrosis lenticularis disseminata is seen in which of the following conditions?
5

Buschke-Ollendorf syndrome
Buschke-Ollendorf syndrome is an autosomal dominant disorder characterized by dermatofibrosis
lenticularis disseminata (cutaneous elastomas distributed symmetrically over the buttocks, trunk and
proximal extremities), and osteopoikilosis (round opacities in bones).
Q/Q(M)-474280 Report a Problem
Dermatofibrosis lenticularis disseminata and osteopoikilosis are findings seen with mutations of which
of the following genes?
1

LEMD3
2

Fibrillin 2
3

ABCC6
4

Lysyl hydroxylase
5

Lysyl oxidase
Q/Q(M)-477899 Report a Problem

Dermatofibrosis lenticularis disseminata and osteopoikilosis are findings seen with mutations of which
of the following genes?
1

LEMD3
Buschke-Ollendorf syndrome is caused by a loss-of-function mutation in LEMD3.
Q/Q(M)-477899 Report a Problem

Which gene is most commonly mutated in melanoma on non sun-damaged skin?
1

BRAF
2

NRAS
3

C-kit
4

CTLA-4
71

5

CDKN2A
Q/Q(M)-482760 Report a Problem

Which gene is most commonly mutated in melanoma on non sun-damaged skin?
1

BRAF
Regarding the genetics of melanoma, BRAF mutations are more common than NRAS mutations on non
sun-damaged skin. C-kit is associated with mucosal melanoma. Acral melanoma are associated with c-
kit more commonly than with BRAF.
Q/Q(M)-482760 Report a Problem

A 16 month-old girl presents with patchy alopecia, whorled erythematous scaly eruption, and
asymmetric limb shortening. What laboratory or radiologic test may aid in diagnosis?
1

Brain MRI
2

Alkaline phosphatase
3

Chest radiograph
4

Bone films
5

Complete blood count
Q/Q(M)-477872 Report a Problem

A 16 month-old girl presents with patchy alopecia, whorled erythematous scaly eruption, and
asymmetric limb shortening. What laboratory or radiologic test may aid in diagnosis?
4

Bone films
The patient has Conradi-Hunermann Syndrome. This is a X-linked dominant disorder characterized by
ichthyosiform erythroderma in Blaschko's lines in infancy which resolves with follicular atrophoderma,
patchy alopecia, short stature, cataracts, scoliosis, assymetric limb shortening. Bone films will
demonstrate stippled epiphyses. Ichthyosis and stippled epiphyses resolve after infancy.
Q/Q(M)-477872 Report a Problem

The finding of 'maltese crosses' in the urine is characteristic of which of the following conditions?
1

Alkaptonuria
2

Fabry disease
3

Gaucher disease
4

Neimann-Pick disease
5

Hunter syndrome
Q/Q(M)-478042 Report a Problem

The finding of 'maltese crosses' in the urine is characteristic of which of the following conditions?
2

Fabry disease
The 'maltese cross' finding in urine is characteristic of Fabry disease. Alkaptonuria will show dark urine
with a pH > 7.0. There are no urinary findings in Hunter syndrome, Gaucher or Neimann-Pick disease.
Q/Q(M)-478042 Report a Problem
72

A 9 year old boy has sparse, short, brittle hair. A blood test reveals low serum copper. What is the
inheritance pattern of this disease?
1

X-linked recessive
2

X-linked dominant
3

Autosomal dominant
4

Autosomal recessive
5

Mitochondrial
Q/Q(M)-482706 Report a Problem

A 9 year old boy has sparse, short, brittle hair. A blood test reveals low serum copper. What is the
inheritance pattern of this disease?
1

X-linked recessive
Menkes kinky hair syndrome is an x-linked recessive disorder caused by defects in ATP7A, an ATP-
dependent copper transporter. It is characterized by pili torti, trichorrhexis nodosa, short, britle hair, lax
skin, CNS deterioration, seizures, and tortuous arteries.
Q/Q(M)-482706 Report a Problem

A 4 year old girl suffers multiple fractures. She also has thin skin, easy bruising, and blue sclera. An
echocardiogram reveals mitral valve prolapse. What type of osteogenesis imperfecta does she most
likely have?
1

Type I
2

Type II
3

Type III
4

Type IV
5

Type V
Q/Q(M)-482705 Report a Problem


A 4 year old girl suffers multiple fractures. She also has thin skin, easy bruising, and blue sclera. An
echocardiogram reveals mitral valve prolapse. What type of osteogenesis imperfecta does she most
likely have?
1

Type I
Osteogenesis imperfect is a defect in collagen I which leads to thin skin, easy bruising, blue sclera, and
multiple fractures. Mitral valve prolapse is seen especially in Type I.
Q/Q(M)-482705 Report a Problem

Findings of eyelid papules (string of pearls) and a hoarse cry in infants is characteristic of which of the
following syndromes?
1

Lipoid proteinosis
2

Amyloidosis
3

Pseudoxanthoma elasticum
4

Disseminated xanthomas
73

5

None of these answers are correct
Q/Q(M)-477900 Report a Problem

Findings of eyelid papules (string of pearls) and a hoarse cry in infants is characteristic of which of the
following syndromes?
1

Lipoid proteinosis
Findings of the eyelid string of pearls and a hoarse cry during the first years of life (due to vocal cord
infiltration) is characteristic of Lipoid Proteinosis (AKA Urbach-Wiethe disease or Hyalinosis cutis et
mucosae). It is an autosomal recessive condition with mutations in the extracellular matrix protein 1
gene. Other findings include calcifications of the temporal lobe and hippocampus, hairloss, atrophic
scars and waxy papules on the face, verrucous nodules and a thick tongue. The other conditions could
be considered on the differential for Lipoid Proteinosis, but do not have the findings described above.
Q/Q(M)-477900 Report a Problem

Which of the following eye findings is caused by the rupture of Bruch's membrane?
1

Angioid streaks
2

Blue sclerae
3

Retinal detachment
4

Ruptured globe
5

Keratoconus
Q/Q(M)-477897 Report a Problem
Which of the following eye findings is caused by the rupture of Bruch's membrane?
1

Angioid streaks
The rupture of Bruch's membrane causes angioid streaks in pseudoxanthoma elasticum. Bruch's
membrane is the innermost layer of choroid with a central layer of elastic fibers. The other findings are
found in Ehlers-Danlos syndrome and are not related to Bruch's membrane.
Q/Q(M)-477897 Report a Problem
The x-linked recessive type of dyskeratosis congenita is:
1

Dyskerin
2

TERC
3

CDKN2A
4

PTEN
5

Menin
Q/Q(M)-478020 Report a Problem

The x-linked recessive type of dyskeratosis congenita is:
1

Dyskerin
The dyskerin gene, whose product is involved in ribosomal RNA synthesis, is mutated in X-linked
recessive dyskeratosis congenita. TERC is linked with autosomal domininant transmission of the
syndrome. CDKN2A is involved in familial dysplastic nevi/melanoma syndrome, PTEN in Cowden
74

syndrome and Menin in MEN type I.
Q/Q(M)-478020 Report a Problem

A patient presents with several light blue cyst-like lesions on the eyelid. They consult their list of
problems and bring up plantar hyperkeratosis and dysplastic toenails. On oral exam, you note that they
have both upper and lower dentures. The patient relates that after losing their "baby teeth", only 3 teeth
grew in their place. What syndrome does this person most likely have?
1

Schopf-Schulz-Passarge
2

Gardner syndrome
3

Hypohidrotic ectodermal dysplasia
4

Cowden syndrome
5

Cronkhite-Canada
Q/Q(M)-477995 Report a Problem

A patient presents with several light blue cyst-like lesions on the eyelid. They consult their list of
problems and bring up plantar hyperkeratosis and dysplastic toenails. On oral exam, you note that they
have both upper and lower dentures. The patient relates that after losing their "baby teeth", only 3 teeth
grew in their place. What syndrome does this person most likely have?
1

Schopf-Schulz-Passarge
Schopf-Schulz-Passarge syndrome is associated with hydrocystomas of the eyelids, hypotrichosis (near
complete loss of hair early in life), hypodontia, nail abnormalities and multiple palmoplantar eccrine
syringofibroadenomas. The other listed syndromes do not fit the description above.
Q/Q(M)-477995 Report a Problem

Ivory-colored papules between the angles of the scapulae are characteristic of which syndrome:
1

Hurler
2

Scheie
3

Morquio
4

Hunter
5

Sanfilippo
Q/Q(M)-478068 Report a Problem

Ivory-colored papules between the angles of the scapulae are characteristic of which syndrome:
4

Hunter
These syndromes are all mucopolysaccharidoses. These papules are characteristic of Hunter syndrome
which is caused by a deficiency in iduronate sulfatase.
Q/Q(M)-478068 Report a Problem
Angioid streaks on retinal exam are characteristic of which of the following syndromes?
1

Pseudoxanthoma elasticum
75

2

Choroid malformations
3

Eyelid papillomas
4

Lester iris
5

Salt & pepper retinitis pigmentosa
Q/Q(M)-477898 Report a Problem

Angioid streaks on retinal exam are characteristic of which of the following syndromes?
1

Pseudoxanthoma elasticum
Angioid streaks are characteristic of pseudoxanthoma elasticum. They are caused by rupture of Bruch's
membrane of the choroid. Choroid malformations are found in Sturge-Weber syndrome, eyelid
papillomas in xeroderma pigmentosum, Lester iris in Nail-patella syndrome and salt & pepper retinitis
pigmentosa in Refsum syndrome.
Q/Q(M)-477898 Report a Problem

What is the most common ocular findings seen in this condition?
1

Angioid streaks
2

Phakomas
3

Lisch nodules
4

Bitots spots
5

Coloboma
Q/Q(M)-482073 Report a Problem


What is the most common ocular findings seen in this condition?
1

Angioid streaks
Pseudoxanthoma elasticum, PXE, is an inherited disorder that affects selected connective tissue in
some parts of the body. Elastic tissue in the body becomes mineralized, that is, calcium and other
minerals are deposited in the tissue. The most common ocular finding is angioid streaks. The other
ocular findings are seen in these syndromes: Phakomas is seen in tuberous sclerosis, Lisch nodules in
neurofibromatosis, Bitots spots in Vitamin A deficiency and coloboma in focal dermal hypoplasia.
Q/Q(M)-482073 Report a Problem

Hyperextesible skin, gaping wounds, cigarette-paper scars, molluscoid pseudotumors, and calcified
subcutaneous nodules are characteristic of which syndrome?
1

Marfan syndrome
2

Congenital contractural arachnodactyly
76

3

Ehlers-Danlos Syndrome
4

Cutis laxa
5

Pseudoxanthoma elasticum
Q/Q(M)-474274 Report a Problem


Hyperextesible skin, gaping wounds, cigarette-paper scars, molluscoid pseudotumors, and calcified
subcutaneous nodules are characteristic of which syndrome?
3

Ehlers-Danlos Syndrome
The most common type of EDS is EDS type I (gravis). Cutaneous features include hyperextensible
skin, gaping wounds, cigarette-paper scars, molluscoid pseudotumors, calcified subcutaneous nodules,
and bruises. Systemic features include hypermobile joints with dislocation, hernias, mitral valve
prolapse, blue sclerae, Gorlins sign (tongue reaches nose), and absence of the lingual frenulum.
Q/Q(M)-474274 Report a Problem

Defects in Fibrillin 2 are linked with:
1

Congenital contractural arachnodactyly
2

Cutis Laxa
3

Arthrochalasis multiplex congenita
4

Occipital horn syndrome
5

Lipoid proteinosis
Q/Q(M)-477819 Report a Problem

Defects in Fibrillin 2 are linked with:
1

Congenital contractural arachnodactyly
Fibrillin 2 defects arelinked primarily with congenital contractural arachnodactyly. This syndrome is
associated with long limbs, arachnodactyly, scoliosis and crumpled ears. Occasionally, fibrillin 2 can
be associated with Marfan syndrome also. The other conditions are not linked to fibrillin mutations.
Q/Q(M)-477819 Report a Problem

Which finding would you expect in a child with dyskeratosis congenita?
1

Increased risk for pancreatic cancers
2

Oral leukoplakia
3

Hirsutism
4

Normal hematologic status
5

Normal nails
Q/Q(M)-478021 Report a Problem

Which finding would you expect in a child with dyskeratosis congenita?
2

Oral leukoplakia
Mild to moderate mental retardation occurs in up to 50% of cases. A Fanconi-type pancytopenia may
occur resulting in early death. These children have alopecia, not hirsutism, along with nail dystrophy.
77

There is an increased risk for development of premalignant oral leukoplakia, not pancreatic
malignancy.
Q/Q(M)-478021 Report a Problem

The most common neoplasm seen in Maffucci Syndrome is:
1

Enchondromas
2

Angiosarcomas
3

Osteosarcomas
4

Lymphangiosarcomas
5

chondrosarcoma
Q/Q(M)-474050 Report a Problem

The most common neoplasm seen in Maffucci Syndrome is:
1

Enchondromas
Maffucci syndrome comprises of superficial and deep venous malformations, enchondromas, and short
stature. Enchondromas are the most common neoplasm, while chondrosarcomas are the most common
malignancies.
Q/Q(M)-474050 Report a Problem
A teenage female presents with the complaint of "nail fungus". On exam, she has triangular lunulae,
palmoplantar hyperhidrosis, micronychia and an absent patella. Which of the following gene defects is
most likely in this patient?
1

LMX1B
2

COL5A1
3

EEC1
4

LKB1
5

NEMO
Q/Q(M)-481625 Report a Problem
A teenage female presents with the complaint of "nail fungus". On exam, she has triangular lunulae,
palmoplantar hyperhidrosis, micronychia and an absent patella. Which of the following gene defects is
most likely in this patient?
1

LMX1B
This patient has nail-patella syndrome. The gene defect is an autosomally transmitted defect in
LMX1B. This gene is involved in dorsal/ventral limb patterning and is in close proximit to the
COL5A1 gene. EEC1 is defective in ectrodactyly-ectodermal dysplasia-cleft lip/palate syndrome and
LKB1 in Peutz Jeghers syndrome.
Q/Q(M)-481625 Report a Problem


Which of the following is defective in Ehlers-Danlos syndrome (EDS) with congenital adrenal
hyperplasia?
1

Tenascin-X
2

Lysyl oxidase
78

3

Lysyl hydroxylase
4

None of these answers are correct
5

All of these answers are correct
Q/Q(M)-477879 Report a Problem


Which of the following is defective in Ehlers-Danlos syndrome (EDS) with congenital adrenal
hyperplasia?
1

Tenascin-X
Tenascin-X defects are associated with EDS and with congenital adrenal hyperplasia. The phenotype is
that of typical EDS with hyperextensible skin, hypermobile joints, and tissue fragility. Lysyl oxidase is
defective in X-linked EDS (type V) and Occipital horn syndrome (type IX). Lysyl hydroxylase is
defective in ocular-scoliotic (type VI) EDS.
Q/Q(M)-477879 Report a Problem


What is the mode of transmission for lamellar ichthyosis?
1

Autosomal dominant
2

Autosomal recessive
3

X-linked dominant
4

X-linked recessive
5

Sporadic
Q/Q(M)-477853 Report a Problem

What is the mode of transmission for lamellar ichthyosis?
2

Autosomal recessive
Lamellar ichthyosis which is characterized by collodian membrane in newborns and platelike scale in
children and adults is an autosomal recessive syndrome. The gene defect is transglutaminase 1
(TGM1).
Q/Q(M)-477853 Report a Problem

Ehlers-Danlos Syndrome with congenital adrenal hyperplasia is caused by mutations affecting which of
the following?
1

Lysyl hydroxylase
2

Collagen 5
3

Fibronectin
4

Lysyl oxidase
5

Tenascin-X
Q/Q(M)-474276 Report a Problem
Ehlers-Danlos Syndrome with congenital adrenal hyperplasia is caused by mutations affecting which of
the following?
79

5

Tenascin-X
Ehlers-Danlos Syndrome type with congenital adrenal hyperplasia is caused by mutations in tenascin-
X.
Q/Q(M)-474276 Report a Problem

What is the classic radiologic findings associated with this disorder?
1

Dural calcifications
2

Calcifications of the falx-cerebri
3

Tram-track calcifications of the temporal and occipital cortex
4

Osteopatha striata
5

Osteopoikilosis
Q/Q(M)-476642 Report a Problem


What is the classic radiologic findings associated with this disorder?
3

Tram-track calcifications of the temporal and occipital cortex
Sturge-Weber syndrome is a sporadic disroder characterized by a facial capillary malformation in a
trigeminal nerve distribution. Patients with Sturge-Weber may have cerebral atrophy, ipsilateral
vascular malformations of the leptomeninges, seizures, and glaucoma. The classic radiologic finding is
tram-track calcifications of the temporal and occipital cortex.
Q/Q(M)-476642 Report a Problem

What is the genetic defect of this syndrome?
1

ABCA12 gene
2

Transglutaminase 1 (TGM1).
3

GJB2 (connexin 26) gene.
4

Steroid sulfatase (STS) gene
5

Profilaggrin gene
Q/Q(M)-482080 Report a Problem
80


What is the genetic defect of this syndrome?
1

ABCA12 gene
Mutations in the ABCA12 gene cause harlequin ichthyosis. X-linked ichthyosis (XLI) caused by a
steroid sulfatase (STS) deficiency. Keratitis-ichthyosis-deafness syndrome is caused by a mutation in
the GJB2 (connexin 26) gene. Ichthyosis vulgaris is caused by a defect in profilaggrin gene and
lamellar ichthyosis involves a mutation in the gene for transglutaminase 1 (TGM1).
Q/Q(M)-482080 Report a Problem

A 3 year old boy has white hair of the central frontal scalp and depigmented symmetrical patches on the
knees since birth. No ocular abnormalities or deafness are noted. What is true of this disorder?
1

It is an autosomal recessive disorder of melanocyte development
2

There is typically no progression of depigmented patches
3

Regression of the white forelock has been noted
4

One form of this condition is associated with iris pigmentary abnormalities
5

Results from a mutation in a tumor suppressor gene
Q/Q(M)-482513 Report a Problem

A 3 year old boy has white hair of the central frontal scalp and depigmented symmetrical patches on the
knees since birth. No ocular abnormalities or deafness are noted. What is true of this disorder?
2

There is typically no progression of depigmented patches
The patient in this scenario has piebaldism, an autosomal dominant disorder of pigmentation caused
most commonly by dominant negative missense mutations in the KIT proto-oncogene. Piebaldism
almost always has no progression of the depigmented patches, and is thought to be a static disoder. One
exception was noted in a family with a novel Val620Ala (1859T>C) mutation in the KIT gene where
development of new depigmented patches did occur. Regression of the white forelock has been noted in
select cases. Piebaldism is thought to be unresponsive to systemic treatments or phototherapy. If the
patient presented with either eye abnormalities or deafness, Waardenburg syndrome would be
considered in the diagnosis.
Q/Q(M)-482513 Report a Problem

Which of the following is NOT a complication of Kasabach-Merritt Syndrome:
1

Thrombocytopenia
81

2

CHF
3

Ataxia
4

Disseminated intravascular coagulation
5

Gastrointestinal bleeding
Q/Q(M)-474049 Report a Problem

Which of the following is NOT a complication of Kasabach-Merritt Syndrome:
3

Ataxia
Kasabach-Merritt Syndrome results from platelet trapping due to the presence of a single or multiple
large hemangiomas. Hematologic complications include thrombocytopenia, microangiopathic
hemolytic anemia, DIC, and acute hemorrhage. The presence of large hemangiomas leads to high
output failure (CHF) and can also compress on surrounding structures.
Q/Q(M)-474049 Report a Problem

Which disease is found more commonly in mothers of patients with chronic granulomatous disease?
1

Sarcoidosis
2

Erythema nodosum
3

Churg-Straus disease
4

Wegener's disease
5

Discoid lupus erythematous
Q/Q(M)-477512 Report a Problem

Which disease is found more commonly in mothers of patients with chronic granulomatous disease?
5

Discoid lupus erythematous
Female carriers of chronic granulomatous disease have an increase incidence of discoid lupus,
infections and apthous stomatitis.
Q/Q(M)-477512 Report a Problem


The characteristic dental findings in patients with tuberous sclerosis are:
1

Peg teeth
2

Anodontia
3

Enamel pits
4

Odontogenic cysts
5

Retention of primary teeth
Q/Q(M)-474051 Report a Problem

The characteristic dental findings in patients with tuberous sclerosis are:
3

Enamel pits
Enamel pits are the characteristic dental findings in tuberous sclerosis. Peg teeth are found in
incontinentia pigmenti and anhidrotic ectodermal dysplasia. Anodontia is found in hypomelanosis of ito
and incontinentia pigmenti. Odontogenic cysts are seen in Gorlin syndrome, and retention of primary
teeth is characteristic of Job syndrome.
82

Q/Q(M)-474051 Report a Problem

A 50 year man presents with generalized metallic-grey hyperpigmentation. His past medical history
includes diabetes, hepatomegaly and arrythmias. Laboratory tests should include:
1

Copper levels
2

Lead levels
3

Iron levels
4

Arsenic levels
5

Cyanide levels
Q/Q(M)-474027 Report a Problem

A 50 year man presents with generalized metallic-grey hyperpigmentation. His past medical history
includes diabetes, hepatomegaly and arrythmias. Laboratory tests should include:
3

Iron levels
Hemochromatosis is an autosomal recessive disease resulting in increased intestinal iron absorption and
iron deposition in a variety of organs. Clinical features include generalized metallic-grey
hyperpigmentation, koilonychia, sparse or absent hair, hepatomegaly, cardiac failure/arrhythmias,
insulin-dependent diabetes, hypogonadism and polyarthritis
Q/Q(M)-474027 Report a Problem

The following enzyme defect is most commonly seen in CHILD Syndrome.
1

3-beta-hydroxysteroid dehydrogenase
2

3-beta-hydroxysteroid isomerase
3

Aryl sulfatase E
4

NAD oxido reductase
5

DNA helicase
Q/Q(M)-477876 Report a Problem
The following enzyme defect is most commonly seen in CHILD Syndrome.
1

3-beta-hydroxysteroid dehydrogenase
CHILD Syndrome is a X-linked dominant disorder characterized by unilateral ichthyosiform
erythroderma, ipsilateral limb deformity, and ipsilateral organ hypoplasia. The most commom gene
defect is NSDHL which encodes 3-beta hydroxysteroid dehydrogenase. EBP gene defects which
encode 3-beta-hydroxysteroid isomerase have been described, however this is the usual defect in
Conradi-Hunermann Syndrome. Aryl sulfatase E is mutated in X-linked recessive chondrodysplasia
punctata.
Q/Q(M)-477876 Report a Problem

The combination of gastrointestinal polyposis, nail atrophy, alopecia, generalized pigmentation of skin,
and melanotic macules of the fingers is characteristic of which of the following syndromes?
1

Nicolau-Balus syndrome
2

Peutz-Jeghers syndrome
3

Cronkhite-Canada syndrome
83

4

Cowden syndrome
5

Bannayan-Riley-Ruvalcaba syndrome
Q/Q(M)-474289 Report a Problem

The combination of gastrointestinal polyposis, nail atrophy, alopecia, generalized pigmentation of skin,
and melanotic macules of the fingers is characteristic of which of the following syndromes?
3

Cronkhite-Canada syndrome
Cronkhite-Canada syndrome is a sporadic gastrointestinal polyposis syndrome associated with nail
atrophy, alopecia, generalized pigmentation of the skin, and melanotic macules on the fingers.
Q/Q(M)-474289 Report a Problem

A child with phenylketonuria likely presents with which cutaneous problems?
1

Blue-gray generalized hyperpigmentation
2

Alopecia universalis
3

Generalized hypopigmentation
4

Generalized hyperpigmentation
5

Leg ulcers
Q/Q(M)-474030 Report a Problem



A child with phenylketonuria likely presents with which cutaneous problems?
3

Generalized hypopigmentation
Phenylketonuria is an autsomal recessive disorder caused by a mutation on the long arm of
chromosome 12. A deficiency of phenylalanine hydroxylase or its cofactor tetrahydrobiopterin leads to
accumulation of phenylalanine. Clinical features include generalized hypopigmentation, eczematous
dermatitis, sclerodermoid changes, seizures, psychomotor delay, urine with mousy odor, mental
retardation.
Q/Q(M)-474030 Report a Problem

Mutations in calcium transporters cause which pair of diseases?
1

Erythrokeratodermia variabilis and progressive symmetric erythrokeratodermia
2

Lamellar ichthyosis and nonbullous congenital ichthyosiform erythroderma
3

Refsum syndrome and Sjogren-Larsson syndrome
4

Chondrodysplasia punctata and CHILD syndrome
5

Dariers disease and Hailey-Hailey disease
Q/Q(M)-474247 Report a Problem

Mutations in calcium transporters cause which pair of diseases?
5

Dariers disease and Hailey-Hailey disease
Dariers disease and Hailey-Hailey disease are caused by mutations in the SERCA calcium ATPase.
84

The former is characterized by hyperkeratotic papules in seborrheic areas, palmar keratoses and pits,
red-white longitudinal nail bands, v-shaped distal nail nicks, and cobblestoning of oral and rectal
mucosae. The latter is characterized by acantholytic erosions in skin folds.
Q/Q(M)-474247 Report a Problem


Which of the following is caused by a mutation in a gene that leads to defective NF-KB activation?
1

MEN IIa
2

Piebaldism
3

Chediak-Higashi
4

Vohwinkels
5

Incontinentia pigmenti
Q/Q(M)-474021 Report a Problem

Which of the following is caused by a mutation in a gene that leads to defective NF-KB activation?
5

Incontinentia pigmenti
Incontinential pigmenti is an X-linked dominant disorder caused by a mutation in the NEMO gene
located at Xq28. NEMO is an NF-KB modulator. A genetic defect in NEMO leads to defective
activation of NF-KB (a transcription factor). MEN IIa is caused by a mutation in the RET proto-
oncogene, piebaldism is caused by the C-kit protocogene, chediak-higashi is caused by a mutation in
lysosomal tracking, and Vohwinkels is caused by a defect in the loricrin gene, which codes for a
structural protein.
Q/Q(M)-474021 Report a Problem

The presence of natal teeth and pincer nails suggests which disease entity?
1

Congenital syphillis
2

Thalidomide exposure in utero
3

Incontinentia pigmenti
4

Pachyonychia congenita
5

Anhidrotic ectodermal dysplasia
Q/Q(M)-475873 Report a Problem

The presence of natal teeth and pincer nails suggests which disease entity?
4

Pachyonychia congenita
Pachyonychia congentia is an autosomal dominant condition characterized by a constellation of
findings affecting ectodermal structures. These include the presence of natal teeth, steatocystoma
multiplex, follicular hyperkeratosis of the knees, elbows and extensor extremities, eruptive vellus hair
cysts, and oral leukokeratosis which is not pre-malignant. In addition, nail findings include twenty-nail
dystrophy, subungual hyperkeratosis with increase transverse curvature ("pincer nails") and candidal
paronychia. There are two forms of pachyonychia congenital: Type 1 (Jadassohn-Lewandowsky
syndrome) caused by defects in keratin 6a and 16, and Type 2 (Jackson-Lawler type) caused by defects
in keratins 6b and 17. Anhidrotic ectodermal dysplasia is associated with peg-shaped teeth,
hypoanodontia, and a non-specific nail dystrophy. Likewise, incontientia pigmenti also is characterized
by anodontia and peg-shaped teeth and dystrophic changes of the nail. Finally congenital syphilis is a
85

well-recognized cause of pegged teeth. Limb deformities are the most serious sequelae of thalidomide
exposure in utero.
Q/Q(M)-475873 Report a Problem

"Mousy" odor of urine is characteristic of which of the following syndromes?
1

Phenylketonuria
2

Hunter disease
3

Alkaptonuria
4

Maple Syrup Urine disease
5

Hurler disease
Q/Q(M)-478067 Report a Problem

"Mousy" odor of urine is characteristic of which of the following syndromes?
1

Phenylketonuria
Phenylketonuria will cause a "mousy" odor in the urine. Patients with Alkaptonuria will have black
urine. Maple syrup urine disease will have a sweet odor. Hunter/Hurler diseases are not associated with
urinary issues.
Q/Q(M)-478067 Report a Problem

A 17 y/o man presents with facial acne that he would like treated. You notice that he has fine brown
scale on his neck and do a complete skin exam. This scale is present on the remainder of his body,
sparing his palms, soles and flexural areas. He informs you that his uncles on his mothers side have
similar skin findings. He is not concerned about the skin and would like to proceed with acne treatment
only. What other clinical exam should you perform to screen for a potential malignancy to which this
patient is at higher risk for acquiring?
1

Testicular
2

Abdominal
3

Lymph node
4

Lung
5

Rectal
Q/Q(M)-477718 Report a Problem

A 17 y/o man presents with facial acne that he would like treated. You notice that he has fine brown
scale on his neck and do a complete skin exam. This scale is present on the remainder of his body,
sparing his palms, soles and flexural areas. He informs you that his uncles on his mothers side have
similar skin findings. He is not concerned about the skin and would like to proceed with acne treatment
only. What other clinical exam should you perform to screen for a potential malignancy to which this
patient is at higher risk for acquiring?
1

Testicular
Men with x-linked ichthyosis are at increased risk of testicular cancer and cryptorchidism. A testicular
exam is simple to perform and a good screening exam for detecting testicular abnormalities. The
remaining exams are not useful as these patients are not at higher risk for other types of cancer.
Q/Q(M)-477718 Report a Problem
86


Ichthyosiform erythroderma in lines of Blaschko, follicular atrophoderma, and stippled epiphyses are
characteristic of which of the following disorders?
1

Refsum syndrome
2

Chondrodysplasia punctata
3

Erythrokeratodermia variabilis
4

Netherton syndrome
5

Sjogren-Larsson syndrome
Q/Q(M)-474246 Report a Problem

Ichthyosiform erythroderma in lines of Blaschko, follicular atrophoderma, and stippled epiphyses are
characteristic of which of the following disorders?
2

Chondrodysplasia punctata
Chondrodysplasia punctata is a peroxisomal disorder caused by mutations in several genes that is
associated with ichthyosiform erythroderma in lines of Blaschko, follicular atrophoderma, and stippled
epiphyses.
Q/Q(M)-474246 Report a Problem

Coast of Maine caf au lait macules are characteristic of which condition
1

Hypomelanosis of Ito
2

Carney complex
3

McCune-Albright syndrome
4

Gauchers syndrome
5

Tuberous sclerosis
Q/Q(M)-474255 Report a Problem
Coast of Maine caf au lait macules are characteristic of which condition
3

McCune-Albright syndrome
McCune-Albright syndrome is a sporadic condition caused by somatic mutations in the Gs subunit of
adenylate cyclase. Key clinical features include coast of Maine caf au lait macules, polyostotic
fibrous dysplasia, and precocious puberty.
Q/Q(M)-474255 Report a Problem


A 7 year old girl with abnormally short hair has progressive seizures, lethargy, ataxia, and mental
retardation. A blood test reveals elevated levels of ammonia. What is the affected gene?
1

Argininosuccinase
2

ATP7A
3

Ectodysplasin A
4

Connexin 30
5

p63
Q/Q(M)-482708 Report a Problem
87



A 7 year old girl with abnormally short hair has progressive seizures, lethargy, ataxia, and mental
retardation. A blood test reveals elevated levels of ammonia. What is the affected gene?
1

Argininosuccinase
Argininosuccinic aciduria is an autosomal recessive disorder caused by mutations in argininosuccinase.
It is characterized by trichorrhexis nodosa, hyperammonemia, hepatomegaly and vomiting, seizures,
lethargy, coma, ataxia, and mental retardation.
Q/Q(M)-482708 Report a Problem

A patient with Klinefelter Syndrome may be expected to experience which of the following:
1

Recurrent pulmonary infections
2

Recurrent leg ulcers
3

Pulmonary valve stenosis
4

Gastroesophageal reflux
5

Scarring alopecia
Q/Q(M)-474018 Report a Problem

A patient with Klinefelter Syndrome may be expected to experience which of the following:
2

Recurrent leg ulcers
Klinefelter syndrome results from nondisjunction during meiosis, leading to the XXY genotype.
Patients are characteristically tall (long lower extremities) with scant body and pubic hair. Klinefelter
patients have numerous varicosities predisposing them to recurrent leg ulcers.
Q/Q(M)-474018 Report a Problem

Crumpled ears are associated with which disorder?
1

Ehlers-Danlos Syndrome
2

Marfan syndrome
3

Congenital contractural arachnodactyly
4

Cutis laxa
5

Buschke-Ollendorf syndrome
Q/Q(M)-474277 Report a Problem


Crumpled ears are associated with which disorder?
3

Congenital contractural arachnodactyly
Congenital contractural arachnodactyly is an autosomal dominant disorder caused by mutations in
fibrillin 2. Affected patients have long limbs, arachnodactyly, scoliosis, and crumpled ears.
Q/Q(M)-474277 Report a Problem

Anodontia is a bone finding seen in which of the following conditions:
88

1

Hypomelanosis of Ito
2

Letterer-Siwe disease
3

Tuberous sclerosis
4

Jackson Sertoli syndrome
5

Hyper-IgE syndrome
Q/Q(M)-477738 Report a Problem

Anodontia is a bone finding seen in which of the following conditions:
1

Hypomelanosis of Ito
Hypomelanosis of Ito, or Incontinentia pigmenti achromians is a condition characterized by marble-
cake hypopigmentation, epilepsy, alopecia, scoliosis and mental/motor retardation. The characteristic
dental abnormality is anodontia. The remaining syndromes are not associated with anodontia.
Q/Q(M)-477738 Report a Problem

Which of the following is caused by a mutation in a gene which codes for steroid sulfatase?
1

Fabry disease
2

Lesch-nyhan disease
3

X-linked ichthyosis
4

Lamellar icthyosis
5

Chediak-Higashi
Q/Q(M)-474055 Report a Problem


Which of the following is caused by a mutation in a gene which codes for steroid sulfatase?
3

X-linked ichthyosis
The mutation in X-linked icthyosis is found in the gene for aryl sulfatase C, a steroid sulfatase. The
genetic defect in Fabry disease occurs in alpha-galactosidase A (which hydrolyzes glycolipids and
glycoproteins), the defect in lamellar icthyosis codes for transglutaminase 1 and the defect in chediak
higashi occurs in a lysosomal transport protein.
Q/Q(M)-474055 Report a Problem

A thirty-year-old woman presents with new progressively worsening headaches. Upon exam, you
notice multiple acral keratotic papules and papillomas of the tongue. What malignancy does this patient
need to be screened for?
1

colon cancer
2

ovarian cancer
3

breast cancer
4

basal cell carcinoma
5

melanoma
Q/Q(M)-482450 Report a Problem
89


A thirty-year-old woman presents with new progressively worsening headaches. Upon exam, you
notice multiple acral keratotic papules and papillomas of the tongue. What malignancy does this patient
need to be screened for?
3

breast cancer
This patient has Lhermitte-Duclos disease, which is a hamartomatous overgrowth of cerebellar
ganglion cells. Approximately half of patients have Cowden syndrome. Breast cancer is the correct
answer, which affects 25-35% of female patients. Patients can also develop thyroid and genitourinary
carcinoma (endometrial, urethral, renal cell, and transitional cell carcinoma of the renal pelvis).
Malignant degeneration of hamartomatous colon polyps is rare.
Q/Q(M)-482450 Report a Problem


A double row of eyelashes is associated with:
1

Lymphedema-distichiasis syndrome
2

Cornelia de Lange syndrome
3

Rubinstein-Taybi syndrome
4

Russell-Silver syndrome
5

Hunters syndrome
Q/Q(M)-477816 Report a Problem

A double row of eyelashes is associated with:
1

Lymphedema-distichiasis syndrome
A double row of eyelashes is defined as distichiasis and is associated with the Lymphedema-distichiasis
syndrome. This syndrome is transmitted in an autosomal dominant fashion and is related to a mutation
in FOXC2. Findings include late onset lymphedema, distichiasis, corneal irritation, ectropion, webbed
neck and congenital heart defects. The remaining syndromes do not include distichiasis as a feature.
Q/Q(M)-477816 Report a Problem

Findings of milia, cylindromas and the condition shown in the pathology image are characteristic of
which of the following syndromes?
1

Gorlin's syndrome
2

Familial cylindromatosis
3

Brook-Spiegler syndrome
4

Rasmusen syndrome
5

Rombo syndrome
Q/Q(M)-477990 Report a Problem
90



Findings of milia, cylindromas and the condition shown in the pathology image are characteristic of
which of the following syndromes?
3

Brook-Spiegler syndrome
Brooke-Spiegler syndrome is an uncommon disease with a predisposition to develop cutaneous adnexal
neoplasms such as cylindromas, trichoepitheliomas, spiradenomas, trichoblastomas, basal-cell
carcinomas, follicular cysts, organoid nevi, and malignant transformation of pre-existing tumors in the
affected individuals.
Q/Q(M)-477990 Report a Problem

Retinal hemangioblastomas are found in which syndrome:
1

Osler-Weber-Rendu disease
2

Von-Hippel Lindau disease
3

Kasabach-Merritt syndrome
4

Klippel-Trenaunay Weber syndrome
5

Sturge-Weber syndrome
Q/Q(M)-476964 Report a Problem

Retinal hemangioblastomas are found in which syndrome:
2

Von-Hippel Lindau disease
Von Hippel-Lindau syndrome is an autosomal dominant condition caused by a defect in the VHL
tumor suppressor gene. This disease is characterized by retinal hemangioblastomas, often resulting in
visual impairment and blindness if left untreated. In addition, many tumors are seen including
pheochromocytoma, renal cell carcinoma, and hemangioblastomas of the cerebellum, medulla, and
spinal cord. Pancreatic and renal cysts are also a feature of this condition. Finally, polycythemia can
occur as a result of erythropoietin production by renal cell carcinoma. Von Hippel-Lindau syndrome is
a progressive, universally fatal condition which presents most often in the fourth decade of life.
Q/Q(M)-476964 Report a Problem

The most common cutaneous association with monilethrix is:
1

Eczema
2

Hypopigmentation
3

Hyperpigmentation
4

Keratosis Pilaris
5

Atrophy
91

Q/Q(M)-474045 Report a Problem

The most common cutaneous association with monilethrix is:
4

Keratosis Pilaris
Monilethrix is an autosomal dominant condition which, by definition, presents with beaded hear.
Clinically, patients present with short, sparse lusterless hair. Keratosis pilaris is the most common
associated feature.
Q/Q(M)-474045 Report a Problem


Lamellar ichthyosis is caused by a defect in transglutaminase 1. It can present as a colloidion baby at
birth with subsequent large thick plates of scale especially on flexures, ectropion and eclabium. If two
unaffected carrier parents have a child, how likely is their child to have this condition?
1

25%
2

75%
3

Only male offspring are affected
4

Only female offspring are affected
5

None of these answers are correct
Q/Q(M)-477719 Report a Problem

Lamellar ichthyosis is caused by a defect in transglutaminase 1. It can present as a colloidion baby at
birth with subsequent large thick plates of scale especially on flexures, ectropion and eclabium. If two
unaffected carrier parents have a child, how likely is their child to have this condition?
1

25%
Lamellar ichthyosis is an autosomal recessive condition. If each parent is a heterozygous carrier, there
is a 25% chance that the child will be affected, 50% chance that the child will be a heterozygous carrier
and a 25% chance that the child will not be a carrier or affected with lamellar ichthyosis.
Q/Q(M)-477719 Report a Problem


Enchondromas and chondrosarcomas are most strongly associated with which of the following
syndromes?
1

Proteus syndrome
2

Osler-Weber-Rendu syndrome
3

Maffucci syndrome
4

Nonne-Milroy disease
5

Blue rubber bleb nevus syndrome
Q/Q(M)-474264 Report a Problem


Enchondromas and chondrosarcomas are most strongly associated with which of the following
syndromes?
3

Maffucci syndrome
92

Maffucci syndrome is a sporadic condition caused by defects in the parathyroid hormone/parathyroid
hormone related protein type 1 receptor. There are venous malformations of distal extremities, and
benign enchondromas that can degenerate into chondrosarcomas.
Q/Q(M)-474264 Report a Problem

The combination of painful palmoplantar keratoderma and pseudoherpetic keratitis is characteristic of
which of the following syndromes?
1

Naxos syndrome
2

Vohwinkel syndrome
3

Richner-Hanhart syndrome
4

Howel-Evans syndrome
5

Schopf-Schulz-Passarge syndrome
Q/Q(M)-474250 Report a Problem

The combination of painful palmoplantar keratoderma and pseudoherpetic keratitis is characteristic of
which of the following syndromes?
3

Richner-Hanhart syndrome
Richner-Hanhart syndrome (tyrosenemia type II) is an autosomal recessive disorder caused by a
deficiency in hepatic tyrosine aminotransferase. This disease is characterized by painful PPK,
pseudoherpetic keratitis and blindness. Treatment is low-tyrosine/phenylalanine diet.
Q/Q(M)-474250 Report a Problem

A patient with hypohydrosis and hyperpyrexia, anodontia, and sparse hair has which syndrome:
1

Dyskeratosis congenita
2

Pachyonychia congenita
3

Anhidrotic ectodermal dysplasia
4

Hidrotic ectodermal dysplasia
5

Papillon Lefevre
Q/Q(M)-474048 Report a Problem


A patient with hypohydrosis and hyperpyrexia, anodontia, and sparse hair has which syndrome:
3

Anhidrotic ectodermal dysplasia
Anhidrotic ectodermal dysplasia is an x-linked recessive disorder which presents with the triad of
hypohydrosis (or anhidrosis) with hyperpyrexia, anodontia (other dental findings include peg teeth,
molars with hooked cusps) and sparse hair. Patients tend to overheat.
Q/Q(M)-474048 Report a Problem

Focal Dermal Hypoplasia (Goltz Syndrome) can differentiated from Incontinentia Pigmenti by
1

Type of inheritance
93

2

Presence of Linear lesions along the lines of Blaschko
3

Presence of blistering lesions
4

Hair and teeth abnormality
5

Eye and CNS abnormality
Q/Q(M)-482137 Report a Problem


Focal Dermal Hypoplasia (Goltz Syndrome) can differentiated from Incontinentia Pigmenti by
3

Presence of blistering lesions
Both Focal Dermal Hypoplasia (Goltz Syndrome) and Incontinentia Pigmenti are inherited as X linked
dominant (lethal in males).And both can have lesions along the lines of Blaschko with many
similarities in systemic involvement. Howevere, Incontinentia Pigmenti is differntiated from Focal
Dermal Hypoplasia by presence of blistering lesions in addition to hyperkeratosis and
hyperpigmentation.
Q/Q(M)-482137 Report a Problem


Germline KILLIN methylation is associated with which of the following syndromes?
1

Bannayan-Riley-Ruvalcaba Syndrome
2

Proteus Syndrome
3

Cowden Syndrome
4

Birt-Hogg-Dube Syndrome
5

Galli-Galli Disease
Q/Q(M)-482404 Report a Problem


Germline KILLIN methylation is associated with which of the following syndromes?
3

Cowden Syndrome
Loss-of-function mutations in phosphatase and tensin homolog gene (PTEN) mutations cause 80% of
Cowden Syndrome, a rare autosomal-dominant disorder, characterized by high risks of breast, thyroid,
and other cancers. Other mechanisms of loss of function such as hypermethylation of the KILLIN gene
has been identified in Cowden Syndrome. Bannayan-Riley-Ruvalcaba and Proteus syndrome are
associated with PTEN loss-of-function mutations. Birt-Hogg-Dube is associated with loss-of-function
mutations in FLCN gene. Galli-Galli Disease is not a malignancy-associated disease and is the result of
KRT5 mutations.
Q/Q(M)-482404 Report a Problem

All of the following disorders are exacerbated by UV radiation except:
1

Bloom syndrome
2

Hartnups disease
3

Refsum syndrome
4

Cockayne syndrome
5

Rothmund-Thomopson syndrome
94

Q/Q(M)-477217 Report a Problem


All of the following disorders are exacerbated by UV radiation except:
3

Refsum syndrome
Refsums syndrome is an autosomal recessive disorder caused by mutations in phytanoyl-CoA
hydroxylase. Clinically, patients have mild icthyosis, cerebellar ataxia, polyneuropathy, salt and pepper
retinitis pigmentosa, sensorineural deafness, and arrhythmias with heart block. They are not overly
sensitive to UV radiation.
Q/Q(M)-477217 Report a Problem


This form of EB simplex has a defect in actin assembly, not keratin formation.
1

Dowling Meara
2

Weber-Cockayne
3

Koebner
4

Kindler
5

Ogna
Q/Q(M)-482207 Report a Problem

This form of EB simplex has a defect in actin assembly, not keratin formation.
4

Kindler
Kindler syndrome has been reclassified as a subtype of EB simplex due to congenital blistering being
the first symptom. These patients do not demonstrate defects in keratin proteins. Instead, the KIND1
gene defect leads to disruption of actin assembly.
Q/Q(M)-482207 Report a Problem



Non-bullous icthyosiform erythroderma is caused by which of the following mutations:
1

Transglutaminase-1 gene (TGM1)
2

12R-lipoxygenase gene (ALOX12B)
3

Lipoxygenase-3 gene (ALOXE3)
4

Both 12R-lipoxygenase gene (ALOX12B) and lipoxygenase-3 gene (ALOXE3) are correct
5

All of these answers are correct
Q/Q(M)-478118 Report a Problem

Non-bullous icthyosiform erythroderma is caused by which of the following mutations:
5

All of these answers are correct
Non-bullous congenital erythroderma (NCIE)is an autosomal recessive disorder characterized by a
collodion baby presentation at birth, and generalized erythroderma with fine white scale, palmoplantar
keratoderma, and heat intolerance. NCIE may be caused by mutations in transglutaminase-1 gene
(TGM1), the 12R-lipoxygenase gene (ALOX12B), and the lipoxygenase-3 gene (ALOXE3). Mutations
in the keratinocyte TGM1 gene interferes with normal cross-linking of structural proteins and the lipid
95

envelope, leading to defective cornification and desquamation. ALOXE3 functions as an epoxy alcohol
synthase using the product of ALOX12B as the preferred substrate; either gene can be the site of
mutations causing NCIE.
Q/Q(M)-478118 Report a Problem

A 5 year old boy is being seen by neurology for seizures and by ENT for a hoarse voice. He is referred
to dermatology for evaluation of verrucous nodules of the elbows and knees. While reviewing his chart,
it is noted that he has calcifications in the hippocampus seen on imaging. What is the most likely
diagnosis of this patient?
1

Lipoid proteinosis
2

Psoriasis
3

Severe combined immunodeficiency disorder
4

Poland syndrome
5

Vohwinkel syndrome
Q/Q(M)-482703 Report a Problem
A 5 year old boy is being seen by neurology for seizures and by ENT for a hoarse voice. He is referred
to dermatology for evaluation of verrucous nodules of the elbows and knees. While reviewing his chart,
it is noted that he has calcifications in the hippocampus seen on imaging. What is the most likely
diagnosis of this patient?
1

Lipoid proteinosis
Lipoid proteinosis is an autosomal recessive disorder caused by defects in the extracellular matrix
protein 1 gene. It is characterized by scars and yellow papules of the face and oropharynx, yellowish
papules on the eyelid margin, hoarse voice, verrucous nodules of the elbows and knees, and
calcification of the temporal lobe and hippocampus with occasional seizures.
Q/Q(M)-482703 Report a Problem

A BSCL2 gene mutation with the cutaneous findings of generalized lipodystrophy, hyperlipemia,
hepatomegaly, acanthosis nigricans, elevated basal metabolic rate and non-ketotic insulin resistant
diabetes mellitus are characteristic of which of the following syndromes?
1

Berardinelli-Seip congenital lipodystrophy
2

Familial partial lipodystrophy
3

Bjornstad syndrome
4

All of the answers are correct
5

None of the answers are correct
Q/Q(M)-477903 Report a Problem
A BSCL2 gene mutation with the cutaneous findings of generalized lipodystrophy, hyperlipemia,
hepatomegaly, acanthosis nigricans, elevated basal metabolic rate and non-ketotic insulin resistant
diabetes mellitus are characteristic of which of the following syndromes?
1

Berardinelli-Seip congenital lipodystrophy
Berardinelli-Seip congenital lipodystrophy is described above. Familial partial lipodystrophy is
characterized by a defect in LMNA and has symmetric lipoatrophy of trunk and limbs with sparing of
neck, shoulders, buffalo hump area and genitalia, tuboeruptive xanthomas, acanthosis nigricans and
96

hypertriglyceridemia. Bjornstad syndrome is characterized by pili torti and deafness.
Q/Q(M)-477903 Report a Problem


Which of the following is a potentially serious complication of the blue rubber bleb nevus syndrome?
1

Development of chondrosarcomas
2

Development of angiosarcomas
3

Gastrointestinal hemorrhage
4

Development of lymphedema
5

Development of fluid retention
Q/Q(M)-474266 Report a Problem


Which of the following is a potentially serious complication of the blue rubber bleb nevus syndrome?
3

Gastrointestinal hemorrhage
Blue rubber bleb nevus syndrome is characterized by multiple tender venous malformations of skin and
gastrointestinal tract, which can lead to gastrointestinal bleeding.
Q/Q(M)-474266 Report a Problem

Ectopia lentis (downward displacement of the lens) is characteristic of:
1

Marfan syndrome
2

Homocystinuria
3

Phenylketonuria
4

Multiple Carboxylase deficiency
5

Ehlers-Danlos syndrome
Q/Q(M)-478065 Report a Problem


Ectopia lentis (downward displacement of the lens) is characteristic of:
2

Homocystinuria
Ectopia lentis (downward displacement) is seen in homocystinuria. Upward displacement is seen in
Marfan syndrome. There are no changes in the lens in phenylketonuria or multiple carboxylase
deficiency.
Q/Q(M)-478065 Report a Problem

A seven month old infant diagnosed with eczema on her face returns for a diaper-rash follow-up. A
one-month trial of topical antifungals has failed to improve the infants systems. The part of the
physical exam that might prove most useful include:
1

Stool samples
2

Fontanelle examination
3

Examination of palms and soles
4

Palpation of abdomen
97

5

Hearing test
Q/Q(M)-474028 Report a Problem

A seven month old infant diagnosed with eczema on her face returns for a diaper-rash follow-up. A
one-month trial of topical antifungals has failed to improve the infants systems. The part of the
physical exam that might prove most useful include:
3

Examination of palms and soles
Acrodermatitis enteropathica presents itself in infancy once breastfeeding has stopped. It can clinically
mimic atopic dermatitis, seborrheic dermatitis and candidiasis. Clinical features include scaly red rash
around mouth, eyes, and palms, diarrhea, stomatitis, glossitis, alopecia, and failure to thrive.
Q/Q(M)-474028 Report a Problem

Tyrosinase positive albinism (oculocutaneous albinism type 2) is caused by a mutation in which of the
following:
1

Tyrosinase
2

P gene
3

Tyrosinase related protein 1
4

C-kit
5

NEMO
Q/Q(M)-474252 Report a Problem

Tyrosinase positive albinism (oculocutaneous albinism type 2) is caused by a mutation in which of the
following:
2

P gene
Oculocutaneous albinism (OCA) type 1 (Tyrosinase negative albinism) is caused by mutations in the
tyrosinase gene. OCA type 2 (tyrosinase positive albinism) is caused by mutations in the P gene. OCA
type 3 is caused by mutations in the tyrosinase related protein 1 gene. C-kit mutations cause piebaldism
and NEMO mutations cause incontinentia pigmenti.
Q/Q(M)-474252 Report a Problem

In which of the following Genodermatoses would one find cutaneous hyperpigmentation, blue lunulae
and Kayser-Fleishcher rings:
1

Marfan's Disease
2

Hemochromatosis
3

Gaucher's Disease
4

Wilson's Disease
5

Osteogenesis Imperfecta
Q/Q(M)-478695 Report a Problem

In which of the following Genodermatoses would one find cutaneous hyperpigmentation, blue lunulae
and Kayser-Fleishcher rings:
4

Wilson's Disease
98

In Wilson's disease (Hepatolenticular Degeneration) one will find a vague greenish discoloration of the
skin on the face, neck, and gentalia Hyperpigmentation), azure lunulae (sky-blue moons) of the nails,
and Kayser-Fleischer rings. This is due to the body retaining excessive amounts of copper.
Q/Q(M)-478695 Report a Problem

Comma-shaped corneal opacities are characteristic of which type of ichthyosis?
1

Ichthyosis vulgaris
2

X-linked ichthyosis
3

Lamellar ichthyosis
4

Nonbullous congenital ichthyosiform erythroderma
5

Refsum syndrome
Q/Q(M)-474242 Report a Problem

Comma-shaped corneal opacities are characteristic of which type of ichthyosis?
2

X-linked ichthyosis
X-linked ichthyosis patients have comma-shaped corneal opacities that are asymptomatic yet highly
characteristic.
Q/Q(M)-474242 Report a Problem

A 4-year old boy presents with generalized white scale. The mother reports that her son was born with
a tight membrane enveloping his body. Peripheral blood smear is within normal limits. What is the
most likely diagnosis?
1

Congenital ichthyosiform erythroderma
2

Neutral lipid storage disease
3

Lamellar ichthyosis
4

Netherton syndrome
5

Ichthyosis vulgaris
Q/Q(M)-477854 Report a Problem

A 4-year old boy presents with generalized white scale. The mother reports that her son was born with
a tight membrane enveloping his body. Peripheral blood smear is within normal limits. What is the
most likely diagnosis?
1

Congenital ichthyosiform erythroderma
The most likely diagnosis is Congenital ichthyosiform erythroderma. In neutral lipid storage disease,
the peripheral blood smear would demonstrate lipid vacuoles in leukocytes and monocytes. Lamellar
ichthyosis is characterized by plate-like scale in children/adults. Netherton syndrome is characterized
by ichthyosis linearis circumflexa. Ichthyosis vulgaris does not typically present with collodian baby.
Q/Q(M)-477854 Report a Problem

A 7 year old boy presents to your office with short and sparse hair. He is also short for his age. A hair
99

mount reveals alternating light and dark bands under polarized light. Which of the following statements
is correct regarding this condition?
1

Eyebrows and eyelashes are not affected
2

There is no increased risk of skin cancer
3

Amino acid analysis of hair reveals high sulfur content
4

There is no increased hair fragility.
5

Dark bands represent air
Q/Q(M)-482487 Report a Problem

A 7 year old boy presents to your office with short and sparse hair. He is also short for his age. A hair
mount reveals alternating light and dark bands under polarized light. Which of the following statements
is correct regarding this condition?
2

There is no increased risk of skin cancer
The patient described has trichothiodystrophy, with the hair finding of trichoschisis. These patients
have photosensitivity but no increased risk of skin cancer. They also have intellectual impairment,
decreased fertility, short stature, and progeria-like facies. Low sulfur content of the hair is found.
Eyebrows and eyelashes are also affected. Dark bands in pili annulati, not trichoschisis, are air cavities.
Q/Q(M)-482487 Report a Problem

Which of the following findings is characteristic of a mutation in lamin A?
1

Lipoatrophic sclerodermoid skin
2

Alopecia
3

Craniomegaly with small face
4

Severe premature atherosclerosis with early death
5

All of the answers are correct
Q/Q(M)-477901 Report a Problem

Which of the following findings is characteristic of a mutation in lamin A?
5

All of the answers are correct
A mutation in Lamin A causes Progeria (Hutchinson-Gilford syndrome). Other findings include nail
atrophy and muscle/bone wasting. Presentation is in the first or second year of life. An increased urine
hyaluronic acid can be helpful in diagnosis.
Q/Q(M)-477901 Report a Problem

Which of the following conditions is worsened by ingestion of lithium?
1

Dariers Disease
2

Hailey-Hailey Disease
3

Haim-Munk syndrome
4

Hereditary lymphedema (Nonne-Milroy disease)
5

Epidermolytic hyperkeratosis
Q/Q(M)-477721 Report a Problem
100


Which of the following conditions is worsened by ingestion of lithium?
1

Dariers Disease
Patients with Dariers disease should not be treated with lithium due to its worsening or in some cases
unmasking the disease. The mechanism for this is not known. Lithium treatment does not worsen the
other listed conditions.
Q/Q(M)-477721 Report a Problem

The arylsulfatase C gene is mutated in which disease?
1

X-linked ichthyosis
2

Refsum syndrome
3

Haim-Munk syndrome
4

Naxos syndrome
5

Griscelli syndrome
Q/Q(M)-477717 Report a Problem

The arylsulfatase C gene is mutated in which disease?
1

X-linked ichthyosis
Arylsulfatase C is also known as steroid sulfatase and is mutated in X-linked ichthyosis. This condition
is inherited in a X-linked recessive pattern. Clinical findings include: brown scale sparing palms, soles
and flexures, comma-shaped corneal opacities, failure of labor progression and cryptorchidism. It is
also mutated in X-linked recessive type chondrodysplasia punctata.
Q/Q(M)-477717 Report a Problem

Sphenoid wing dysplasia is seen in:
1

Tuberous sclerosis
2

Tay Syndrome
3

Mafucci syndrome
4

NF-1
5

NF-2
Q/Q(M)-474054 Report a Problem

Sphenoid wing dysplasia is seen in:
4

NF-1
Sphenoid wing dysplasia is seen in neurofibromatosis type I. Patients with Tay syndrome have short
stature, patients with tuberous sclerosis have phalangeal cysts and periosteal thickening, patients with
Mafucci syndrome have enchondromas and short stature, and patients with NF-2 do not have any
characteristic musculoskeletal findings.
Q/Q(M)-474054 Report a Problem

A 16 year-old girl presents with a family history of Gardner syndrome. Her mother is very concerned
that her daughter may have the syndrome as it runs in her family and she has many skin complaints.
101

What is likelihood that this girl has Gardner syndrome based on what you now about the inheritance
pattern and the fact that her father is unaffected and her mother is a heterozygote for this condition?
1

50%
2

25%
3

10%
4

75%
5

90%
Q/Q(M)-481631 Report a Problem

A 16 year-old girl presents with a family history of Gardner syndrome. Her mother is very concerned
that her daughter may have the syndrome as it runs in her family and she has many skin complaints.
What is likelihood that this girl has Gardner syndrome based on what you now about the inheritance
pattern and the fact that her father is unaffected and her mother is a heterozygote for this condition?
1

50%
Her mother is a heterozygote for the condition and her father is homozygous as a non-carrier. Gardner
syndrome is passed in an autosomal dominant fashion. She has a 50% chance of also being a
heterozygote from her mother and a 50% chance of being completely unaffected.
Q/Q(M)-481631 Report a Problem

To help diagnose trichothiodystrophy, which of the following levels are decreased in hairs of affected
individuals?
1

Arginine
2

Histidine
3

Phenylalanine
4

Glycine
5

Cysteine
Q/Q(M)-477951 Report a Problem


To help diagnose trichothiodystrophy, which of the following levels are decreased in hairs of affected
individuals?
5

Cysteine
Cysteine and Methionine levels are decreased in hair and nails of patients with trichothiodystrophy.
Other sulfur containing amino acids include: Gluthathione, Taurine, and Homocysteine. Testing hairs
for decreased sulfur content is an indirect method of determining this. The other listed amino acids are
present in normal levels in the hair and nails of trichothiodystrophy patients.
Q/Q(M)-477951 Report a Problem

What is the gene defect in harlequin fetus?
1

Transglutaminase
102

2

Steroid sulfatse
3

ABCA12
4

ABCC6
5

None of these answers are correct
Q/Q(M)-477855 Report a Problem

What is the gene defect in harlequin fetus?
3

ABCA12
Harlequin fetus is an autosomal recessive disorder. The gene defect is ABCA12.
Q/Q(M)-477855 Report a Problem

What of the following is present in desmoplastic melanoma?
1

b-raf
2

c-myc
3

c-kit
4

Ras
5

p53
Q/Q(M)-482858 Report a Problem


What of the following is present in desmoplastic melanoma?
3

c-kit
C-kit staining is present in desmoplastic melanoma. It is also seen in acral melanocytic lesions. Ras is
found in all melanocytic lesions. P53 mutations are found in actinic keratoses, SCCIS, and SCC.
Q/Q(M)-482858 Report a Problem


Ichthyosis with confetti is a severe, sporadic ichthyosis caused by mutations in which gene?
1

Keratin 1
2

Keratin 2e
3

Keratin 10
4

Keratin 14
5

Keratin 15
Q/Q(M)-482534 Report a Problem

Ichthyosis with confetti is a severe, sporadic ichthyosis caused by mutations in which gene?
3

Keratin 10
Ichthyosis with confetti, a severe, sporadic skin disease in humans, is caused by mutations in the gene
encoding keratin 10 (KRT10); all result in frameshifts into the same alternative reading frame,
producing an arginine-rich C-terminal peptide that redirects keratin 10 from the cytokeratin filament
103

network to the nucleolus.
Q/Q(M)-482534 Report a Problem

What is the most likely gene mutation in this individual who has migratory patches and fixed plaques
as depicted in this picture?
1

Connexin 26
2

Connexin 30.3 and 31
3

Calcium ATPase 2C1
4

Loricrin
5

Keratins 1 and 10
Q/Q(M)-476636 Report a Problem


What is the most likely gene mutation in this individual who has migratory patches and fixed plaques
as depicted in this picture?
2

Connexin 30.3 and 31
Erythrokeratodermis variabilis is an autosomal dominant disease characterized by transient patches of
geographic erythema and fixed hyperkeratotic plaques. The disease is due to gene defects in connexin
30.3 and 31.
Q/Q(M)-476636 Report a Problem

Which of the following condition is NOT found in Von-Hippel Lindau syndrome?
1

Connective tissue nevi
2

Bilateral retinal hemangioblastomas
3

Cerebellar/CNS hemangioblastomas
4

Renal cell carcinoma
5

Pheochromocytoma
Q/Q(M)-477781 Report a Problem

Which of the following condition is NOT found in Von-Hippel Lindau syndrome?
1

Connective tissue nevi
Von Hippel-Lindau syndrome is characterized by all the options listed except connective tissue nevi.
Other findings include pancreatic cysts/carcinoma and cutaneous capillary malformations of the head
and neck and polycythemia.
Q/Q(M)-477781 Report a Problem
104


A patient with this autosomal recessive disorder caused by a defect in helicase is an increased risk for
which malignancy?
1

Acute leukemia
2

Renal cell carcinoma
3

Medullary thyroid carcinoma
4

Squamous cell carcinoma of the lung
5

Prostate carcinoma
Q/Q(M)-476830 Report a Problem


A patient with this autosomal recessive disorder caused by a defect in helicase is an increased risk for
which malignancy?
1

Acute leukemia
Bloom's syndrome is an autosomal recessive disorder caused by a mutation in DNA helicase. It is
characterized by photodistributed erythema in a butterfly distribution, malar hypoplasia with a
prominent nose, high pitched voice, and an increased risk for malignancy (acute leukemia, lymphoma,
and GI adenocarcinoma.)
Q/Q(M)-476830 Report a Problem

A 11 year-old female patient with hypoparathyroidism is referred to your clinic secondary to chronic
mucocutanous candidiasis which is refractory to standard treatments. The patient also has
malabsorption and severe chronic diarrhea. You determine that she has autoimmune
polyendocrinopathy-candiasis-ectodermal dystrophy syndrome(APECED). Since only two of three
major criteria are needed to make this diagnosis, this patient having demonstrated hypoparathyroidism
and chronic mucocutanous candidiasis is diagnosed with APECED. What is the third major criteria that
would qualify a patient to meet the diagnosis of APECED?
1

Ectodermal dysplasia
2

Insulin dependent diabetes mellitus
3

Chronic autoimmune hepatitis
4

Addison's disease
5

Hypothyroidism
Q/Q(M)-481618 Report a Problem
105

A 11 year-old female patient with hypoparathyroidism is referred to your clinic secondary to chronic
mucocutanous candidiasis which is refractory to standard treatments. The patient also has
malabsorption and severe chronic diarrhea. You determine that she has autoimmune
polyendocrinopathy-candiasis-ectodermal dystrophy syndrome(APECED). Since only two of three
major criteria are needed to make this diagnosis, this patient having demonstrated hypoparathyroidism
and chronic mucocutanous candidiasis is diagnosed with APECED. What is the third major criteria that
would qualify a patient to meet the diagnosis of APECED?
4

Addison's disease
This patient has APECED, also know as autoimmune polyendocrinopathy syndrome type 1(APS type
1). Two of three major criteria are needed to make this diagnosis - chronic mucocutaneous candidiasis,
hypoparathyroidism and Addison's disease. They usually present within the first 5 years of life with
chronic mucocutaneous candidiasis, then before the age of ten with hypoparathyroidism, and finally in
early adulthood with Addison's disease. They often present with chronic diarrhea and malabsorption.
Ectodermal dysplasia usually does not present until the fifth decade. Autoimmune skin disease, such as
vitiligo and alopecia areata are not uncommon.
Q/Q(M)-481618 Report a Problem
A patient with thyroid carcinoma and cobblestone-like changes of the oral mucosa will also likely have:
1

Trichoepitheliomas
2

Fibrofolliculomas
3

Tricholemmomas
4

Cylindromas
5

Syringomas
Q/Q(M)-476965 Report a Problem
A patient with thyroid carcinoma and cobblestone-like changes of the oral mucosa will also likely have:
3

Tricholemmomas
The patient described may have Cowden's syndrome, an autosomal dominant condition caused by a
defect in the PTEN tumor suppressor gene. Patients with Cowden's disease are at increased risk for
thyroid and breast carcinoma. In addition, they characteristically have multiple hamartomatous polyps
of the gastrointestinal tract that are typically benign. Cutaneous features of Cowden's syndrome which
may serve as clues to the diagnosis include multiple oral papillomas with a "cobblestone" appearance
on the lips, gingival, and buccal mucosa, acral keratotic papules on the dorsal hands and wrists,
palmoplantar punctate keratoses and multiple facial tricholemmomas. Patients with this condition need
careful malignancy surveillance.
Q/Q(M)-476965 Report a Problem
A 6-year-old boy presents with brachyonychia and three firm subcutaneous nodules with a bluish hue
about the trunk. Biopsy of a representative lesion is consistent with a pilomatricoma. What is the most
likely associated syndrome?
1

Noonan syndrome
2

Carney Complex
3

Nail-Patella syndrome
4

Werner syndrome
106

5

Rubinstein-Taybi syndrome
Q/Q(M)-482926 Report a Problem

A 6-year-old boy presents with brachyonychia and three firm subcutaneous nodules with a bluish hue
about the trunk. Biopsy of a representative lesion is consistent with a pilomatricoma. What is the most
likely associated syndrome?
5

Rubinstein-Taybi syndrome
Rubenstein-Taybe syndrome is caused by a amutation in CREB-Binding protein and presents with
brachyonychia, eruptive keloids, and multiple pilomatricomas. Nail-Patella syndrome is associated with
anonychia, along with hypoplastic patellae, triangular lunulae, radial subluxation, Lester iris, iliac
horns, and glomerulonephritis. The other disorders are not associated with brachyonychia.
Q/Q(M)-482926 Report a Problem
A patient with pseudoherpetic keratitis and a painful PPK would improve with a diet low in:
1

Tyrosine/Phenylalanine
Pseudoherpetic keratitis and a painful PPK describes a patient with Richner-Hanhart syndrome (tyrosenemia
type II). Treatment is with a diet low in tyrosine and phenylalanine. A diet low in glycine, cytosine, biotin or zinc
would not be helpful in this syndrome.
Q/Q(M)-477729 Report a Problem
A patient with Crowes sign and an optic glioma has which of the following disorders?
1

Neurofibromatosis I
The diagnostic criteria for neurofibromatosis I include meeting 2 or more of the following 7 criteria: (1) >5 caf
au lait macules (CALMs) that are >5mm in a prepubertal person or >15 mm in a postpubertal person, (2) >1
neurofibroma or 1 plexiform neurofibroma, (3) axillary/inguinal freckling (Crowes sign), (4) optic glioma, (5)
>1 Lisch nodule (iris hamartoma), (6) sphenoid dysplasia, (7) 1st degree relative with neurofibromatosis I.
Q/Q(M)-474259 Report a Problem
Primary pigmented nodular adrenocortical disease and psammomatous melanotic schwannomas are
characteristic of which of the following syndromes?
2

Carney complex
Carney complex is an autosomal dominant disorder caused by mutations in PRKAR1A (protein kinase A
regulatory subunit 1-alpha). Key features include cardiac, cutaneous and mammary myxomas, pigmented skin
lesions, endocrine abnormalities (pituitary, testicular, thyroid, etc), primary pigmented nodular adrenocortical
disease, and psammomatous melanotic schwannomas.
Q/Q(M)-474256 Report a Problem


Osteopathia striata is seen in which of the following disorders?
107

4

Focal dermal hypoplasia
Focal dermal hypoplasia (Goltz syndrome) is an X-linked dominant disorder that is lethal in males. There is
linear atrophy following Blaschkos lines with areas of fat herniation with underlying osteopathia striata,
which is radiologically characterized by linear bony hyperdensity. Other features include mucocutaneous
papillomas and pits, alopecia, nail dystrophy, tooth abnormalities, and colobomas.
Q/Q(M)-474281 Report a Problem
Meleda is an island off the coast of Croatia. Its inhabitants have an increased frequency of a malodorous
transgradiens palmoplantar keratoderma in a stocking-glove distribution. This condition is autosomal recessive
with a defect in:
1

SLURP1 gene
SLURP1 (Secreted LY6/UPAR-related protein 1) is defective in Mal de Meleda. Keratin 1/9 are defective in Unna-
Vorner/Thost palmoplantar keratoderma, an AD diffuse symmetric non-transgradiens PPK. TOC gene is
defective in Howel-Evans syndrome, an AD PPK associated with focal, pressure-related, non-transgradiens PPK
and esophageal cancer/oral leukoplakia. Loricrin mutations are seen in Vohwinkel syndrome variant and
symmetric progressive erythrokeratodermia. Cathepsin C defects are seen in Haim-Munk syndrome
(PPK+periodontitis+acroosteolysis+onychogryphosis) and Papillon-Lefevre syndrome (sharply demarcated
transgradiens, stocking-glove PPK+periodontitis+dural calcifications and choroids attachments).
Q/Q(M)-477724 Report a Problem
Patients with x-linked icthyosis are more prone to getting which two malignancies?
4

Testicular and ALL
Patients with x-linked icthyosis have a 20% chance of having cryptorchidism and are more prone to both
testicular cancer and acute lymphocytic leukemia.
Q/Q(M)-477498 Report a Problem
Birt-Hogg-Dube syndrome is most strongly associated with which of the following malignancies?
3

Renal cell carcinoma
Birt-Hogg-Dube syndrome is characterized by multiple fibrofolliculomas, trichodiscomas, acro-collagenomas,
lipomas, and oral fibromas. Patients develop renal cell carcinoma, colon cancer, and medullary thyroid
carcinoma.
Q/Q(M)-474287 Report a Problem

What condition is associated with a mutation in CXCR4?
5

WHIM syndrome
Ataxia telangiectasia is associated with mutations in ATM gene. Leukocyte adhesion deficiency is associated
108

with mutations in the common chain of CD18 which is a beta-2 integrin family member, FUCT1, or KINDLIN3.
Chronic granulomatous disease is due to mutations in the NADPH oxidase. SCID is a heterogenous group of
genetic disorders due to mutations in Jak-3, IL-7Ralpha, CD45, CD3delta/CD3episilon, RAG1/RAG2, and Artemis
(DCLREC1C). WHIM syndrome (wars, hypogammaglobulinemia, infections, myelokathexis) is due to a gain-of-
function mutation in CXCR4 which results in the retention of neutrophils in the bone marrow. It is inherited in
an autosomal-dominant fashion.
Q/Q(M)-482555 Report a Problem
Mosaic mutations in PTEN are seen in which of the following conditions?
2

Proteus syndrome
Proteus syndrome is a sporadic condition caused by postzygotic mosaic mutations in PTEN. Clinical features
include subcutaneous lymphovenous malformations, capillary malformations, lipomas, connective tissue nevi
of palms/soles, hemihypertrophy, frontal bossing, hyperostoses of epiphyses & skull (especially external
auditory canal), scoliosis, bilateral ovarian cystadenomas, and parotid monomorphic adenomas.
Q/Q(M)-474261 Report a Problem
A deficiency in sialophorin, a surface glycoprotein, is thought to play a role in which genetic disorder?
1

Wiskott-aldrich syndrome
Wiskott-Aldrich syndrome is an x-liked recessive disorder localized to Xp11.3. The gene involved codes for WAS,
and the protein product has been implicated in lymphocyte and megakaryocyte signal transduction.
Q/Q(M)-474035 Report a Problem
Which disease can clinically mimic pellagra but is inherited in an autosomal recessive fashion and is due to a
defect in the transport of neutral amino acids?
3

Hartnup Disease
The clinical manifestation of Hartnup disease is similar to that of pellagra because the resultant defect in the
transport of amino acids leads to low levels of tryptophan. Since tryptophan is required to make nicotinic acid,
pts with Hartnup disease manifest the same symptoms as niacin-deficient patients (pellagra).
Q/Q(M)-474036 Report a Problem
A 15 year old male patient is seen for evaluation of a congenital nevus. As a part of the review of systems, it is
discovered that he has recurrent joint dislocations. Multiple family members have a similar problem. What is
the defective gene in Type 3 Ehlers-Danlos syndrome?
1

Collagen 3
Type 3 Ehlers-Danlos syndrome, or benign hypermobile Ehlers-Danlos syndrome, is a relatively mild form of
EDS with recurrent joint dislocations. The syndrome is a result of defective collagen 3. Defective collagen 5 is
seen in EDS types 1 and 2 (gravis and mitis, respectively). Lysyl oxidase deficiency is seen in Type 5 (x-linked).
Collagen 2 mutations can lead to Type 7 (arthrochalasis multiplex congenita). The PLOD gene is involved in type
109

6 (ocular-scoliotic).
Q/Q(M)-482699 Report a Problem
A 32 year-old woman is 5 weeks pregnant and is diagnosed with hyperthyroidism. Her doctor gives her a
prescription for on methimazole 10 mg PO tid. Which of the following fetal abnormalities could be caused by
this exposure?
1

Aplasia cutis congenita
In-utero methimazole exposures has been linked to aplasia cutis congenita and should not be used in pregnant
women. The FDA pregnancy class is D. The other listed options are not linked with maternal methimazole
usage.
Q/Q(M)-477942 Report a Problem
Which type of epidermolysis bullosa simplex is associated with early death?
3

Dowling-Maera
The Dowling-Maera variant of epidermolysis bullosa simplex is associated with widespread bullae, significant
mucous membrane and laryngeal/esophageal involvement, nail dystrophy, and early death.
Q/Q(M)-474239 Report a Problem
A patient has recurrent infections and is found to have a disorder of phagocytic cells from an inability of
phagocytes to undergo the respiratory burst needed to kill certain types of bacteria and fungi. Carriers of this
disease are also at risk for which condition?
3

Discoid lupus
This patient has chronic granulomatous disease characterized by dysfunctional phagocytic cells. The most
common molecular defect in chronic granulomatous disease is a mutation in the gene encoding the b subunit
of cytochrome b. Carriers have a significant incidence of discoid lupus erythematosus, photosensitivity,
Raynaud's phenomenon, and aphthous ulcers. Carriers of ataxia telangiectasia are at risk for breast cancer.
Q/Q(M)-482308 Report a Problem
A patient with multiple facial trichilemmomas is at risk of which of the following cancers?
2

Breast carcinoma
Cowden syndrome (multiple hamartoma syndrome) is an autosomal dominant disorder caused by mutations in
PTEN, a phosphatase that dephosphorylates tyrosine, serine, and threonine. Clinically, there are numerous
facial trichilemmomas, oral papillomas, acral keratotic papules, sclerotic fibromas, breast fibroadenomas and
adenocarcinomas, thyroid adenomas and adenocarcinomas, and hamartomatous polyps of the gastrointestinal
tract.
Q/Q(M)-474290 Report a Problem
110

Which vascular disorder is characterized by facial vascular malformation and ipsilateral intracranial and retinal
arteriovenous malformations(AVMs)?
4

Bonnet Dechaune Blanc syndrome
Bonnet Bechaune Blanc syndome, also know as Wyburn-Mason syndrome, is characterized by a facial vascular
malformation and ipsilateral intracranial and retinal AVMs. Encephalotrigeminal angiomatosis is another name
for Sturge-Weber. Von Lohuizen's disease is another name for cutis marmorata telangiectatica congenita.
Q/Q(M)-477510 Report a Problem
Which of the following syndromes are linked to a PTEN gene mutation?
4

All of these options are correct
PTEN gene produces a phosphatase the regulates the cell-cycle and apoptosis, therefore acting as a tumor
suppressor gene. Tissues affected by this mutation are those with increased proliferation such as epidermis,
the oral and gastrointestinal mucosa, and the thyroid and breast epithelium. All of the syndromes listed have
mutations in PTEN.
Q/Q(M)-477783 Report a Problem

A patient presents with bilateral retinal hemangioblastomas and a capillary malformation on her neck. Which
gene mutation is most likely?
1

VHL
These findings are most characteristic of Von Hippel-Lindau syndrome. VHL is a tumor suppressor gene which is
mutated in this syndrome. Other findings include: renal and pancreatic cysts/carcinoma, pheochromocytoma,
and cerebellar/other CNS hemangioblastomas. Endoglin is defective in Osler-Weber-Rendu, PTH/PTHrP type I
receptor in Maffucci syndrome, VEGF receptor-3 in Nonne-Milroy disease (hereditary lymphedema) and MFH1
in lymphedema and ptosis syndrome.
Q/Q(M)-477782 Report a Problem

Which of the following genodermatoses has premalignant oral leukoplakia as a feature?
2

Dyskeratosis congenita
Dyskeratosis congenita has premalignant oral leukoplakia in addition to cutaneous poikiloderma and nail
dystrophy. Pachyonychia congenita also has leukoplakia, but it is benign in nature. Additional features of
pachyonychia congenita are thickened nails with nail bed hyperkeratosis and palmoplantar keratoderma.
Keratosis follicularis, also known as Darier\'s disease, is characterized by benign whitish papules on the oral
mucosa, cutaneous keratotic papules in seborrheic distribution, and longitudinal erythronychia with V-nicking
of the distal nail plate. Papillon-Lefevre has gingivitis, without oral leukoplakia, and acral hyperkeratosis.
Anhidrotic ectodermal dysplasia does not have leukoplakia. It consists of anhidrosis or hypohidrosis,
111

hypotrichosis, and anodontia.
Q/Q(M)-482679 Report a Problem
What is the gene mutation that most commonly causes herlitz junctional epidermolysis bullosa?
1

LAMB3
LAMB3 is the most common mutation, it is usually a nonsense mutation causing a premature stop codon
resulting in the absence of laminin 332. Non herlitz type is due to a missense or splice site mutation resulting in
rudimentary/decreased laminin 332. COL7A1 is mutated in dystrophic epidermolysis bullosa. NEMO gene is
mutated in incontinnentia pigmenti. ATP2A2 gene is mutated in Dariers. ATP2C1 is mutated in Hailey Hailey.
Q/Q(M)-482759 Report a Problem

What is the gene defect in this condition, which is also called Mendes da Costa syndrome?
5

Connexin 31
Mendes da COsta syndrome is also called Erythrokeratoderma Variabilis. It is an autosomal dominantly
inherited due to a mutation in connexin 31 or connexin 30.3. It is characterized by transient geographic patches
of erythema and hyperkeratotic plaques.
Q/Q(M)-476803 Report a Problem


Urticaria pigmentosa is linked to a defect in the c-kit protooncogene. What autosomal dominant skin disease
also has been linked to this defect?
1

Piebaldism
2

Hypomelanosis of Ito
3

Waardenburg syndrome
4

Hermansky-Pudlak syndrome
112

5

Incontinentia pigmenti
Q/Q(M)-477732 Report a Problem
A 4 year-old boy presents with 2 soft, dark-blue, compressible nodules on his extremities. His mother has noted
that these lesions have increased sweating and that they were present at birth. No one else in the family has
had similar skin lesions. What step is indicated first to help determine the diagnosis?
1

Stool guiac
2

MRI of the abdomen
3

CBC
4

Immediate referral to a gastroenterologist
5

Biopsy of a skin lesion
Q/Q(M)-477804 Report a Problem

A 4 year-old boy presents with 2 soft, dark-blue, compressible nodules on his extremities. His mother has noted
that these lesions have increased sweating and that they were present at birth. No one else in the family has
had similar skin lesions. What step is indicated first to help determine the diagnosis?
1

Stool guiac
Blue rubber bleb nevus syndrome is described above. There are multiple venous malformations on the
extremities and trunk, often present at birth to early childhood. The number of these lesions increase with age.
The lesions may have increased sweating and can be combined with lymphatic-venous malformations. Skin
lesions can be a clue to gastrointestinal venous malformations which can lead to secondary bleeding and
anemia. The most reasonable screening test to determine if the patient has GI hemorrhage is a stool guiac. An
MRI or complete blood count can be helpful, but are not the best test to start with. A skin biopsy is not
indicated. If there is GI blood loss, evaluation by a gastroenterologist is useful.
Q/Q(M)-477804 Report a Problem
Patients with Russell-Silver syndrome exhibit:
1

Clinodactyly of fifth finger
2

Peg teeth
3

Osteopathia striata
4

Broad thumbs
5

Shortened 4th and 5th metacarpals
113

Q/Q(M)-474015 Report a Problem

Patients with Russell-Silver syndrome exhibit:
1

Clinodactyly of fifth finger
Characteristic features of Russell-Silver include short stature, bony asymmetry, triangular facies, clinodactyly of
fifth finger, and precocious sexual development with cryptochordism/hypospadias. Broad thumbs are seen in
Rubinstein-Taybi and shortened fourth and fifth metacarpals are seen in Turner syndrome. Osteopathia striata
is characteristic of focal dermal hypoplasia.
Q/Q(M)-474015 Report a Problem

Which of the following is not a feature of Cockayne syndrome?
1

Dwarfism
2

Salt and pepper retina
3

Increased sister chromatid exchange
4

Increased risk of skin cancer
5

Poikiloderma
Q/Q(M)-482257 Report a Problem

Which of the following is not a feature of Cockayne syndrome?
4

Increased risk of skin cancer
Patients with Cockayne syndrome do not have an increased risk of skin cancer. All the other findings are
associated with Cockayne syndrome, as well as photosensitivity, deafness, cataracts, and sunken
facies. Increased chromatid exchange is also seen in Bloom syndrome, dyskeratosis congenita, and Fanconi's
anemia.
Q/Q(M)-482257 Report a Problem
Palmoplantar keratoderma with deafness is caused by a defect in which gene?
1

SLURP-1
2

Plakophilin
3

Mitochondrial serine transferase RNA
114

4

Lysosomal papain like cysteine proteinase
5

Unknown
Q/Q(M)-477435 Report a Problem
Palmoplantar keratoderma with deafness is caused by a defect in which gene?
3

Mitochondrial serine transferase RNA
Palmoplantar keratoderma with deafness is caused by a defect in mitochondrial serine transferase RNA. A
defect in SLURP-1 causes Mal de Meleda. A defect in plakophilin causes ectodermal dysplasia with skin fragility.
A defect in cathepsin C lysososomal papain like cysteine proteinase causes Papillon LeFevre and Haim Munk.
Q/Q(M)-477435 Report a Problem
What is the most common genetic defect associated with this syndrome?
1

Neurofibromin
2

Merlin
3

Tuberin
4

Hamartin
5

Folliculin
Q/Q(M)-482072 Report a Problem


What is the most common genetic defect associated with this syndrome?
1

Neurofibromin
Neurofibromatosis I occur due to a microdeletion at 17q11.2 involving the NF1 gene, which encodes for
neurofibromin. It is an autosomal dominant disorder characterized by numerous benign tumors
(neurofibromas) of the peripheral nervous system, caf au lait macules, freckling in the area of the armpit
(crows sign), two or more growths on the iris of the eye (known as Lisch nodules or iris hamartomas), tumor on
the optic nerve (optic glioma), abnormal development of the spine (scoliosis), the temple (sphenoid) bone of
115

the skull, or the tibia (one of the long bones of the shin) and a first degree relative (parent, sibling, or child)
with NF1. The other proteins in the list are associated with other syndromes: In neurofibromatosis type 2, a
NF2 gene mutation has been identified which encodes for a protein called Merlin, in tuberous sclerosis two
genetic mutations have been identified on two separate chromosomes namely tuberin and hamartin, and a
Folliculin mutation is seen in Birt Hogg Dube syndrome.
Q/Q(M)-482072 Report a Problem
Epidermal nevus syndromes inheritance pattern is:
1

Sporadic
2

X-linked recessive
3

X-linked dominant
4

Autosomal recessive
5

Autosomal dominant
Q/Q(M)-477730 Report a Problem
Epidermal nevus syndromes inheritance pattern is:
1

Sporadic
Epidermal nevus syndrome has many findings, including: sporadic inheritance, nevus unius lateris, capillary
malformations, caf au lait macules, mantal retardation and seizures, deafness, hemiparesis, hemihypertrophy
of limbs, kyphoscoliosis and rare solid tumors. A biopsy is helpful to rule out epidermolytic hyperkeratosis. If
positive, the patients offspring are at risk for generalized epidermolytic hyperkeratosis.
Q/Q(M)-477730 Report a Problem

You receive a hospital consult from the gastroenterology service for a 42-year old woman with esophageal
cancer. They would like your opinion on the yellow, thickened areas on her palms and soles in areas of
pressure. When you speak with her, she says that her father had similar problems and it runs in her family.
Which of the following is defective?
1

TOC gene
2

Desmoplakin
3

Plakoglobin
4

Connexin 30.3
5

Connexin 31
116

Q/Q(M)-477728 Report a Problem
You receive a hospital consult from the gastroenterology service for a 42-year old woman with esophageal
cancer. They would like your opinion on the yellow, thickened areas on her palms and soles in areas of
pressure. When you speak with her, she says that her father had similar problems and it runs in her family.
Which of the following is defective?
1

TOC gene
This case describes Howell-Evans syndrome. This AD syndrome characteristically has a PPK in areas of pressure,
oral leukoplakia and esophageal carcinomas. Desmoplakin is defective in Carvajal syndrome and plakoglobin in
Naxos syndrome. Connexin 31 and 30.3 are linked to erythrokeratoderma variabilis, which includes a PPK, but
not esophageal carcinoma.
Q/Q(M)-477728 Report a Problem
Ataxia- Telangiectasia
1

Is associated with significantly increased risk of ovarian cancer in carriers
2

Telangiectasias present in late adulthood
3

Carriers do not have an increased risk of malignancy
4

Is inherited in XLR (X-linked recessive)
5

First clinical sign is neurologic
Q/Q(M)-482868 Report a Problem
Ataxia- Telangiectasia
5

First clinical sign is neurologic
First sign is ataxia. Telangiectasias occur later in puberty. Carriers show an increased risk of breast cancer.
Q/Q(M)-482868 Report a Problem
In a patient suspected of having multiple endocrine neoplasia type IIb, which lab test would be appropriate?
1

Calcitonin
2

Glucagon
3

Parathyroid hormone
4

Calcium
5

Cortisol
117

Q/Q(M)-476960 Report a Problem
In a patient suspected of having multiple endocrine neoplasia type IIb, which lab test would be appropriate?
1

Calcitonin
Multiple endocrine neoplasia (MEN) syndrome type Iib, also called multiple mucosal neuroma syndrome is an
autosomal dominant condition due to a defect in the RET protooncogene on chromosome 10q11.2. This rare
condition is associated with mucosal neuromas on the tongue and lips, medullary thyroid carcinoma,
pheochromocytoma, and gastrointestinal ganglioneuromatosis. In addition, patients can also present with a
marfanoid habitus and facial dysmorphism. Mucosal neuromas can be a dermatologic clue to the underlying
diagnosis as these lesions appear during early childhood and present as pink, pedunculated nodules. As the
major cause of mortality in these patients is medullary thyroid cancer, which nearly all patients will have by
early adulthood, aggressive screening, with serial calcitonin level, and prophylactic thyroidectomy are
warranted.
Q/Q(M)-476960 Report a Problem

Lamellar ichthyosis is caused by mutations in which of the following genes?
1

Steroid sulfatase
2

Fatty aldehyde oxidoreductase
3

Phytanoyl coenzyme A hydroxylase deficiency
4

Arylsulfatase E
5

Transglutaminase
Q/Q(M)-474243 Report a Problem

Lamellar ichthyosis is caused by mutations in which of the following genes?
5

Transglutaminase
Nonbullous congenital ichthyosiform erythroderma and lamellar ichthyosis are caused by mutations in the
transglutaminase gene.
Q/Q(M)-474243 Report a Problem

The development of which malignancy is most commonly associated with lymphomatoid papulosis?
1

Non-Hodgkin's lymphoma
118

2

Mycosis fungoides
3

Multiple myeloma
4

Immunoblastic lymphoma
5

Waldenstrom's macroglobulinemia
Q/Q(M)-475877 Report a Problem

The development of which malignancy is most commonly associated with lymphomatoid papulosis?
2

Mycosis fungoides
Lymphomatoid papulosis is a recurrant eruption of unclear etiology characterized by the appearance of red-
brown papules and nodules which spontaneously disappear in 3 to 8 weeks. It is notable for histologic features
which suggest a CD30 positive malignant lymphoma. There is controversy regarding whether lymphomatoid
papulosis (LyP) is a malignant, premalignant or benign condition. The disease may last from months to years
and in up to 20% of patients, it may be preceded by, followed by, or associated with another type of cutaneous
malignancy. Generally, this is mycosis fungoides, a CD30-positive large T-cell lymphoma or Hodgkin's disease.
Because of this potential risk, long-term follow-up of these patients is required.
Q/Q(M)-475877 Report a Problem
Painful crises and 'whorled' corneal opacities are seen with which of the following enzyme abnormalities?
1

Homogentisic acid oxidase
2

Alpha-galactosidase A
3

Glucocerebrosidase
4

Iduronate sulfatase
5

Glucoronidase
Q/Q(M)-478044 Report a Problem
Painful crises and 'whorled' corneal opacities are seen with which of the following enzyme abnormalities?
2

Alpha-galactosidase A
Painful crises and whorled corneal opacities are found in Fabry disease which is caused by a defect in alpha-
galactosidase A. The remaining conditions do not have these findings.
Q/Q(M)-478044 Report a Problem
119

Mucosal neuromas, pheochromocytoma and medullary thyroid carcinoma in a patient with a marfanoid body
habitus is associated with which of the following gene defects?
1

Menin
2

RET proto-oncogene
3

PTEN
4

BHD
5

STK11
Q/Q(M)-478008 Report a Problem
Mucosal neuromas, pheochromocytoma and medullary thyroid carcinoma in a patient with a marfanoid body
habitus is associated with which of the following gene defects?
2

RET proto-oncogene
The RET proto-oncogene is mutated in Multiple Endocrine Neoplasia type II. Type IIb is described above. Other
findings include rare parathyroid abnormalities, megacolon, thickened lips and thick, everted upper eyelids.
Menin is associated with MEN type I, PTEN with Cowden disease, BHD with Birt-Hogg-Dube syndrome and
STK11 with Peutz-Jeghers syndrome.
Q/Q(M)-478008 Report a Problem
You are examining a child with mild albinism, immunodeficiency and silver grey highlights in his hair. You
diagnose the child with Chediak-Higashi syndrome. Why are you confident that this isnt Griscelli syndrome?
1

Giant lysosomal granules are present in neutrophils in the blood smear
2

Griscelli syndrome does not have albinism as a feature
3

Griscelli syndrome has no changes in hair color
4

All of these answers are correct
5

None of these answers are correct
Q/Q(M)-477734 Report a Problem
You are examining a child with mild albinism, immunodeficiency and silver grey highlights in his hair. You
diagnose the child with Chediak-Higashi syndrome. Why are you confident that this isnt Griscelli syndrome?
1

Giant lysosomal granules are present in neutrophils in the blood smear
Chediak-Higashi syndrome and Griscelli syndrome have similar features including silver-grey highlights of hair,
immunodeficiency, mild albinism and an accelerated phase of disease. Examining a peripheral blood smear is
120

helpful in distinguishing between these two syndromes. Patients with the LYST defect (a lysosomal storage
transport gene) have Chediak-Higashi syndrome and will have giant lysosomal granules visible in white blood
cells on a blood smear.
Q/Q(M)-477734 Report a Problem
On cutaneous exam, angiokeratoma corporis diffusum is characteristic of which of the following conditions?
1

Sialodosis
2

Fucosidosis
3

Fabry disease
4

All of these options are correct
5

None of these options are correct
Q/Q(M)-478043 Report a Problem
On cutaneous exam, angiokeratoma corporis diffusum is characteristic of which of the following conditions?
4

All of these options are correct
Findings of angiokeratoma corporis diffusum are found in all three listed conditions. They cannot by
distinguished by skin exam.
Q/Q(M)-478043 Report a Problem
Adenosine deaminase deficiency is seen in which immunodeficient disease?
1

Wiskott-Aldrich syndrome
2

Chronic granulomatous disease
3

Job syndrome
4

Severe combined immunodeficiency syndrome
5

Leineri's disease
Q/Q(M)-477185 Report a Problem

Adenosine deaminase deficiency is seen in which immunodeficient disease?
4

Severe combined immunodeficiency syndrome
Severe combined immunodeficiency is a heterogeneous group of disorders characterized by decreased
121

humoral and cell mediated immunity. Patients may have recurrent infections including cutaneous ones, GVHD
(due to in utero cmaternal lymphocytes), sepsis, oral candidiasis, and diarrhea. Implicated genes include the IL-
2 receptor (x-linked recessive form) and adenosine deaminase deficiency (autosomal recessive form).
Q/Q(M)-477185 Report a Problem
Deficiency of filaggrin is the strongest known predisposing genetic factor for the development of atopic
dermatitis. What is the mode of inheritance of mutations in filaggrin?
1

Autosomal dominant
2

Autosomal semidominant
3

Autosomal recessive
4

X-linked dominant
5

X-linked recessive
Q/Q(M)-482537 Report a Problem
Deficiency of filaggrin is the strongest known predisposing genetic factor for the development of atopic
dermatitis. What is the mode of inheritance of mutations in filaggrin?
2

Autosomal semidominant
Autosomal dominant means that a mutation in a gene on one of the two chromosomes in any autosomal
chromosome pair results in disease. Autosomal recessive means that mutations in the same gene on both of
the chromosomes in the pair must occur for the disease to occur. Autosomal semidominant means that a
mutation in one gene of the autosomal chromosome pair results in a mild version of the disease, while
mutations in the gene on both chromosomes results in the full-blown disease. Ichthyosis vulgaris is caused by
mutations in filaggrin and inherited in a autosomal semidominant manner. This is the genetic explanation for
why the phenotype of ichthyosis vulgaris can vary significantly across individuals.
Q/Q(M)-482537 Report a Problem
A 20-year old male with a history of pheochromocytoma and medullary thyroid cancer presents with mucosal
papules. His overall body appearance is most likely to demonstrate:
1

Cushingoid features
2

Marfanoid features
3

Short stature
4

Lipodystrophy
5

Unilateral limb shortening
122

Q/Q(M)-482248 Report a Problem
A 20-year old male with a history of pheochromocytoma and medullary thyroid cancer presents with mucosal
papules. His overall body appearance is most likely to demonstrate:
2

Marfanoid features
This patient has multiple endocrine neoplasia type IIb (MEN IIb) characterized by pheochromocytoma, thyroid
cancer, and rare parathyroid carcinoma as well as mucosal neuromas cutaneously. These patients have a
marfanoid body habitus. This syndrome is caused by the RET proto-oncogene.
Q/Q(M)-482248 Report a Problem
Which of the following is a feature of Neurofibromatosis type II?
1

Congenital hypertrophy of the retinal pigment epithelium
2

Lisch nodules
3

Juvenile posterior subcapsular lenticular opacities
4

Lester iris
5

Optic gliomas
Q/Q(M)-474258 Report a Problem
Which of the following is a feature of Neurofibromatosis type II?
3

Juvenile posterior subcapsular lenticular opacities
Neurofibromatosis type II is an autosomal dominant disorder caused by mutations in schwannomin/merlin.
Clinical features include cutaneous schwannomas and neurofibromas, bilateral vestibular schwannomas, and
juvenile posterior subcapsular lenticular opacities.
Q/Q(M)-474258 Report a Problem
Dystrophic epidermolysis bullosa is associated with mutations in collagen VII. Trauma or friction induced
blistering in these patients have a plane a splitting in the:
1

Sublamina densa
2

Stratum spinosum
3

Lamina lucida
4

Stratum basale
5

None of these answers are correct
123

Q/Q(M)-477714 Report a Problem

Dystrophic epidermolysis bullosa is associated with mutations in collagen VII. Trauma or friction induced
blistering in these patients have a plane a splitting in the:
1

Sublamina densa
The split in dystrophic epidermolysis bullosa is found in the sublamina densa, where the collagen VII anchors
the epidermis to the anchoring plaques in the dermis. The remaining options are incorrect.
Q/Q(M)-477714 Report a Problem

Which of the following elastic tissue diseases demonstrates calcified elastic fibers?
1

Cutis laxa
2

Marfan syndrome
3

Anetoderma
4

Pseudoxanthoma elasticum
5

Buschke-ollendorf syndrome
Q/Q(M)-477341 Report a Problem

Which of the following elastic tissue diseases demonstrates calcified elastic fibers?
4

Pseudoxanthoma elasticum
Pseudoxanthoma elasticum is usually an autosomally recessive inherited condition due to a defective transport
protein, ABCC6. The clinical manifestations of the disease arise from fragmented and calcified fibers of the skin,
eyes and arteries. Patients may have yellow papules, loose redundant skin, angioid streaks and hemorrhage.
Histologically, the hallmark of pseudoxanthoma elasticum is calcified elastic fibers.
Q/Q(M)-477341 Report a Problem
A 5 month old girl presents with failure to thrive. She has had life-long atopic dermatitis treated with topical
hydrocortisone cream and has persisent hypernatremia. On your exam, she has generalized erythema and
scaling of her body and trichorrhexis invaginata on examination of hairs from her eyebrows. Which syndrome is
she most likely to have?
1

Leiner syndrome
2

Omenn Syndrome
124

3

Netherton Syndrome
4

Wiskott-Aldrich Syndrome
5

Severe atopic dermatitis
Q/Q(M)-477737 Report a Problem

A 5 month old girl presents with failure to thrive. She has had life-long atopic dermatitis treated with topical
hydrocortisone cream and has persisent hypernatremia. On your exam, she has generalized erythema and
scaling of her body and trichorrhexis invaginata on examination of hairs from her eyebrows. Which syndrome is
she most likely to have?
3

Netherton Syndrome
Netherton syndrome is caused by a mutation in the SPINK5 gene, encoding LEKT1. This is a serine protease
inhibitor which is important in downregulating inflammation. Early presentation is with failure to thrive,
generalized erythema/scale, hypernatremia, and sparse hair with the characteristic finding of trichorrhexis
invaginata. Pili torti and trichorrhexis nodosa also can be seen. Eyebrow hair is most commonly affected.
Omenn syndrome is an autosomal recessive form of severe combined immunodeficiency (SCID) with findings of
failure to thrive, erythroderma, scaling, chronic diarrhea, lymphadenopathy, and hepatosplenomegaly. Leiner
syndrome can present with failure to thrive, immunodeficiency and seborrheic dermatitis. Wiskott-Aldrich
syndrome is an x-linked recessive condition with mutations of the WAS gene. Presentation includes atopic
dermatitis, thrombocytopenia, recurrent bacterial infection, lymphoreticular malignancy with non-Hodgkin's
lymphoma being the most common, and increased IgA, D and E. With the characteristic hair changes, atopic
dermatitis alone is not the most likely diagnosis.
Q/Q(M)-477737 Report a Problem

What is the inheritance pattern of dermatosis with acantholysis?
1

Autosomal dominant
2

Autosomal recessive
3

X-linked dominant
4

X-linked recessive
5

Sporadic
Q/Q(M)-476804 Report a Problem

What is the inheritance pattern of dermatosis with acantholysis?
125

1

Autosomal dominant
Hailey-Hailey, or Familial Benign Pemphigus, is an autosomal dominant genodermatosis which is caused by a
mutation in ATP2C1. Vesicles and erythematous plaques develop in the skin folds such as axillae and groin area.
Q/Q(M)-476804 Report a Problem

What phenotype results from a low activity of double stranded RNA adenosine deaminase?
1

Waardenberg's syndrome type 2
2

Piebaldism
3

Tietz syndrome
4

dyschromatosis symmetrica hereditaria
5

oculocutaneous albinism type 4
Q/Q(M)-477436 Report a Problem

What phenotype results from a low activity of double stranded RNA adenosine deaminase?
4

dyschromatosis symmetrica hereditaria
Dyschromatosis symmetrica hereditaria (or acropigmentation symmetrica of Dohi) is an autosomal dominant
disease with hypo and hyper pigmented macules and patches on the dorsal hands and feet associated with a
low activity of double stranded RNA adenosine deaminase.
Q/Q(M)-477436 Report a Problem
A child has ichthyosis and is found to have leukocytes with vacuoles filled with lipids on peripheral smear. She
most likely has:
1

Chanarin-Dorfman syndrome
2

Ichthyosis hystrix
3

Naxos disease
4

Ichthyosis bullosa of Siemens
5

Ichthyosis vulgaris
Q/Q(M)-482304 Report a Problem
126


A child has ichthyosis and is found to have leukocytes with vacuoles filled with lipids on peripheral smear. She
most likely has:
1

Chanarin-Dorfman syndrome
This child has Chanarin-Dorfman syndrome, also called neutral lipid storage disease. This is an autosomal
recessive disorder characterized by accumulation of triglycerides in the cytoplasm of leukocytes, muscle, liver,
fibroblasts, and other tissues. Patients generally have normal blood lipid levels and a finely scaling ichthyosis.
They can also have extracutaneous involvement such as cataracts, decreased hearing, myopathy, and
neurologic abnormalities.
Q/Q(M)-482304 Report a Problem

Epidermolysis bullosa with muscular dystrophy is caused by mutations in which of the following?
1

Keratins 5 and 14
2

Plectin
3

Loricrin
4

Collagen 7
5

Collagen 17
Q/Q(M)-474240 Report a Problem

Epidermolysis bullosa with muscular dystrophy is caused by mutations in which of the following?
2

Plectin
Epidermolysis bullosa with muscular dystrophy is caused by mutations in plectin.
Q/Q(M)-474240 Report a Problem

Which of the following syndromes is X-linked dominant?
1

Anhidrotic ectodermal dysplasia
2

Dyskeratosis congenita
3

X-linked icthyosis
4

Orofaciodigital syndrome 1
127

5

Menkes kinky hair syndrome
Q/Q(M)-477497 Report a Problem

Which of the following syndromes is X-linked dominant?
4

Orofaciodigital syndrome 1
Orofaciodigital sydrome 1 is an X-linked dominantly inherited disorder caused by a defect in the CXORF5 gene.
The rest of the above conditions are inherited in an x-linked recessive pattern.
Q/Q(M)-477497 Report a Problem
Which of the following subtypes of Ehlers-Danlos Syndrome (EDS) is associated with early demise?
1

Type 1 (Gravis)
2

Type 4 (Vascular)
3

Type 5 (X-linked)
4

Type 7 (Arthrochalasis multiplex congenita)
5

Type 10 (Fibronectin)
Q/Q(M)-474275 Report a Problem

Which of the following subtypes of Ehlers-Danlos Syndrome (EDS) is associated with early demise?
2

Type 4 (Vascular)
Ehlers-Danlos Syndrome type 4 (Vascular) is caused by mutations in collagen 3 in some cases. There is a
tendency to develop arterial and visceral rupture resulting in early death.
Q/Q(M)-474275 Report a Problem

A 20-year-old woman presents with hypodontia, sparse hair, palmoplantar hyperkeratosis, and nail dystrophy.
Examination of her eyelids reveal multiple, translucent-appearing papules. The most likely gene defect is:
1

WNT10A
2

PTCH
3

CYLD
4

BRAF
128

5

PTEN
Q/Q(M)-482824 Report a Problem

A 20-year-old woman presents with hypodontia, sparse hair, palmoplantar hyperkeratosis, and nail dystrophy.
Examination of her eyelids reveal multiple, translucent-appearing papules. The most likely gene defect is:
1

WNT10A
Schopf-Schulz-Passarge syndrome is characterized by multiple apocrine hydrocystomas (most commonly
appearing on the eyelids) and syringofibroadenomas, in addition to hypodontia, hypotrichosis,
onychodystrophy, and palmoplantar keratoderma. It is inherited as an autosomal recessive condition and is
caused by mutations in WNT10A, which encodes a cutaneous signaling molecule involved in ectodermal
appendageal development.
Q/Q(M)-482824 Report a Problem


Patients with progeria typically die of which of the following conditions?
1

Infection
2

Metastatic carcinoma
3

Atherosclerotic heart disease
4

Nail atrophy
5

Progressive systemic sclerosis
Q/Q(M)-474283 Report a Problem

Patients with progeria typically die of which of the following conditions?
3

Atherosclerotic heart disease
Progeria (Hutchinson-Gilford syndrome) is a sporadic condition characterized by lipoatrophy, sclerodermoid
skin, alopecia, nail atrophy, craniomegaly with small face, muscle/bone wasting, and severe premature
atherosclerosis resulting in early death.
Q/Q(M)-474283 Report a Problem

Retention of primary teeth a dental finding of which of the following conditions?
129

1

Hypomelanosis of Ito
2

Letterer-Siwe disease
3

Tuberous sclerosis
4

Jackson Sertoli syndrome
5

Hyper-IgE syndrome
Q/Q(M)-477740 Report a Problem
Retention of primary teeth a dental finding of which of the following conditions?
5

Hyper-IgE syndrome
Hyper-Immunoglobulin E syndrome is an autosomal dominant condition with impaired regulation of IgE
function and deficient neutrophil chemotaxis. There is increased susceptibilty to infections and increased IgE
serum levels. Retained primary teeth and lack of development of secondary teeth are characteristic findings.
The remaining conditions do not have this as a prominent finding.
Q/Q(M)-477740 Report a Problem
Which of the following is caused by a defect in keratins 4 & 13?
1

White sponge nevus
2

Epidermolysis bullosa simplex
3

Epidermolysis bullosa simplex with myotonic dystrophy
4

Junctional EB with pyloric atresia
5

Cloustons syndrome
Q/Q(M)-474023 Report a Problem

Which of the following is caused by a defect in keratins 4 & 13?
1

White sponge nevus
White sponge nevus is caused by a defect in keratins 4 & 13. The remaining entities have the corresponding
defects: EB simplexkeratins 5 & 14 EB simplex with myotonic dystrophyplectin Junctional EB with pyloric
atresiaIntegrin ?-6, ?4 Cloustons syndrome (hidrotic ectodermal dysplasia)connexin 30.
Q/Q(M)-474023 Report a Problem
Which of the following is NOT a characteristic skin finding in patients with Down Syndrome?
130

1

Syringomas
2

Elastosis perforans serpiginosa
3

Single palmar crease
4

Flat nipples
5

Small tongue
Q/Q(M)-474053 Report a Problem

Which of the following is NOT a characteristic skin finding in patients with Down Syndrome?
5

Small tongue
Down syndrome is caused by nondisjunction and results in trisomy 21. Clinical features include single palmar
crease, flat nipples, increased nuchal skin folds, syringomas, elastosis perforans serpiginosa, xerosis, epicanthic
folds of eyes, protruding scrotal tongue and fissured thickened lips.
Q/Q(M)-474053 Report a Problem
Spastic ditetraplegia is associated with which of the following disorders?
1

Sjogren-Larsson syndrome
2

X-linked ichthyosis
3

Lamellar ichthyosis
4

KID syndrome
5

Refsum syndrome
Q/Q(M)-474244 Report a Problem
Spastic ditetraplegia is associated with which of the following disorders?
1

Sjogren-Larsson syndrome
Sjogren-Larsson syndrome is an autosomal recessive disorder caused by mutations in the fatty aldehyde
oxidoreductase/alcohol dehydrogenase gene. This disorder is characterized by ichthyosis, spastic ditetraplegia,
mental retardation, epilepsy, glistening dot retinal pigmentation, and dental enamel dysplasia.
Q/Q(M)-474244 Report a Problem

Which of the following is correct about nevoid basal cell carcinoma syndrome?
131

1

known as Gardner syndrome
2

autosomal recessive
3

multiple keratoacanthomas
4

calicification of the falx
5

odontogenic steatomas
Q/Q(M)-482481 Report a Problem

Which of the following is correct about nevoid basal cell carcinoma syndrome?
4

calicification of the falx
Nevoid basal cell carcinoma syndrome, also known as Gorlin syndrome, is an autosomal dominant disease, with
abnormalities in the PTCH gene. It presents with multiple basal cell cancers early in life, frontal bossing,
hypertelorism, odontogenic keratocysts, and bifid ribs. The presence of intracranial calcification of the falx
cerebri is characteristic.
Q/Q(M)-482481 Report a Problem
Eyelid string of pearls are seen in which of the following conditions?
1

Focal dermal hypoplasia
2

Lipoid proteinosis
3

Hutchinson-Gilford syndrome
4

Beare-Stevenson cutis gyrata syndrome
5

Gauchers disease
Q/Q(M)-474282 Report a Problem
Eyelid string of pearls are seen in which of the following conditions?
2

Lipoid proteinosis
Lipoid proteinosis is an autosomal recessive condition characterized by yellow papules on the face and
oropharynx, eyelid string of pearls, hoarse voice, verrucous nodules of elbows and knees, and bean-shaped
temporal and hippocampal calcification with occasional seizures. Histologically, there are PAS+ deposits in the
affected tissue.
Q/Q(M)-474282 Report a Problem
132



The most common ocular association with cutis marmorata telangiectatica congenital is:
1

Cataracts
2

Glaucoma
3

Retinoblastoma
4

Corneal opacity
5

Angioid streaks
Q/Q(M)-474058 Report a Problem
The most common ocular association with cutis marmorata telangiectatica congenital is:
2

Glaucoma
Glaucoma is the most common associated eye finding in CMTC patients. Glaucoma is also seen in patients with
neurofibromatosis type 1 and Sturge Weber patients.
Q/Q(M)-474058 Report a Problem
A child presenting with the scalp findings shown and a right arm hypoplasia would be diagnosed with which of
the following syndromes?
1

Adams-Oliver syndrome
2

Bart's syndrome
3

Progeria
4

Dunnigan syndrome
5

None of these options are correct
Q/Q(M)-477941 Report a Problem
133



A child presenting with the scalp findings shown and a right arm hypoplasia would be diagnosed with which of
the following syndromes?
1

Adams-Oliver syndrome
Adams-Oliver syndrome is defined by aplasia cutis congenita (ACC) (shown in the image), usually of the midline
scalp with limb hypoplasia. Bart's syndrome also has ACC as a finding, but it is usually present on the lower
extremities and associated dominant dystrophic epidermolysis bullosa. Progeria is a premature aging syndrome
associated with a lamin-A mutation. Dunnigan syndrome is also known as Familial partial lipodystrophy and is
associated with a mutation in the BSCL2 gene. Neither are associated with findings of ACC.
Q/Q(M)-477941 Report a Problem
A patient with a port wine stain covering one enlarged leg likely has which of the following associated
symptoms?
1

Lymphatic and deep venous insufficiency
2

Visceromegaly with omphalocele
3

Bilateral retinal hemangioblastomas
4

Enchondromas
5

Distichiasis
Q/Q(M)-474260 Report a Problem
A patient with a port wine stain covering one enlarged leg likely has which of the following associated
symptoms?
1

Lymphatic and deep venous insufficiency
Klippel-Trenaunay-Weber syndrome is a sporadic condition characterized by port-wine stains typically covering
one lower extremity that is enlarged with underlying lymphatic and deep venous insufficiency.
Q/Q(M)-474260 Report a Problem
Medulloblastoma is seen in which syndrome?
134

1

Gardners syndrome
2

Multiple endocrine neoplasia 2b
3

Muir-Torre syndrome
4

Basal cell nevus syndrome
5

Neurofibromatosis Type 1
Q/Q(M)-477145 Report a Problem
Medulloblastoma is seen in which syndrome?
4

Basal cell nevus syndrome
Basal cell nevus syndrome is an autosomal dominant syndrome caused by a mutation in PTC gene, which acts in
the Sonic hedgehog pathway. Cutaneous manifestations of this genodermatosis include basal cell carcinomas,
palmoplantar pits, epidermoids cysts. Other findings include odotogenic cysts, frontal bossing, bifid ribs,
calcification of the falx cerebri and medulloblastomas.
Q/Q(M)-477145 Report a Problem
Which of the following is NOT part of the Carney complex?
1

Peg or conical teeth
2

Cardiac, cutaneous or mammary myxomas
3

Pigmented skin lesions
4

Endocrine abnormalities
5

Primary pigmented nodular adrenocortical disease
Q/Q(M)-477778 Report a Problem
Which of the following is NOT part of the Carney complex?
1

Peg or conical teeth
Peg/conical teeth are not part of the Carney complex. This is found in incontinentia pigmenti and anhidrotic
ectodermal dysplasia. The remaining skin findings are part of this complex sometimes known as NAME
syndrome. It consists of multiple, diffuse mucocutaneous lentigines, cardiac and subcutaneous myxomas and
endocrine abnormalities may be present. Other findings include: testicular tumors, thyroid disease, primary
pigmented nodular adrenocortical disease, psammomatous melanotic schwannomas and hormone-secreting
pituitary adenomas.
135

Q/Q(M)-477778 Report a Problem
An infant with a very hoarse cry presents to dermatology. His parents have noted that normal scratches are
slow to heal and leave broad scars. White and yellow lesions have been noted on the lips, eyelids, and buccal
mucosa. The child has had frequent upper respiratory tract infections. What defective gene is associated with
the most likely implicated disease?
1

Extracellular matrix protein-1
2

Collagen III
3

Elastin
4

Tenascin-X
5

Fibrillin-1
Q/Q(M)-482614 Report a Problem
An infant with a very hoarse cry presents to dermatology. His parents have noted that normal scratches are
slow to heal and leave broad scars. White and yellow lesions have been noted on the lips, eyelids, and buccal
mucosa. The child has had frequent upper respiratory tract infections. What defective gene is associated with
the most likely implicated disease?
1

Extracellular matrix protein-1
The child described most likely has lipoid proteinosis, a rare inherited disease of connective tissues that is
inherited in an autosomal recessive manner. The implicated defective protein is extracellular matrix protein-1.
Collagen III and Tenascin-X defects are seen in Ehlers-Danlos syndrome. Elastin defects are seen in cutis laxa.
Fibrillin-1 defects are seen in Marfan disease.
Q/Q(M)-482614 Report a Problem
A patient with renal cell carcinoma caused by mutations in fumarate hydratase deficiency likely suffers which of
the following conditions?
1

Von-Hippel-Lindau syndrome
2

Cowden syndrome
3

Birt-Hogg-Dube syndrome
4

Familial multiple cutaneous leiomyomatosis
5

Multiple endocrine neoplasia
Q/Q(M)-474291 Report a Problem
136

A patient with renal cell carcinoma caused by mutations in fumarate hydratase deficiency likely suffers which of
the following conditions?
4

Familial multiple cutaneous leiomyomatosis
Familial multiple cutaneous leiomyomatosis is an autosomal dominant condition caused by mutations in the
fumarate hydratase gene. Clinically, there are multiple cutaneous leiomyomas, uterine leiomyomas and
leiomyosarcomas, as well as renal cell carcinomas.
Q/Q(M)-474291 Report a Problem
Mutation in lamin A (nuclear envelope protein) has been found in:
1

Peutz-Jeghers syndrome
2

Buschek-Ollendorf syndrome
3

Progeria (Hutchinson-Gilford)
4

Albrights syndrome
5

Marfan syndrome
Q/Q(M)-473974 Report a Problem

Mutation in lamin A (nuclear envelope protein) has been found in:
3

Progeria (Hutchinson-Gilford)
Progeria (Hutchinson-Gilford syndrome) is a sporadic condition characterized by lipoatrophy, sclerodermoid
skin, alopecia, nail atrophy, craniomegaly with small face, muscle/bone wasting, and severe premature
atherosclerosis resulting in early death. Recent studies have shown that mutations in nuclear envelope protein
lamin A is associated with progeria.
Q/Q(M)-473974 Report a Problem
A 11 year-old female patient with hypoparathyroidism is referred to your clinic secondary to chronic
mucocutanous candidiasis which is refractory to standard treatments. The patient also has malabsorption and
severe chronic diarrhea. You determine that she has autoimmune polyendocrinopathy-candiasis-ectodermal
dystrophy syndrome(APECED). What is the gene defect for APECED?
1

XAP101 gene
2

AIRE gene
3

CGI-58 gene
4

ALOX gene
137

5

PAHX gene
Q/Q(M)-481619 Report a Problem
A 11 year-old female patient with hypoparathyroidism is referred to your clinic secondary to chronic
mucocutanous candidiasis which is refractory to standard treatments. The patient also has malabsorption and
severe chronic diarrhea. You determine that she has autoimmune polyendocrinopathy-candiasis-ectodermal
dystrophy syndrome(APECED). What is the gene defect for APECED?
2

AIRE gene
The gene responsible for APECED is the AIRE gene, an autoimmune regulator. The XAP101 encodes dyskerin, a
defect in which causes dyskeratosis congenita. A defect in CGI-58 causes Chanarin-Dorfman or neutral lipid
storage disease with ichthyosis. Defect in the ALOX genes cause congenital ichthyosiform erythroderma. A
defect in the PAHX gene causes Refsum syndrome.
Q/Q(M)-481619 Report a Problem

A patient with multiple sebaceous adenomas should be screened with which of the following examinations?
1

Retinal examination
2

Laryngoscopy
3

Colonoscopy
4

MRI of the spine
5

Renal ultrasound
Q/Q(M)-474294 Report a Problem

A patient with multiple sebaceous adenomas should be screened with which of the following examinations?
3

Colonoscopy
Muir-Torre syndrome is an autosomal dominant disorder caused by the HMSH2 and MLH1 DNA mismatch
repair genes. Clinically, there are numerous sebaceous adenomas, epitheliomas and carcinomas and multiple
keratoacanthomas associated with indolent colon and other visceral adenocarcinomas. Patients and first-
degree relatives should be screened by colonoscopy as colonic adenocarcinomas may precede the
development of cutaneous tumors.
Q/Q(M)-474294 Report a Problem
In biopsies from blisters in patients with junctional epidermolysis bullosa, the split is found in the:
1

Basal cell layer of the epidermis
138

2

Lamina lucida
3

Lamina densa
4

Squamous cell layer of the epidermis
5

None of the answers are correct
Q/Q(M)-477711 Report a Problem

In biopsies from blisters in patients with junctional epidermolysis bullosa, the split is found in the:
2

Lamina lucida
The split seen in junctional epidermolysis bullosa is in the lamina lucida. The other locations can be involved in
blistering disease, but not junctional epidermolysis bullosa.
Q/Q(M)-477711 Report a Problem

Yellow-brown depositions in Descemet's membrane of the corneas is diagnostic for:
1

Hemochromatosis
2

Neurofibromatosis
3

Hepatolenticular degeneration
4

Pseudoxanthoma elasticum
5

Diabetes mellitus
Q/Q(M)-478063 Report a Problem

Yellow-brown depositions in Descemet's membrane of the corneas is diagnostic for:
3

Hepatolenticular degeneration
Hepatolenticular degeneration or Wilson Disease is caused by a defect in biliary excretion of copper leading to
accumulation in the liver, brain, cornea, pretibial hyperpigmentation, hepatomegaly and cirrhosis. The Kayser-
Fleischer ring is the yellow-brown copper deposition in Descemet's membrane of the cornea. Other findings
include ataxia, dysarthria and dementia. Hemochromatosis does not have an eye finding. Neurofibromatosis
has Lisch nodules and Pseudoxanthoma elasticum has angioid streaks of the retina.
Q/Q(M)-478063 Report a Problem
139

A teenage female presents with the complaint of "nail fungus". On exam, she has triangular lunulae,
palmoplantar hyperhidrosis, micronychia and an absent patella. Which of the following screening tests should
you order first?
1

Urinalysis
2

CBC
3

Fasting lipids
4

Renal ultrasound
5

X-ray of the knees, elbows and pelvis
Q/Q(M)-481626 Report a Problem
A teenage female presents with the complaint of "nail fungus". On exam, she has triangular lunulae,
palmoplantar hyperhidrosis, micronychia and an absent patella. Which of the following screening tests should
you order first?
1

Urinalysis
Patients with nail-patella syndrome can have glomerulonephritis and renal dysplasia leading to renal failure.
Screening with a urinalysis is a reasonable first test. If this is abnormal, referral for a renal ultrasound could be
useful. Fasting lipids and a CBC are not indicated.
Q/Q(M)-481626 Report a Problem
The porphyrias are a group of diseases related by abnormal heme synthesis. Which is the only porphyria that is
inherited in an autosomal recessive manner?
1

Congenital erythropoietic porphyria
2

Variegate porphyria
3

Porphyria cutanea tarda
4

Acute intermittent porphyria
5

Erythropoietic protoporphyria
Q/Q(M)-482618 Report a Problem

The porphyrias are a group of diseases related by abnormal heme synthesis. Which is the only porphyria that is
inherited in an autosomal recessive manner?
1

Congenital erythropoietic porphyria
140

The porphyrias are generally inherited in an autosomal dominant manner. Congenital erythropoietic porphyria
is unique in being inherited in an autosomal recessive manner. Helpful mnemonic: all porphyrias are autosomal
dominant exCEPt one.
Q/Q(M)-482618 Report a Problem

A triangular-shaped lunula is a characteristic finding in which disease?
1

Darier's disease
2

Dyskeratosis congenita
3

Epidermal nevus syndrome
4

Incontinentia pigmenti
5

Nail-patella syndrome
Q/Q(M)-475862 Report a Problem

A triangular-shaped lunula is a characteristic finding in which disease?
5

Nail-patella syndrome
Nail-patella syndrome, also called hereditary osteo-onychodysplasia is a rare autosomal dominant condition
caused by a defect in the LMX1B gene. It is characterized by triangular lunulae, palmoplantar hyperhidrosis,
renal dysplasia, glomerulonephritis, and hyperpigmentation of the papillary margin of the iris, an
ophthalmologic finding also known as Lester iris. Other nail findings include micronychia with hemionychia,
anonychia, and longitudinal fissures. Bony findings include absent or hypoplastic patella, posterior iliac horns,
radial head subluxation, thickened scapulae, and scoliosis. Nail findings in Darier's disease include red and
white longitudinal bands, subungual hyperkeratosis and V-shaped nicking of the distal nail plate. Dystrophic
nails with longitudinal ridges, pterygium, and atrophic or absent nails can be found in dyskeratosis congenita.
Dystrophic changes of the nails can be seen in approximately 5-10% of patients with incontinentia pigmenti.
Q/Q(M)-475862 Report a Problem

What cutaneous manifestation is associated with familial cerebral cavernomas?
1

Verrucous hemangioms
2

Glomeruloid hemangiomas
3

hyperkeratotic cutaneous capillary-venous malformations(HCCVM)
4

segmental facial hemangiomas
141

5

Tufted angiomas
Q/Q(M)-477482 Report a Problem
What cutaneous manifestation is associated with familial cerebral cavernomas?
3

hyperkeratotic cutaneous capillary-venous malformations(HCCVM)
Familial cerebral cavernomas are due to a defect in the CCM gene which encodes the KRIT-1 protein. These
patients often times have hyperkeratotic cutaneous capillary-venous malformations.
Q/Q(M)-477482 Report a Problem
A 2 year old girl presents with sunken eyes, large ears, microcephaly and a photodistributed eruption on her
face. Eye exam reveals salt and pepper retina. The gene responsible for this syndrome codes for a:
1

Transcription factor
2

Surface glycoprotein
3

Lysosomal protease
4

DNA helicase
5

Mismatch repair gene
Q/Q(M)-474037 Report a Problem

A 2 year old girl presents with sunken eyes, large ears, microcephaly and a photodistributed eruption on her
face. Eye exam reveals salt and pepper retina. The gene responsible for this syndrome codes for a:
4

DNA helicase
The patient described has Cockayne syndrome, an autosomal recessive disorder believed to be due to a
mutation in either DNA helicase or defective nucleotide excision repair. UV irradiated cells have decreased DNA
and RNA synthesis and increased chromosomal breaks. Clinical features include cachectic dwarfism with
microcephaly, thin nose, large ears, photosensitive eruption, cataracts, salt & pepper retina, and diffuse
demyelination.
Q/Q(M)-474037 Report a Problem
Which of the following is caused by a defect in cathepsin C?
1

Rubenstein-Taybi
2

Bullous Icthyosis of Siemens
3

Muir-Torre
142

4

Papillon-Lefevre
5

Nail-Patella syndrome
Q/Q(M)-474022 Report a Problem
Which of the following is caused by a defect in cathepsin C?
4

Papillon-Lefevre
Papillon-Lefevre is a palmoplantar keratoderma caused by a mutation in chromosome 11q14, leading to a
defect in Cathepsin C, a lysosomal enzyme. Clinical manifestations of papillon lefevre include sharply
demarcated palmoplantar keratoderma with extension to dorsal surface (transgrediens), spare hair,
periodontitis, and pyoderma.
Q/Q(M)-474022 Report a Problem

The NEMO gene is defective in Bloch-Sulzberger syndrome. What other syndrome has been linked with defects
in the NEMO gene?
1

Hypohidrotic ectodermal dysplasia with immune deficiency
2

Hypomelanosis of Ito
3

Tuberous sclerosis
4

Waardenburg syndrome
5

Piebaldism
Q/Q(M)-477747 Report a Problem

The NEMO gene is defective in Bloch-Sulzberger syndrome. What other syndrome has been linked with defects
in the NEMO gene?
1

Hypohidrotic ectodermal dysplasia with immune deficiency
Hypohidrotic ectodermal dysplasia with immune deficiency, is caused by mutations in the NEMO (IKK-gamma
gene). As opposed to the X-linked dominant inheritance of Bloch-Sulzberger syndrome (incontinentia
pigmenti), this is a X-linked recessive disorder. Hypomelanosis of Ito is sporadically inherited and is not linked
with a gene defect. Tuberous sclerosis is autosomal dominant and has been linked to defects in tuberin and
hamartin tumor suppressor genes. Waardenburg syndrome has four subtypes, linked with the PAX-3, MITF and
SOX10/endothelin-3 receptor genes. Piebaldism is linked to defects in the c-kit protooncogene.
Q/Q(M)-477747 Report a Problem
143


Xeroderma pigmentosum (XP) variant is different than classic XP in which of the following way?
1

Defective DNA nucleotide excision repair of the global genome
2

Defective post-replication repair
3

Increased chromosomal breakage and sister chromatid exchanges
4

Defective DNA nucleotide excision repair of actively transcribing genes
5

Low IgM
Q/Q(M)-477706 Report a Problem
Xeroderma pigmentosum (XP) variant is different than classic XP in which of the following way?
2

Defective post-replication repair
XP variant is DNA nucleotide excision repair proficient, but the defect is in post replication repair of DNA.
Increased chromosomal breakage and sister chromatid exchanges is found in Blooms syndrome, an
autosomal recessive syndrome caused by a defect in BLM gene, whose product functions as a helicase. Clinical
findings include: Telangiectasias, short stature, malar erythema, recurrent infection, increased frequency of
leukemia and lymphoma, normal intelligence. Defective DNA nucleotide excision repair of actively transcribing
genes is a feature of Cockaynes syndrome, an autosomal recessive syndrome with clinical findings including:
Cachexia, short stature, pigmentary retinal degeneration, progressive deafness and no increase in neoplasms.
Xeroderma pigmentosum has seven different complementation groups (A-G), each associated with a different
form of impairment of DNA nucleotide excision repair.
Q/Q(M)-477706 Report a Problem
Low-cystine content in hair and nails may contribute to the phenotype seen in:
1

Wilsons disease
2

Menkes Kinky Hair syndrome
3

Tay Syndrome
4

Nethertons
5

Bjornstad
Q/Q(M)-474047 Report a Problem
Low-cystine content in hair and nails may contribute to the phenotype seen in:
3

Tay Syndrome
144

Tay Syndrome is also known as trichothiodystrophy, or (P)IBIDS: (photosensitivity), icthyosis, brittle hair,
intellectual impairment, decreased fertility, and short stature. Hair shaft has a characteristic tiger tail
appearance under polarized light and the low cystine content in hair and mails is thought to be responsible for
the phenotype seen.
Q/Q(M)-474047 Report a Problem

A 12 year old boy presents complaining of pain and stinging of his skin within minutes of going out to play in
the park. Examination reveals rare milia of the hands and posterior neck. A diagnosis of porphyria is made.
What is the most likely defective gene in this patient?
1

Ferrochelatase
2

Uroporphyrinogen decarboxylase
3

Porphybilinogen deaminase
4

Uroporphyrinogen III synthase
5

Protoporphyrinogen oxidase
Q/Q(M)-482616 Report a Problem
A 12 year old boy presents complaining of pain and stinging of his skin within minutes of going out to play in
the park. Examination reveals rare milia of the hands and posterior neck. A diagnosis of porphyria is made.
What is the most likely defective gene in this patient?
1

Ferrochelatase
Erythropoietic protoporphyria is a relatively common form of porphyria, characterized by photosensitivity.
Over time, superficial waxy scars and milia are identified in sun-exposed areas. The defective gene is
ferrochelatase. Uroporphyrinogen decarboxylase is mutated in porphyria cutanea tarda. Porphybilinogen
deaminase is defective in acute intermittent porphyria. Uroporphyrinogen III synthase is defective in congenital
erythropoietic porphyria. Protoporphyrinogen oxidase is mutated in variegate porphyria.
Q/Q(M)-482616 Report a Problem
Which syndrome is characterized by broad thumbs, a large beaked nose, and capillary malformation?
1

Klinefelter
2

Proteus syndrome
3

Bloom syndrome
4

Rubinstein-Taybi
145

5

Ehlers-Danlos syndrome
Q/Q(M)-474016 Report a Problem
Which syndrome is characterized by broad thumbs, a large beaked nose, and capillary malformation?
4

Rubinstein-Taybi
Rubinstein-Taybi syndrome has been associated with a deletion localized to the short arm of chromosome 16.
Patients are severely retarded with strabismus, crytorchidism, and congenital heart defects. They have a
characteristic beaked nose with nasal septum below alae accompanied by a broad nasal bridge, downslanting
palpebral fissures, and broad thumbs and halluces.
Q/Q(M)-474016 Report a Problem
Bilateral ovarian cystadenomas and parotid monomorphic adenomas are seen in which of the following
conditions?
1

Cowden syndrome
2

Proteus syndrome
3

Von-Hippel-Lindau syndrome
4

Noonan syndrome
5

Beckwith-Wiederman syndrome
Q/Q(M)-474267 Report a Problem
Bilateral ovarian cystadenomas and parotid monomorphic adenomas are seen in which of the following
conditions?
2

Proteus syndrome
Proteus syndrome is a sporadic condition caused by postzygotic mosaic mutations in PTEN. Clinical features
include subcutaneous lymphovenous malformations, capillary malformations, lipomas, connective tissue nevi
of palms/soles, hemihypertrophy, frontal bossing, hyperostoses of epiphyses and skull (esp. external auditory
canal), scoliosis, bilateral ovarian cystadenomas, and parotid monomorphic adenomas.
Q/Q(M)-474267 Report a Problem

Pruritus is Sjogren Larsson syndrome is attributed to accumulation of what molecule(s)?
1

Bile salts
2

Histamine
146

3

Leukotriene
4

All of these answers are correct
5

None of these answers are correct
Q/Q(M)-477856 Report a Problem
Pruritus is Sjogren Larsson syndrome is attributed to accumulation of what molecule(s)?
3

Leukotriene
Accumulation of leukotriene B4 contributes to pruritus in Sjogren Larsson syndrome. Leukotriene inhibitors
may be helpful in controlling symptoms.
Q/Q(M)-477856 Report a Problem
Pili trianguli et canaliculi is characteristic of which of the following syndromes?
1

Uncombable hair syndrome
2

Bjornstad syndrome
3

Menkes kinky hair syndrome
4

Leiners disease
5

Netherton's syndrome
Q/Q(M)-477946 Report a Problem
Pili trianguli et canaliculi is characteristic of which of the following syndromes?
1

Uncombable hair syndrome
Uncombable hair syndrome is characterized by pili trianguli et canaliculi. On examination of the hair, it is
triangular with a canal-like groove runs along the shaft. The clinical findings are that of blond, shiny, "spun
glass" hair. It is an autosomal dominant syndrome with no known gene locus. The remaining syndromes do not
have this hair finding present.
Q/Q(M)-477946 Report a Problem
What is the underlying gene defect for this transgrediens form of palmoplanter keratoderma
1

SLURP-1
2

TOC gene
3

Plakoglobin
147

4

Keratin type 1
5

Keratin type 9
Q/Q(M)-482117 Report a Problem

What is the underlying gene defect for this transgrediens form of palmoplanter keratoderma
1

SLURP-1
Attached picture is Mal de Meleda (keratosis palmoplantaris transgrediens) which is an autosomal recessive
form of diffuse PPK, associated with keratotic plaques that extend to the dorsal aspects of the hands and feet
("transgrediens") and may overlie joints . Hyperhidrosis, superinfection, and occasionally perioral erythema,
brachydactyly, and nail abnormalities are associated. Mal de Meleda is due to mutations in ARSB, which
encodes SLURP-1. The other choices represent gene defects for "non-transgrediens" forms of PPK (Plakoglobin
in Naxos syndrome, TOC gene in Howel-Evans syndrome, K1 in non-epidermolytic PPK "Unna-Thost", and K9 in
epidermolytic PPK "Vorner"
Q/Q(M)-482117 Report a Problem

A patient with coarse facies, broad nasal bridge, and extensive eczema might be expected to have which
abnormal laboratory value?
1

Hypertriglyceridemia
2

High Copper levels
3

Low Zinc levels
4

High IgE levels
5

Anemia
Q/Q(M)-474032 Report a Problem
A patient with coarse facies, broad nasal bridge, and extensive eczema might be expected to have which
abnormal laboratory value?
148

4

High IgE levels
Patients with Hyper IgE Syndrome (Job syndrome) have impaired regulation of the IgE function and
increased susceptibility to infections. In addition to recurrent cutaneous infections including cold abscesses,
patients have widespread eczematous dermatitis, recurrent sinopulmonary infections and typically have coarse
facies with broad nasal bridge and a prominent nose.
Q/Q(M)-474032 Report a Problem
Which of the following syndromes is associated with hematologic abnormalities?
1

Sturge Weber disease
2

Klippel-Trenaunay-Parks-Weber
3

Kasabach-Merritt syndrome
4

Blue rubber bleb nevus syndrome
5

Blooms syndrome
Q/Q(M)-477120 Report a Problem
Which of the following syndromes is associated with hematologic abnormalities?
3

Kasabach-Merritt syndrome
Kasabach-Merritt syndrome is associated with hematologic abnormalities, such as thrombocytopenia,
microangiopathic hemolytic anemia, disseminated intravascular coagulation. The condition develops from
platelet-trapping within a large hemangioma, most commonly a kaposiform hemangioendothelioma in the
retroperitoneal location.
Q/Q(M)-477120 Report a Problem
A mentally retarded patient is found to also have a malar rash and downward lens displacement. Another
clinical feature might include:
1

Hypertrichosis
2

Short stature
3

Multiple nevi
4

Periodontitis
5

Deep venous thromboses
Q/Q(M)-474026 Report a Problem
149

A mentally retarded patient is found to also have a malar rash and downward lens displacement. Another
clinical feature might include:
5

Deep venous thromboses
The above described patient has homocystinuria, an autosomal recessive disorder caused by a deficiency in
cystathionine beta-synthase, leading to an accumulation of homocysteine. Clinical features include: malar flush,
DVTs, livedo reticularis, leg ulcers, downward lens displacement, myopia, glaucoma, sparse fine hair, mental
retardation, and musculoskeletal anomalies.
Q/Q(M)-474026 Report a Problem
Which of the following immunoglobulins is commonly decreased in Wiskott-Aldrich syndrome?
1

IgA
2

IgD
3

IgE
4

IgM
5

IgG
Q/Q(M)-478026 Report a Problem
Which of the following immunoglobulins is commonly decreased in Wiskott-Aldrich syndrome?
4

IgM
IgM is decreased in WAS. IgA, IgD and IgE levels are all elevated. IgG is not abnormal in WAS.
Q/Q(M)-478026 Report a Problem
Which malignancy is seen in approximately 15-20% of people with the disease characterized by a defect in a
parathyroid hormone receptor protein?
1

Osteosarcoma
2

Angiosarcoma
3

Chondrosarcoma
4

Rhabdomyosarcoma
5

Epitheliod sarcoma
Q/Q(M)-477511 Report a Problem
150

Which malignancy is seen in approximately 15-20% of people with the disease characterized by a defect in a
parathyroid hormone receptor protein?
3

Chondrosarcoma
Approximately 15-20% of patients with Maffucci's syndrome will develop chondrosarcoma. Maffucci's
syndrome is due to a defect in a parathyroid hormone receptor protein.
Q/Q(M)-477511 Report a Problem

The diagnostic test for chronic granulomatous disease is:
1

Dimethylglyoxime test
2

Nitroblue tetrazolium reduction assay
3

Histamine skin test
4

Bone marrow biopsy
5

Potassium hydroxide
Q/Q(M)-474034 Report a Problem

The diagnostic test for chronic granulomatous disease is:
2

Nitroblue tetrazolium reduction assay
Chronic granulomatous disease is characterized by a defect in the ability to kill catalase positive organisms
within phagocytic leukocytes. This results from a neutrophilic defect in the cytochrome found in the NADPH
oxidative pathway responsible for a respiratory burst. The nitroblue tetrazolium (NBT) reduction assay
demonstrates the leukocyes ability to reduce the dye and produce a blue color change. Patients with
chronic granulomatous disease are unable to reduce the dye.
Q/Q(M)-474034 Report a Problem
Dental enamel pits are seen in which of the following conditions?
1

Hypomelanosis of Ito
2

Letterer-Siwe disease
3

Tuberous sclerosis
4

Jackson Sertoli syndrome
5

Hyper-IgE syndrome
151

Q/Q(M)-477739 Report a Problem

Dental enamel pits are seen in which of the following conditions?
3

Tuberous sclerosis
Tuberous sclerosis is an autosomal dominant condition caused by mutations of the TSC1 (hamartin) or TSC2
(tuberin) genes. These are tumor suppressor genes. Skin findings include hypopigmented macules, connective
tissue nevus, facial angiofibromas, periungual fibromas and cafe au lait macules. Dental enamel pits and
gingival fibromas are oral findings that are associated with this condition. The remaining conditions do not have
dental pits.
Q/Q(M)-477739 Report a Problem
Beckwith-Wiederman syndrome is characterized by which of the following triads?
1

Hemangioblastomas, renal cysts and renal cell carcinoma
2

Epistaxis, telangictases, and gastrointestinal tract bleeding
3

Enlarged limb, port wine stain, and deep venous thrombosis
4

Omphalocele, venous malformations, and ataxia
5

Exomphalos, macroglossia, and gigantism
Q/Q(M)-474263 Report a Problem
Beckwith-Wiederman syndrome is characterized by which of the following triads?
5

Exomphalos, macroglossia, and gigantism
Beckwith-Wiederman syndrome is also known as EMG syndrome as it includes exomphalos, macroglossia, and
gigantism. It is usually a sporadic condition but is sometimes caused by autosomal dominant mutations in p57.
Clinical features include facial capillary malformations, macroglossia, visceromegaly with omphalocele, and
hemihypertrophy associated with tumors (especially Wilms tumors).
Q/Q(M)-474263 Report a Problem

In chronic granulomatous disease, the diagnosis is made by which of the following tests?
1

Assay for fumarate hydratase
2

Nitroblue tetrazolium reduction assay
3

Assay for sphingomyelinase
152

4

Skin biopsy
5

Assay for glucocerebrosidase
Q/Q(M)-478038 Report a Problem
In chronic granulomatous disease, the diagnosis is made by which of the following tests?
2

Nitroblue tetrazolium reduction assay
Chronic granulomatous disease is diagnosed by the nitroblue tetrazolium reduction assay. The abnormal white
blood cells cannot reduce dye due to the inability to produce the respiratory burst. This is needed to kill
catalase positive organisms after phagocytosis. Fumarate hydratase is defective in familial multiple cutaneous
leiomyomatosis, sphingomyelinase in Niemann-Pick disease and glucocerebrosidase in Gaucher disease.
Q/Q(M)-478038 Report a Problem

Potrebbero piacerti anche